You are on page 1of 193

目 录

1. Figure Analysis ........................................................................................1

2. Figure Recognition ................................................................................75


1. Figure Analysis
1. Explanation
The correct answer is 5.
 First, the rectangular paper is folded in half lengthwise.
 Then, a small triangle is cut out of the folded paper.
 Consequently, the unfolded paper will have two triangle cut-outs which
eliminates answer choices 3 and 4.
 Since the paper is folded lengthwise, the second triangle must mirror
horizontally the cut triangle which eliminates answer choices 1 and 2.
We are left with the 5th choice, which is the correct answer.

2. Explanation
The correct answer is 2.
 First, the rectangular paper is folded in half lengthwise.
 Then, a diagonal is cut out of the upper-left and lower-right corners of the
folded paper.
 Consequently, the unfolded paper will have four diagonal cut-outs.
Since the paper is folded lengthwise, the second cut-outs must mirror horizontally the
original cut-outs. This eliminates answer choices 1, 3, 4, and 5.
We are left with the 2nd choice, which is the correct answer.

3. Explanation
The correct answer is 5.
 First, the square paper is folded in half to make a triangle.
 Then, a pentagon and parallelogram are cut out of the folded paper.
 Consequently, there will be two pentagon cut-outs and two parallelogram
cut-outs when the paper is unfolded again.
 Also, the second pentagon and second parallelogram must be rotated 90°
clockwise and flipped vertically.
since the paper was folded along the diagonal. This eliminates answer choices 1, 2, 3,
and 4.
We are left with the 5th choice, which is the correct answer.

4. Explanation
The correct answer is 3.
First, the rectangular paper is folded in half lengthwise. Then, a rectangle, triangle and
oval are cut out of the folded paper. Consequently, the unfolded paper will have two
rectangle cut-outs, two triangle cut-outs and two oval cut-outs. Since the paper is
folded lengthwise, the second rectangle, second triangle and second oval must mirror
horizontally the cut rectangle, triangle and oval which eliminates answer choices 1, 2,
4 and 5.

1
5. Explanation
The correct answer is 5.
 First, the rectangular paper is folded in half widthwise and then folded in half
lengthwise.
 Then, one triangle is cut out of the folded paper.
 Consequently, when the paper is unfolded lengthwise the paper will have two
triangle cut-outs. When the paper is then unfolded widthwise the paper will
have four triangle cut-outs.
Since the paper is folded widthwise and then lengthwise, the second triangle must
mirror horizontally the cut triangle, and then the last two triangles must mirror
vertically the first two triangles. This eliminates answer choices 1, 2, 3, and 4.
Therefore, we are left with the 5th choice, which is the correct answer.

6. Explanation
The correct answer is 5.
 First, the rectangular paper is folded in half lengthwise and then folded in half
widthwise.
 Then, a triangle and pie are cut out of the folded paper.
 Consequently, when the paper is unfolded widthwise the paper will have two
triangle cut-outs and two pie cut-outs.
 When the paper is then unfolded lengthwise the paper will have four triangle
cut-outs and four pie cut-outs.
Since the paper is folded lengthwise and then widthwise, the second triangle and
second pie must mirror vertically the cut triangle and pie, and then the last two
triangles and two pies must mirror horizontally the first two triangles and pies. This
eliminates answer choices 1, 2, 3, and 4.
Thus, the only possible answer is the 5th choice, and it is the correct answer.

7. Explanation
The correct answer is 1.
 First, the square paper is folded in half widthwise and then folded in half
lengthwise.
 Then, one arrow, triangle, and rectangle shape are cut out of the folded paper.
 Consequently, when the paper is unfolded lengthwise the paper will have two
arrow cut-outs, two triangle cut-outs and two rectangle cut-outs.
 When the paper is then unfolded widthwise the paper will have four arrow
cut-outs, four triangle cut-outs, and four rectangle cut-outs.
Since the paper is folded widthwise and then lengthwise, the second arrow, second
triangle, and second rectangle shapes must mirror horizontally the cut arrow, triangle,
and rectangle, and then the final two sets of the arrow, triangle, and rectangle shapes
must mirror vertically the first two sets of arrow, triangle, and rectangle shapes. This
eliminates answer choices 2, 3, 4, and 5.
Thus, the only remaining possible answer is the 1st choice, and it is the correct
answer.

2
8. Explanation
The correct answer is 1.
 First, the rectangular paper is folded in half widthwise and then folded in half
lengthwise.
 Then, one heart and two rectangles are cut out of the folded paper.
 Consequently, when the paper is unfolded lengthwise the paper will have two
heart cut-outs and four rectangle cut-outs.
 When the paper is then unfolded widthwise the paper will have four heart
cut-outs and eight rectangle cut-outs.
Since the paper is folded widthwise and then lengthwise, the second heart and second
two rectangles must mirror horizontally the cut heart and two rectangles and then the
last two hearts and four rectangles must mirror vertically the first two hearts and four
rectangles. This eliminates answer choices 2, 3, 4, and 5.
The remaining answer is the 1st choice and it is, therefore, the correct answer.

9. Explanation
The correct answer is 4.
 First, the rectangular paper is folded in half widthwise and then folded in half
lengthwise.
 Then, one trapezoid, triangle, and rectangle shape are cut out of the folded
paper.
 Consequently, when the paper is unfolded lengthwise the paper will have two
trapezoid cut-outs, two triangle cut-outs, and two rectangle cut-outs.
 When the paper is then unfolded widthwise the paper will have four trapezoid
cut-outs, four triangle cut-outs, and four rectangle cut-outs.
Since the paper is folded widthwise and then lengthwise, the second trapezoid, second
triangle, and second rectangle shapes must mirror horizontally the cut trapezoid,
triangle, and rectangle shapes, and then the final two sets of the trapezoid, triangle,
and rectangle shapes must mirror vertically the first two sets of trapezoid, triangle and
rectangle shapes. This eliminates answer choices 1, 2, 3, and 5.
Therefore, the 4th choice is the correct answer.

10. Explanation
The correct answer is 2.
 First, the rectangular paper is folded in half widthwise and then folded in half
lengthwise.
 Then, once the circle, rectangle, and triangle shapes are cut out of the folded
paper.
 Consequently, when the paper is unfolded lengthwise the paper will have two
circle cut-outs, two rectangle cut-outs, and two triangle cut-outs.
 When the paper is then unfolded widthwise the paper will have four circle
cut-outs, four rectangle cut-outs, and four triangle cut-outs.
Since the paper is folded widthwise and then lengthwise, the second circle, second
rectangle, and second triangle shapes must mirror horizontally the cut circle, rectangle,

3
and triangle shapes, and then the final two sets of the circle, rectangle, and triangle
shapes must mirror vertically the first two sets of circle, rectangle and triangle shapes.
This eliminates answer choices 1, 3, 4, and 5.
The only remaining option is the 2nd choice and it is, therefore, the correct answer.

11. Explanation
The correct answer is 1.
First the paper was folded diagonally from the lower-left corner to the upper-right
corner. Then it was folded again from the lower-right corner to the upper-left corner.
Then 3 holes in different shapes were punched out. Therefore, when the paper is
unfolded the holes will mirror each other across the diagonals of the page like this:

To solve this question, look for the answer choices in which the holes do not mirror
each other in the right way. The 2nd choice can be eliminated in this way as the
pie-shaped holes and triangular holes do not mirror each other.
Next, concentrate on the area of each remaining answer choice that appears in the
rightmost frame of the question series (in this case the triangular area on the top) and
eliminate all choices that are different. The 3rd choice can be eliminated as the
pie-shaped hole has been rotated. The 4th choice can also be eliminated as the
triangular hole has been rotated. Finally we can eliminate the 5th answer choice as the
pie-shaped and the triangular holes have switched places.
We are left with the 1st choice as the only correct answer.

12. Explanation
The correct answer is 1.
First the paper was folded diagonally from the lower-left corner to the upper-right
corner. Then it was folded again from the lower-right corner to the upper-left corner.
Then 3 holes in different shapes were punched out. Therefore, when the paper is
unfolded the holes will mirror each other across the diagonals of the page like this:

To solve this question, look for the answer choices in which the holes do not mirror
each other in the right way. The 2nd choice can be eliminated in this way as the
pie-shaped holes and triangular holes do not mirror each other.
Next, concentrate on the area of each remaining answer choice that appears in the
rightmost frame of the question series (in this case the triangular area on the top) and
eliminate all choices that are different. The 3rd choice can be eliminated as the

4
pie-shaped hole has been rotated. The 4th choice can also be eliminated as the
triangular hole has been rotated. Finally we can eliminate the 5th answer choice as the
pie-shaped and the triangular holes have switched places.
We are left with the 1st choice as the only correct answer.

13. Explanation
The correct answer is 2.
First, the rectangular paper is folded in half to make a triangle. Then, a trapezoid and
rectangle are cut out of the folded paper. Consequently, there will be two trapezoid
cut-outs and two rectangle cut-outs when the paper is unfolded again which eliminates
answer choices 1, 3 and 4. Also, the second trapezoid and second rectangle must be
rotated 90° clockwise and flipped vertically since the paper was folded along the
diagonal. This eliminates answer choice 5.

14. Explanation
The correct answer is 1.
 First, the square paper is folded in half widthwise.
 Then, two pie shapes are cut out of the folded paper.
 Therefore, the unfolded paper will have four pie shape cut-outs.
 Since the paper is folded widthwise, the shapes on the bottom must mirror the
cut pie shapes on the top.
In the 2nd answer choice, the pies at the bottom do not mirror the pies at the top.
In the 3rd answer choice, the pies at the bottom are exactly the same as the pies at the
top, and in the 4th and 5th answer choice, the top pies do not resemble the pies in the
questions.
We are left with the 1st choice as the correct answer.

15. Explanation
The correct answer is 2.
 First, the rectangular paper is folded in half to make a triangle.
 Then, three triangles are cut out of the folded paper.
 Consequently, there will be six triangle cut-outs when the paper is unfolded
again.
 Also, the three additional triangles must be rotated 90° clockwise and flipped
vertically since the paper was folded along the diagonal. This eliminates
answer choices 1, 3, 4, and 5.
We are left with the 2nd choice, which is the correct answer.

16. Explanation
The correct answer is 2.
 First, the rectangular paper is folded in half widthwise.
 Then, two rectangles and an oval are cut out of the folded paper.
 Consequently, the unfolded paper will have four rectangle cut-outs and two
oval cut-outs.

5
Since the paper is folded widthwise, the second two rectangles and second oval must
mirror vertically the cut two rectangles and oval. This eliminates answer choices 1, 3,
4, and 5.
We are left with the 2nd choice, which is the correct answer.

17. Explanation
The correct answer is 2.
 First, the rectangular paper is folded in half lengthwise and then folded in half
widthwise.
 Then, a moon is cut out of the folded paper.
 Consequently, when the paper is unfolded widthwise the paper will have two
moon cut-outs.
 When the paper is then unfolded lengthwise the paper will have four moon
cut-outs.
Since the paper is folded lengthwise and then widthwise, the second moon must
mirror vertically the cut moon, and then the last two moons must mirror horizontally
the first two moons. This eliminates answer choices 1, 3, 4, and 5.
We are left with the 2nd choice, which is the correct answer.

18. Explanation
The correct answer is 2.
 First, the rectangular paper is folded in half widthwise and then folded in half
lengthwise.
 Then, three circular shapes are cut out of the folded paper.
 Consequently, when the paper is unfolded lengthwise the paper will have six
circular cut-outs.
 When the paper is then unfolded widthwise the paper will have twelve circular
cut-outs.
Since the paper is folded widthwise and then lengthwise, the second set of three
circular shapes must mirror horizontally the three cut circular shapes, and then the
final two sets of three circular shapes must mirror vertically the first two sets of three
circular shapes. This eliminates answer choices 1, 3, 4, and 5.
We are left with the 2nd choice, which is the correct answer.

19. Explanation
The correct answer is 4.

Every time a paper is folded, the layer of paper doubles. Thus, creating a hole, or in
this case two shapes and fold it once, will create two separate pairs of shapes.
Moreover, if the paper is folded twice, four holes will appear.
 The paper in our question has been folded in half widthwise then folded again
in half lengthwise.
 After which, one oval and one rectangle were cut out of the folded paper.

6
 Consequently, when the paper has been unfolded lengthwise the paper will
have two oval cut-outs and two rectangle cut-outs.
 When the paper is then unfolded widthwise, the paper will have four oval
cut-outs and four rectangle cut-outs.
Folded paper creates a symmetry line. Therefore, after unfolding the previously
folded paper and creating holes in it, the unfolded paper will portray a mirror image
along that same symmetry line.
Since the paper was folded widthwise then lengthwise, the second oval and second
rectangle must mirror horizontally.
Likewise, the oval and rectangle and then the last two ovals and two rectangles must
mirror vertically the first two ovals and rectangles. This eliminates answer choices 1,
2, 3, and 5.

20. Explanation
The correct answer is 3.
 First, the rectangular paper is folded in half lengthwise and then folded in half
widthwise.
 Then, a triangle and half-oval are cut out of the folded paper.
 Consequently, when the paper is unfolded widthwise the paper will have two
triangle cut-outs and two half-oval cut-outs.
 When the paper is then unfolded lengthwise the paper will have four triangle
cut-outs and four half-oval cut-outs.
 Since the paper is folded lengthwise and then widthwise, the second triangle
and second half-oval must mirror vertically the cut triangles and half-oval, and
then the last two triangles and two half-ovals must mirror horizontally the first
two triangles and half-ovals. This eliminates answer choices 1, 2, 4, and 5.
Therefore, 3 is the correct answer.

21. Explanation
The correct answer is 5.
 First, the rectangular paper is folded in half widthwise and then folded in half
lengthwise.
 Then, one oval, triangle, and half-oval shape are cut out of the folded paper.
 Consequently, when the paper is unfolded lengthwise the paper will have two
oval cut-outs, two triangle cut-outs, and two half-oval cut-outs.
 When the paper is then unfolded widthwise the paper will have four oval
cut-outs, four triangle cut-outs, and four half-oval cut-outs.
Since the paper is folded widthwise and then lengthwise, the second set of two oval,
two triangle, and two half-oval shapes must mirror horizontally the cut oval, triangle,
and half-oval shapes, and then the final two sets of oval, triangle, and half-oval shapes
must mirror vertically the first two sets of oval, triangle and half-oval shapes. This
eliminates answer choices 1, 2, 3, and 4.
The only remaining option is the 5th choice and it is, therefore, the correct answer.

7
22. Explanation
The correct answer is 3.
 First, the rectangular paper is folded in half widthwise and then folded in half
lengthwise.
 Then, one arrow and two rectangles are cut out of the folded paper.
 Consequently, when the paper is unfolded lengthwise the paper will have two
arrow cut-outs and four rectangle cut-outs.
 When the paper is then unfolded widthwise the paper will have four arrow
cut-outs and eight rectangle cut-outs.
Since the paper is folded widthwise and then lengthwise, the second arrow and second
two rectangles must mirror horizontally the cut arrow and two rectangles, and then the
last two arrows and four rectangles must mirror vertically the first two arrows and
four rectangles. This eliminates answer choices 1, 2, 4, and 5.
The only remaining answer is the 3rd choice and it is, therefore, the correct answer.

23. Explanation
The correct answer is 3.
 First, the paper was folded diagonally from the upper-left corner to the
lower-right corner.
 Then it was folded again from the lower-left corner to the upper-right corner.
 Then 3 holes in different shapes were punched out.
 Therefore, when the paper is unfolded the holes will mirror each other across
the diagonals of the page like this:

To solve this question look for the answer choices in which the holes do not mirror
each other in the right way. The 2nd and 4th choices can be eliminated in this way.

Next, concentrate on the area of each remaining answer choice that appears in the
rightmost frame of the question series (in this case the triangular area on the right side)
and eliminate all choices that are different. The 1st choice can be eliminated as the
circular hole is under the pentagon hole but should be above it. The 5th choice can
also be eliminated as the pentagon has been rotated.
We are left with the 3rd choice as the only correct answer.

24. Explanation
The correct answer is 5.
 First, the paper was folded diagonally from the upper-right corner to the
lower-left corner.
 Then it was folded again from the upper-left corner to the lower-right corner.

8
 Then 3 holes in different shapes were punched out.
 Therefore, when the paper is unfolded the holes will mirror each other across
the diagonals of the page like this:


To solve this question, look for the answer choices in which the holes do not mirror
each other in the right way. The 3rd choice can be eliminated in this way as the
triangular holes do not mirror each other.

Next, concentrate on the area of each remaining answer choice that appears in the
rightmost frame of the question series (in this case the triangular area on the bottom)
and eliminate all choices that are different. The 1st choice can be eliminated as the
triangular hole has been rotated. The 2nd choice can also be eliminated as the circular
and the triangular holes have switched places. Finally, we can eliminate the 4th choice
as the diamond hole has been rotated (and is now square-shaped).
We are left with the 5th choice as the only correct answer.

25. Explanation
The correct answer is 1.
 First, The square paper was folded in half widthwise
 Then, five holes were punched in the folded paper.
 Since the paper was folded in half once, the holes were cut through two layers
of paper.
 Therefore, when the paper is unfolded it will have ten holes.
 Since the paper was folded widthwise down the middle, the holes on the
bottom fold must mirror the holes on the upper fold.
Answer 4 is incorrect as it includes only five holes.

We can eliminate answer choices 3 and 5 since the V shape is pointing in the wrong
direction in both of these choices compared to the image above.

Answer 2 can be eliminated because it includes two holes side by side in the center of
the paper, as opposed to one hole on each fold – one hole above the fold and one
below the fold.

We are left with the 1st answer choice, which is the correct answer.

26. Explanation
The correct answer is 3.

9
First, the square paper was folded in half widthwise and then it was folded in half
again lengthwise.
Four holes were then cut out of the folded paper.
Since the paper was folded in half twice, the holes were cut through four layers of
paper.
Therefore, when the paper is unfolded it will have sixteen holes: 4 x 4 = 16.

Answer choices 1, 2 and 4 each contain fewer than sixteen holes, so they can be
eliminated.
If you look closely at answer choice 5, you can see that it too has fewer than sixteen
holes and can therefore be eliminated.

We are left with the 3rd answer choice, which is the correct answer.

27. Explanation
The correct answer is 1.

First, the square paper was folded in half widthwise and then it was folded in
half again lengthwise.
Three holes were then cut out of the folded paper.
Since the paper was folded in half twice, the holes were cut through four layers of
paper.
Therefore, when the paper is unfolded it will have 12 holes: 3 x 4 = 12.

Answer choices 2 and 4 each contain fewer than 12 holes, so they can be eliminated.
Since the paper was folded in half widthwise and then again in half
lengthwise, each quarter must mirror the quarters above or below it and the quarter
to its left or right.
In answer choice 3, no quarter mirrors the one next to it, so it can be eliminated.
In answer choice 5, the quarters do mirror each other – but the holes are located closer
to the edges of the square than in the image above.

We are left with the 1st answer choice, which is the correct answer.

28. Explanation
The correct answer is 3.
The square paper was first folded diagonally in half and then three holes were cut out
of the folded paper. Since the paper was folded in half once, the holes were cut
through two layers of paper.
Therefore, when the paper is unfolded it will have six holes: 3 x 2 = 6.
Answer 4 includes only three holes, so it can be eliminated.
Since the paper was folded in half diagonally, the holes in the upper-left corner must
diagonally mirror the holes in the lower-right corner.
Therefore, answer choices 1, 2, and 5 can be eliminated.

10
We are left with the 3rd answer choice, which is the correct answer.

29. Explanation
The correct answer is 5.
The square paper was first folded diagonally in half and then three holes were cut out
of the folded paper.
Since the paper was folded in half once, the holes were cut through two layers of
paper.
Therefore, when the paper is unfolded it will have six holes: 3 x 2 = 6.
Answer choices 2 and 4 include only three holes, so they can be eliminated.
Since the paper was folded in half diagonally, the holes in the upper-left corner must
diagonally mirror the holes in the lower-right corner.
Therefore, answer choices 1 and 3 can be eliminated.
We are left with the 5th answer choice, which is the correct answer

30. Explanation
The correct answer is 4.
 First, the paper was folded diagonally in half from the upper-right corner to
the lower-left corner. Then it was folded again from the lower-right corner to
the upper-left corner.
 Then, two holes were cut out of four layers of paper.
 Therefore, when the paper is unfolded it will have eight holes (2 holes x 4
layers) which will mirror each other across the diagonals of the paper like
this:

Answer choices 2, 3, and 5 have fewer than eight holes, so they can be eliminated.
If you look closely at answer choice 1, you can see that although it has the right
number of holes, they are located closer to the center of the paper than in the image
above.
We are left with the 4th choice, which is the correct answer.

31. Explanation
The correct answer is 3.

The square paper is folded diagonally in half, and then four holes are cut out of the
folded paper.
Since the paper was folded in half once, the holes were cut through two layers of
paper.
Therefore, when the paper is unfolded it will have eight holes: 4 x 2 = 8.

11
Answer choices 2 and 4 include only four holes, so they can be eliminated.
Since the paper was folded diagonally, the holes in the upper-right corner
must diagonally mirror the holes in the lower-left corner.
In answer choice 1 we have the right number of holes, but they are in the wrong place
as they do not mirror the holes in the above image diagonally.
In answer choice 5 we also have eight holes, but none of the holes are in the right
position; even the original holes in the image above were moved up.

We are left with the 3rd answer choice, which is the correct answer.

32. Explanation
The correct answer is 4.

First the paper was folded diagonally in half from the lower-right corner to the
upper-left corner.
Then it was folded again from the lower-left corner to the upper-right corner.
Then four holes were cut out of four layers of paper.
Therefore, when the paper is unfolded it will have sixteen holes (4 holes x 4 layers)
which will mirror each other across the diagonals of the paper like this:

Answer choice 3 has fewer than sixteen holes, so it can be eliminated.


In answer choice 1, we have the right number of holes and they mirror each other
correctly, but the holes are too close to the center of the square compared to the holes
in the image above.
In answer choice 2, we have sixteen holes and they are mirrored diagonally, but here,
too, the holes are in the wrong places. This can be seen when we compare the location
of the holes at the top of the square to the ones in the original triangle above.
In answer choice 5, the holes are in the wrong places, which can be seen by
comparing them to the ones in the image above.

We are left with the 4th choice, which is the correct answer.

33. Explanation
The correct answer is 2.

The square paper is first folded in half diagonally, and then five holes are cut out of
the folded paper.

12
Since the paper was folded in half once, the holes were cut through two layers of
paper.
Therefore, when the paper is unfolded it will have ten holes: 5 x 2 = 10.

Answer choice 1 includes only five holes, so it can be eliminated.


Since the paper was folded in half diagonally, the holes in the lower-right corner
must mirror the holes in the upper-left corner.
In answer choice 3 we have the right number of holes, but they are in the wrong
positions - they appear in vertical and horizontal rows rather than diagonal ones.
In answer choice 4 we also have ten holes, but they are in the wrong position - the
lower-right corner does not mirror the upper-left corner.
In answer choice 5 we have ten holes, but the two-hole rows are in the wrong place -
they are located on the diagonal of the square.
We are left with the 2nd answer choice, which is the correct answer.

34. Explanation
The correct answer is 2.

First the paper was folded in half diagonally from the upper-left corner to the
lower-right corner.
Then it was folded again from the lower-left corner to the upper-right corner.
Then four holes were cut out of four layers of paper.

Therefore, when the paper is unfolded it will have sixteen holes (4 holes x 4 layers)
which will mirror each other across the diagonals of the paper like this:

A good rule-of-thumb is to look for the original holes in the answer choices, and first
eliminate the answers that do not include them.
When we compare the holes in the answer choices to the holes in the original image
above, we can immediately eliminate answer choices 3 and 5 because they do not
contain the holes in the original image.
Answer choice 4 has fewer than sixteen holes, so it can also be eliminated.
Looking at answer choice 1, we can see that although it has the right number of holes,
the holes are in the wrong places – for example, the holes at the bottom of the square
are too far from the holes on the right and do not mirror them.
We are left with the 2nd choice, which is the correct answer.

35. Explanation
The correct answer is 3.

13
Here we have a square with two of its corners folded inwards without meeting or
overlapping, creating two layers of paper in the upper-left and lower-right areas of
the square.
Notice that the three holes aligned from the upper-right corner to the lower-left
corner only go through one layer of paper, but the two other holes go through two
layers.
Therefore, the answer should include seven holes (3 holes + 2 holes x 2 layers =7).

Answer choices 1 and 2 have fewer than seven holes, so they can be eliminated.
Answer choice 4 has more than seven holes and the square has not been unfolded – so
it can be eliminated as well.
Answer choice 5 also has more than seven holes, and can therefore be eliminated.

We are left with the 3rd choice, which is the correct answer.

36. Explanation
The correct answer is 1.

The paper was folded from each corner towards the center of the square.
The folds do not meet or overlap, creating two layers of paper in each corner.
Then 4 holes were cut out.
All four holes go through two layers of paper, so the answer should include eight
holes (4 holes x 2 layers).

Answer choices 2 and 5 have fewer than eight holes, so they can be eliminated.
Also, the paper in answer choice 5 hasn’t been unfolded.
Answer choices 3 and 4 have holes in all four corners of the square and not only three
as in the image above.

We are left with the 1st choice, which is the only correct answer.

37. Explanation
The correct answer is 3.

The square paper was first folded in half diagonally, and then folded
again from the upper-left corner towards the lower-left corner.
Then, two holes were cut out of the folded paper.
The upper hole goes through four layers of paper since the paper is
folded twice where it is cut.
The lower hole goes through only two layers since the paper is folded once there.
Therefore, the answer should have six holes: 1 hole x 2 layers + 1 hole x 4 layers = 6.
The holes will flip over the fold each time the paper is unfolded – first upwards and
then diagonally.

14
Answer choices 1, 2 and 4 include fewer than six holes, so they can be eliminated.
Answer choice 5 has the right number of holes, but they are in the wrong places – this
can be seen when we compare the holes on the lower-left corner to the holes in the
image above.

We are left with the 3rd answer choice, which is the correct answer.

38. Explanation
The correct answer is 3.

First the paper was folded in half lengthwise, and then both outer (bottom)
corners were folded towards the center of the square.
Then 2 holes were cut out.
Both holes go through four layers of paper, so the answer should include eight holes:
2 holes x 4 layers = 8.

A good rule-of-thumb is to look for the original holes in the answer choices, and first
eliminate the answers that do not include them.
When we compare the holes in the answer choices to the holes in the original image
above, we can immediately eliminate answer choices 2 and 4 because they do not
contain the holes in the original image.
Answer choice 5 has fewer than eight holes, so it can be eliminated too.
Answer choice 1 has more than eight holes - notice the added hole in the center - so it
can be eliminated as well.

We are left with the 3rd choice, which is the only correct answer.

39. Explanation
The correct answer is 4.

First the paper was folded in half lengthwise, and then the outer upper
corner was folded diagonally towards the center of the square.
Then 1 hole was cut out.
Since the paper was folded twice, the hole goes through four layers of paper.
Therefore, the answer should include four holes: 1 hole x 4 layers = 4.

Answer choice 3 has fewer than four holes, so it can be eliminated.


Answer choice 5 has more than four holes, so it can be eliminated as well.
Answer choices 1 and 2 have the right number of holes, but they are in the wrong
places - when the paper is unfolded diagonally there should be holes very close to
edge of the square, and none of these answers have such holes.

We are left with the 4th choice, which is the correct answer.

15
40. Explanation
The correct answer is (B).

The paper is being folded twice and punched with two holes. However, it is important
to notice how many layers of papers are being punched with holes. In this question,
the paper is punched with two holes. Notice the top-right part of the paper was only
folded once (in the first transition), so it only has two layers of paper. On the other
hand, the hole to its left has four layers, as the paper on that point was folded twice.

Side note #1: Every folding of paper doubles the layers of paper.
As there are two holes punched, one on two layers of paper and the other on four
layers, you can expect there will be six holes in total. Thus, answers (A), (D) and (E)
can be ruled out (as they contain four, five and eight holes, respectively).

The remaining answers (B) and (C) are much alike and the difference between them is
the position of the two holes in the bottom-left side of the square.

Since the paper is being folded twice, there would be two symmetry lines which
would mirror the holes: the diagonal line in the first folding, and the horizontal line in
the second folding.
The unfolding of the paper is done, naturally, in the opposite order of the folding.
Therefore, the first unfolding would be across the horizontal line of the second
transition, and as it only affects the hole on the left, unfolding the paper across it will
result in a symmetric hole in the bottom of the paper:

This step is enough to rule out answer choice (C) as the hole there is not directly
beneath the left hole, thus leaving answer choice (B) as the only remaining option.
Alternatively, you can unfold the paper once more to see the end result; creating a
mirror-image across the diagonal line of the first transition:

16
Side note #2: A folding of the paper creates a symmetry line. Therefore, punching
holes in a folded paper makes a mirror image along that same symmetry line (after
unfolding).

41. Explanation
The correct answer is (C).

The top and bottom edges of the papers are folded inward, then three holes are
punched in.

As folding of the paper creates a symmetry line, punching-in holes in a folded paper
makes a mirror image along that same symmetry line (after unfolding). This question
has two symmetry lines and, as they are horizontal, the holes also would be mirrored
horizontally. The top two holes would have a pair of two holes above them, as the
appropriate symmetry line is above them:

The lower hole has a symmetry line underneath it, and therefore it would have a
mirrored hole below it:

The only answer which follows this rule is answer (C), which is the correct answer.

Answer (A) and (B) are incorrect, as there are a holes above the bottom hole, instead
of below it.

Answer (D) is incorrect, as the hole in the bottom is diagonal, rather than directly
below the original hole.

Answer (E) is incorrect as its missing two holes in its top part.

42. Explanation
The correct answer is (D).

17
The paper has its top-left and bottom-left corners folded diagonally inwards and then
it is punched in with four holes.
The top-right hole is in an area of the paper which was not folded, therefore it will not
be duplicated and only appear in that area.
The other three holes, on the other hand, are in areas that have been folded and
therefore will have duplicates in the unfolded paper.
Thus, you can tell the paper will have seven holes when unfolded, so answer (C) can
be ruled out.

As folding of the paper creates a symmetry line, punching-in holes in a folded paper
make a mirror image along that same symmetry line (after unfolding). The given
paper is folded diagonally twice, and therefore the symmetry lines will be diagonals.
The top-left holes have a diagonal line above them, hence when the paper unfolds,
there will be symmetric duplicates mirrored around it:

Similarly, the bottom-left hole has a symmetry line below it and therefore it will have
a hole underneath it, mirrored around that diagonal:

Therefore, the correct answer is (D).

Answer (A) is incorrect as the top-left holes are not mirrored around the diagonal
symmetrically.

Answer (B) is incorrect as the bottom-left hole has no duplicate. In addition, there is a
hole around in the bottom-right corner which does not belong.

Answer (C), as mentioned, has 8 holes when there are supposed to be 7.

Answer (E) is incorrect. The bottom-left hole is not mirrored around the diagonal
symmetrically. In addition, the holes in the top left part are also in the wrong position.

43. Explanation
The correct answer is 2.

First, the square paper was folded in half widthwise, and then it was folded in half
again lengthwise.

18
Four holes were then cut out of the folded paper.

Since the paper was folded in half twice, the holes were cut through four layers of
paper.
Therefore, when the paper is unfolded it will have sixteen holes: 4 x 4 = 16.

Answer choices 3, 4, and 5 each contain fewer than sixteen holes, so they can be
eliminated.
If you look closely at answer choice 1, you can see that it too has fewer than
sixteen holes and therefore can be eliminated.

We are left with the 2nd answer choice, which is the correct answer.

44. Explanation
The correct answer is 4.
 First, the square paper was folded in half widthwise, and then it was folded in
half again lengthwise.
 Then, three holes were cut out of the folded paper.
 Since the paper was folded in half twice, the holes were cut through four
layers of paper.
 Therefore, when the paper is unfolded it will have twelve holes: 3 x 4 = 12.
 Since the paper was folded in half widthwise and then again in half lengthwise,
each quarter must mirror the quarters above or below it and the quarter to its
left or right.
Answer choices 1, 3, and 5 include fewer than 12 holes, so they can be eliminated.
If you look closely at answer choice 2, you can see that the quarters do not mirror
each other in this way. The holes in the top half are positioned exactly the same as
those in the bottom half instead of mirroring them.

We are left with the 4th answer choice, which is the correct answer.

45. Explanation
The correct answer is 2.
The square paper was first folded diagonally in half and then three holes were cut out
of the folded paper.
Since the paper was folded in half once, the holes were cut through two layers of
paper.
Therefore, when the paper is unfolded it will have six holes: 3 x 2 = 6.
Answer choice 4 includes only three holes, so it can be eliminated.
Since the paper was folded in half diagonally, the holes in the upper-right corner must
diagonally mirror the holes in the lower-left corner.
Therefore, answer choices 1, 3, and 5 can be eliminated.
We are left with the 2nd answer choice, which is the correct answer.

19
46. Explanation
The correct answer is 4.
 The square paper was first folded diagonally in half, and then three holes were
cut out of the folded paper.
 Since the paper was folded in half once, the holes were cut through two layers
of paper.
 Therefore, when the paper is unfolded it will have six holes: 3 x 2 = 6.
Answer choices 1, 2, 3, and 5 have fewer than six holes, so they can each be
eliminated.

Therefore, we are left with the 4th answer choice, which is the correct answer.

47. Explanation
The correct answer is 5.
 First, the square paper was folded diagonally in half.
 Then, four holes were cut out of the folded paper.
 Since the paper was folded in half once, the holes were cut through two
layers of paper.
 Therefore, when the paper is unfolded it will have eight holes: 4 x 2 = 8.
 Since the paper was folded diagonally, the holes in the upper-left corner must
diagonally mirror the holes in the lower-right corner.
Answer choices 1 and 4 include only four holes, so they can be eliminated.
In answer choice 2 we have the right number of holes, but they are in the wrong place
as they do not mirror the holes in the above image diagonally.
In answer choice 3 we also have eight holes, but none of the holes are in the right
position; even the original holes from the image above were moved.

We are left with the 5th answer choice, which is the correct answer.

48. Explanation
The correct answer is 4.

The square paper is first folded in half diagonally, and then five holes are cut out of
the folded paper.

Since the paper was folded in half once, the holes were cut through two layers of
paper.
Therefore, when the paper is unfolded it will have ten holes: 5 x 2 = 10.

A good rule-of-thumb is to look for the original holes in the answer choices, and first
eliminate the answers that do not include them.
When we compare the holes in the answer choices to the holes in the original image
above, we can immediately eliminate answer choices 2, 3, and 5 because they do not
contain vertical hole groups in the center of the square.

20
In answer choice 1 we have the right number of holes, but they are also in the wrong
positions – when we compare the upper-right corner of this answer choice with the
one in the above image, we can see that the two-hole and three-hole groups have
switched places.

We are left with the 4th answer choice, which is the correct answer.

49. Explanation
The correct answer is 4.

Here we have a square with two of its corners folded inwards without meeting or
overlapping, creating two layers of paper in the upper-right and lower-left areas of
the square.
Notice that the hole in the center of the square only goes through one layer of paper,
but the four other holes go through two layers.
Therefore, the answer should include nine holes (1 hole + 4 holes x 2 layers =9).

Answer choices 1, 2 and 5 have fewer than nine holes, so they can be eliminated.
Also, notice that the paper in answer choice 5 has not been unfolded.
Answer choice 3 has more than nine holes – notice that the hole in the center has been
duplicated – so it can be eliminated as well.

We are left with the 4th choice, which is the correct answer.

50. Explanation
The correct answer is 4.

The paper was folded from each corner towards the center of the square.
The folds do not meet or overlap, creating two layers of paper in each corner.
Then four holes were cut out.
All four holes go through two layers of paper, so the answer should include eight
holes (4 holes x 2 layers).

Answer choices 3 and 5 have fewer than eight holes, so they can be eliminated.
Also, the square in answer choice 3 hasn’t been unfolded.
Answer choices 1 and 2 have holes in all four corners of the square and not only three
corners as in the image above, so they too can be eliminated.

We are left with the 4th choice, which is the correct answer.

51. Explanation
The correct answer is 5.

21
First, the paper was folded in half lengthwise, and then both outer (left)
corners were folded towards the center of the square.
Then 2 holes were cut out.
Both holes go through four layers of paper, so the answer should include eight holes:
2 holes x 4 layers = 8.

A good rule-of-thumb is to look for the original holes in the answer choices, and first
eliminate the answers that do not include them.
When we compare the holes in the answer choices to the holes in the original image
above, we can immediately eliminate answer choices 1 and 2 because they do not
contain the holes in the original image.
Answer choice 4 has fewer than eight holes, so it can be eliminated as well.
Answer choice 3 has more than eight holes - notice the added hole in the center - so it
can be eliminated too.
We are left with the 5th choice, which is the correct answer.

52. Explanation
The correct answer is 3.

First the paper was folded in half widthwise, and then the left outer (bottom)
corner was folded diagonally towards the center of the square.
Then 1 hole was cut out.
Since the paper was folded twice, the hole goes through four layers of paper.
Therefore, the answer should include four holes: 1 hole x 4 layers = 4.

A good rule-of-thumb is to look for the original holes in the answer choices, and first
eliminate the answers that do not include them.
When we compare the holes in the answer choices to the holes in the original image
above, we can immediately eliminate answer choice 4 because it does not contain the
holes in the original image.
Answer choice 1 has fewer than four holes, so it can be eliminated too.
Answer choices 2 and 5 have more than four holes, so they can be eliminated as well.
We are left with the 3rd choice, which is the correct answer.

53. Explanation
The correct answer is 5.

First the paper was folded lengthwise from the right edge of the square towards the
middle.
Then the outer corners were folded towards the center of the square.
Then 3 holes were cut out.
The folds do not overlap, so the paper was folded only once in each section.
Therefore, the holes go through two layers of paper, and the answer should
include six holes: 3 holes x 2 layers = 6.

22
Answer choices 2 and 4 have fewer than six holes, so they can be eliminated.
Answer choice 1 has more than six holes, so it can be eliminated as well.
Answer choice 3 has the right number of holes, but they are in the wrong places –
notice that the hole on the left side of the square is in line with the other holes. Since
the fold in that corner is diagonal, the hole should be higher than the other holes .

We are left with the 5th choice, which is the correct answer.

54. Explanation
The correct answer is (E).

The paper is folded vertically in half, its top size is flipped diagonally and then it is
punched with one hole. As folding a paper doubles the amount of layers being
punched and, in this question, the punched part of the paper was folded twice, you can
deduce there will be four holes in total in the unfolded paper. Thus, you can rule out
answers (A), (B) and (D).

Answer (C) is incorrect because the holes on top should be closer to each other. The
second folding is diagonal, so the resulting holes will be closer to the line from the
first folding than to their respective corners.

In Answer (E), the hole is mirrored once diagonally (as the hole above it is not
directly above it) and once vertically (as the holes in both sides of the paper have the
same height).

55. Explanation
The correct answer is 4.
The square paper was first folded in half lengthwise and then five holes were cut out
of the folded paper.
Since the paper was folded in half once, the holes were cut through two layers of
paper.
Therefore, when the paper is unfolded it will have ten holes: 5 x 2 = 10.
Answer choices 3 and 5 only have five holes each, so they can be eliminated. Answer
1 only has 8 holes, so it can be eliminated as well.

Since the paper was folded lengthwise down the middle the holes on the left must
mirror the holes on the right. (In other words, since the original holes made a shape of
an arrowhead pointing left, the result should have two arrowheads - pointing left and
right.)
We can eliminate answer choice 2 since it has two holes next to both the left and right
edges of the paper rather than one hole, as seen in the image above. (In other words,
answer 2 has two arrowheads, but they are pointing up and down.)
We are left with the 4th answer choice, which is the correct answer.

23
56. Explanation
The correct answer is 2.

First, the square paper was folded in half widthwise, and then it was folded in half
again lengthwise.
Four holes were then cut out of the folded paper.

Since the paper was folded in half twice, the holes were cut through four layers of
paper.
Therefore, when the paper is unfolded it will have sixteen holes: 4 x 4 = 16.

Answer choices 3, 4, and 5 each contain fewer than sixteen holes, so they can be
eliminated.
If you look closely at answer choice 1, you can see that it too has fewer than
sixteen holes and therefore can be eliminated.

We are left with the 2nd answer choice, which is the correct answer.

57. Explanation
The correct answer is 4.
First, the square paper was folded in half widthwise, and then it was folded in half
again lengthwise.
Then, three holes were cut out of the folded paper.
Since the paper was folded in half twice, the holes were cut through four layers of
paper.
Therefore, when the paper is unfolded it will have twelve holes: 3 x 4 = 12.
Since the paper was folded in half widthwise and then again in half lengthwise, each
quarter must mirror the quarters above or below it and the quarter to its left or right.
Answer choices 1, 3, and 5 include fewer than 12 holes, so they can be eliminated.
If you look closely at answer choice 2, you can see that the quarters do not mirror
each other in this way. The holes in the top half are positioned exactly the same as
those in the bottom half instead of mirroring them.

We are left with the 4th answer choice, which is the correct answer.

58. Explanation
The correct answer is 2.
The square paper was first folded diagonally in half and then three holes were cut out
of the folded paper.
Since the paper was folded in half once, the holes were cut through two layers of
paper.
Therefore, when the paper is unfolded it will have six holes: 3 x 2 = 6.
Answer choice 4 includes only three holes, so it can be eliminated.

24
Since the paper was folded in half diagonally, the holes in the upper-right corner must
diagonally mirror the holes in the lower-left corner.
Therefore, answer choices 1, 3, and 5 can be eliminated.
We are left with the 2nd answer choice, which is the correct answer.

59. Explanation
The correct answer is 4.
The square paper was first folded diagonally in half, and then three holes were cut out
of the folded paper.
Since the paper was folded in half once, the holes were cut through two layers of
paper.
Therefore, when the paper is unfolded it will have six holes: 3 x 2 = 6.
Answer choices 1, 2, 3, and 5 have fewer than six holes, so they can each be
eliminated.

Therefore, we are left with the 4th answer choice, which is the correct answer.

60. Explanation
The correct answer is 3.

First, the paper was folded in half diagonally from the upper-right corner to the
lower-left corner. Then it was folded again from the upper-left corner to the
lower-right corner.
Then two holes were cut out of four layers of paper.

Therefore, when the paper is unfolded it will have eight holes (2 holes x 4 layers)
which will mirror each other across the diagonals of the paper like this:

Answer choices 1, 4, and 5 have fewer than eight holes, so they can be eliminated.
If you look closely at answer choice 2, you can see that although it has the right
number of holes, they are in the wrong places: the holes are located closer to the
edges of the paper than in the image above.

We are left with the 3rd choice, which is the correct answer.

61. Explanation
The correct answer is 5.
First, the square paper was folded diagonally in half.
Then, four holes were cut out of the folded paper.

25
Since the paper was folded in half once, the holes were cut through two layers of
paper.
Therefore, when the paper is unfolded it will have eight holes: 4 x 2 = 8.
Since the paper was folded diagonally, the holes in the upper-left corner must
diagonally mirror the holes in the lower-right corner.
Answer choices 1 and 4 include only four holes, so they can be eliminated.
In answer choice 2 we have the right number of holes, but they are in the wrong place
as they do not mirror the holes in the above image diagonally.
In answer choice 3 we also have eight holes, but none of the holes are in the right
position; even the original holes from the image above were moved.

We are left with the 5th answer choice, which is the correct answer.

62. Explanation
The correct answer is 5.
First, the paper was folded diagonally in half from the upper-right corner to the
lower-left corner. Then it was folded again from the upper-left corner to the
lower-right corner.
Then, four holes were cut out of four-layer of paper.
Therefore, when the paper is unfolded it will have sixteen holes (4 holes x 4 layers)
which will mirror each other across the diagonals of the paper like this:

Answer choice 4 has fewer than sixteen holes, so it can be eliminated.


Answer choice 1 has more than sixteen holes, so it can be eliminated as well.
Looking at answer choice 2, we can see that although it has the right number of holes,
they are in the wrong place. This can be seen when we compare the location of the
holes on the bottom of the square to the ones in the original image above.
In answer choice 3, we can see that the holes on the sides of the square are placed in
the same position, and do not fit the diagonal mirroring pattern we need.
We are left with the 5th choice, which is the correct answer.

63. Explanation
The correct answer is 4.

First the paper was folded in half diagonally from the upper-right corner to the
lower-left corner.
Then it was folded again from the upper-left corner to the lower-right corner.
Then four holes were cut out of four layers of paper.

26
Therefore, when the paper is unfolded it will have sixteen holes (4 holes x 4 layers)
which will mirror each other across the diagonals of the paper like this:

A good rule-of-thumb is to look for the original holes in the answer choices, and first
eliminate the answers that do not include them.
When we compare the holes in the answer choices to the holes in the original image
above, we can immediately eliminate answer choices 1, 2 and 3 because they do not
contain the holes in the original image.
We can also eliminate answer choice 5 because it has fewer than sixteen holes.

We are left with the 4th choice, which is the only correct answer.

64. Explanation
The correct answer is 4.

Here we have a square with two of its corners folded inwards without meeting or
overlapping, creating two layers of paper in the upper-right and lower-left areas of the
square.
Notice that the hole in the center of the square only goes through one layer of paper,
but the four other holes go through two layers.
Therefore, the answer should include nine holes (1 hole + 4 holes x 2 layers =9).

Answer choices 1, 2 and 5 have fewer than nine holes, so they can be eliminated.
Also, notice that the paper in answer choice 5 has not been unfolded.
Answer choice 3 has more than nine holes – notice that the hole in the center has been
duplicated – so it can be eliminated as well.

We are left with the 4th choice, which is the correct answer.

65. Explanation
The correct answer is 4.

The paper was folded from each corner towards the center of the square.
The folds do not meet or overlap, creating two layers of paper in each corner.
Then four holes were cut out.
All four holes go through two layers of paper, so the answer should include eight
holes (4 holes x 2 layers).

27
Answer choices 3 and 5 have fewer than eight holes, so they can be eliminated.
Also, the square in answer choice 3 hasn’t been unfolded.
Answer choices 1 and 2 have holes in all four corners of the square and not only three
corners as in the image above, so they too can be eliminated.

We are left with the 4th choice, which is the correct answer.

66. Explanation
The correct answer is 2.

The square paper was first folded in half diagonally, and then folded
again from the upper-right corner towards the lower-right corner.
Then, two holes were cut out of the folded paper.
Both holes go through four layers of paper, since the paper is folded twice where they
are cut.
Therefore, the answer should have eight holes (2 holes x 4 layers).
The holes will flip over the fold each time the paper is unfolded – first upwards and
then diagonally.

Answer choice 5 includes only four holes, so it can be eliminated.


Answer choice 4 doesn’t have any holes on the right edge of the square, where they
appear in the image above.
Answer choices 1 and 3 have holes in the wrong places, as they do not mirror the first
holes upwards and diagonally.

We are left with the 2nd answer choice, which is the correct answer.

67. Explanation
The correct answer is 5.

First, the paper was folded in half lengthwise, and then both outer (left)
corners were folded towards the center of the square.
Then 2 holes were cut out.
Both holes go through four layers of paper, so the answer should include eight holes:
2 holes x 4 layers = 8.

A good rule-of-thumb is to look for the original holes in the answer choices, and first
eliminate the answers that do not include them.
When we compare the holes in the answer choices to the holes in the original image
above, we can immediately eliminate answer choices 1 and 2 because they do not
contain the holes in the original image.
Answer choice 4 has fewer than eight holes, so it can be eliminated as well.
Answer choice 3 has more than eight holes - notice the added hole in the center - so it
can be eliminated too.

28
We are left with the 5th choice, which is the correct answer.

68. Explanation
The correct answer is 3.

First the paper was folded in half widthwise, and then the left outer (bottom)
corner was folded diagonally towards the center of the square.
Then 1 hole was cut out.
Since the paper was folded twice, the hole goes through four layers of paper.
Therefore, the answer should include four holes: 1 hole x 4 layers = 4.

A good rule-of-thumb is to look for the original holes in the answer choices, and first
eliminate the answers that do not include them.
When we compare the holes in the answer choices to the holes in the original image
above, we can immediately eliminate answer choice 4 because it does not contain the
holes in the original image.
Answer choice 1 has fewer than four holes, so it can be eliminated too.
Answer choices 2 and 5 have more than four holes, so they can be eliminated as well.
We are left with the 3rd choice, which is the correct answer.

69. Explanation
The correct answer is 1.

First, the square paper was folded in half widthwise and then it was folded in
half again lengthwise.
Three holes were then cut out of the folded paper.
Since the paper was folded in half twice, the holes were cut through four layers of
paper.
Therefore, when the paper is unfolded it will have 12 holes: 3 x 4 = 12.

Answer choices 2 and 4 each contain fewer than 12 holes, so they can be eliminated.
Since the paper was folded in half widthwise and then again in half
lengthwise, each quarter must mirror the quarters above or below it and the quarter
to its left or right.
In answer choice 3, no quarter mirrors the one next to it, so it can be eliminated.
In answer choice 5, the quarters do mirror each other – but the holes are located closer
to the edges of the square than in the image above.

We are left with the 1st answer choice, which is the correct answer.

70. Explanation
The correct answer is 3.
The square paper was first folded diagonally in half and then three holes were cut out
of the folded paper. Since the paper was folded in half once, the holes were cut

29
through two layers of paper.
Therefore, when the paper is unfolded it will have six holes: 3 x 2 = 6.
Answer 4 includes only three holes, so it can be eliminated.
Since the paper was folded in half diagonally, the holes in the upper-left corner must
diagonally mirror the holes in the lower-right corner.
Therefore, answer choices 1, 2, and 5 can be eliminated.
We are left with the 3rd answer choice, which is the correct answer.

71. Explanation
The correct answer is 5.
The square paper was first folded diagonally in half and then three holes were cut out
of the folded paper.
Since the paper was folded in half once, the holes were cut through two layers of
paper.
Therefore, when the paper is unfolded it will have six holes: 3 x 2 = 6.
Answer choices 2 and 4 include only three holes, so they can be eliminated.
Since the paper was folded in half diagonally, the holes in the upper-left corner must
diagonally mirror the holes in the lower-right corner.
Therefore, answer choices 1 and 3 can be eliminated.
We are left with the 5th answer choice, which is the correct answer.

72. Explanation
The correct answer is 4.

 First, the paper was folded diagonally in half from the upper-right corner to
the lower-left corner. Then it was folded again from the lower-right corner to
the upper-left corner.
 Then, two holes were cut out of four layers of paper.
 Therefore, when the paper is unfolded it will have eight holes (2 holes x 4
layers) which will mirror each other across the diagonals of the paper like
this:

Answer choices 2, 3, and 5 have fewer than eight holes, so they can be eliminated.
If you look closely at answer choice 1, you can see that although it has the right
number of holes, they are located closer to the center of the paper than in the image
above.
We are left with the 4th choice, which is the correct answer.

73. Explanation

30
The correct answer is 3.

The square paper is folded diagonally in half, and then four holes are cut out of the
folded paper.
Since the paper was folded in half once, the holes were cut through two layers of
paper.
Therefore, when the paper is unfolded it will have eight holes: 4 x 2 = 8.

Answer choices 2 and 4 include only four holes, so they can be eliminated.
Since the paper was folded diagonally, the holes in the upper-right corner
must diagonally mirror the holes in the lower-left corner.
In answer choice 1 we have the right number of holes, but they are in the wrong place
as they do not mirror the holes in the above image diagonally.
In answer choice 5 we also have eight holes, but none of the holes are in the right
position; even the original holes in the image above were moved up.

We are left with the 3rd answer choice, which is the correct answer.

74. Explanation
The correct answer is 2.

The square paper is first folded in half diagonally, and then five holes are cut out of
the folded paper.
Since the paper was folded in half once, the holes were cut through two layers of
paper.
Therefore, when the paper is unfolded it will have ten holes: 5 x 2 = 10.

Answer choice 1 includes only five holes, so it can be eliminated.


Since the paper was folded in half diagonally, the holes in the lower-right corner
must mirror the holes in the upper-left corner.
In answer choice 3 we have the right number of holes, but they are in the wrong
positions - they appear in vertical and horizontal rows rather than diagonal ones.
In answer choice 4 we also have ten holes, but they are in the wrong position - the
lower-right corner does not mirror the upper-left corner.
In answer choice 5 we have ten holes, but the two-hole rows are in the wrong place -
they are located on the diagonal of the square.
We are left with the 2nd answer choice, which is the correct answer.

75. Explanation
The correct answer is 2.

First the paper was folded in half diagonally from the upper-left corner to the
lower-right corner.
Then it was folded again from the lower-left corner to the upper-right corner.

31
Then four holes were cut out of four layers of paper.

Therefore, when the paper is unfolded it will have sixteen holes (4 holes x 4 layers)
which will mirror each other across the diagonals of the paper like this:

A good rule-of-thumb is to look for the original holes in the answer choices, and first
eliminate the answers that do not include them.
When we compare the holes in the answer choices to the holes in the original image
above, we can immediately eliminate answer choices 3 and 5 because they do not
contain the holes in the original image.
Answer choice 4 has fewer than sixteen holes, so it can also be eliminated.
Looking at answer choice 1, we can see that although it has the right number of holes,
the holes are in the wrong places – for example, the holes at the bottom of the square
are too far from the holes on the right and do not mirror them.
We are left with the 2nd choice, which is the correct answer.

76. Explanation
The correct answer is 3.

Here we have a square with two of its corners folded inwards without meeting or
overlapping, creating two layers of paper in the upper-left and lower-right areas of
the square.
Notice that the three holes aligned from the upper-right corner to the lower-left
corner only go through one layer of paper, but the two other holes go through two
layers.
Therefore, the answer should include seven holes (3 holes + 2 holes x 2 layers =7).

Answer choices 1 and 2 have fewer than seven holes, so they can be eliminated.
Answer choice 4 has more than seven holes and the square has not been unfolded – so
it can be eliminated as well.
Answer choice 5 also has more than seven holes, and can therefore be eliminated.

We are left with the 3rd choice, which is the correct answer.

77. Explanation
The correct answer is 3.

The square paper was first folded in half diagonally, and then folded
again from the upper-left corner towards the lower-left corner.

32
Then, two holes were cut out of the folded paper.
The upper hole goes through four layers of paper since the paper is
folded twice where it is cut.
The lower hole goes through only two layers since the paper is folded once there.
Therefore, the answer should have six holes: 1 hole x 2 layers + 1 hole x 4 layers = 6.
The holes will flip over the fold each time the paper is unfolded – first upwards and
then diagonally.

Answer choices 1, 2 and 4 include fewer than six holes, so they can be eliminated.
Answer choice 5 has the right number of holes, but they are in the wrong places – this
can be seen when we compare the holes on the lower-left corner to the holes in the
image above.

We are left with the 3rd answer choice, which is the correct answer.

78. Explanation
The correct answer is 3.

First the paper was folded in half lengthwise, and then both outer (bottom)
corners were folded towards the center of the square.
Then 2 holes were cut out.
Both holes go through four layers of paper, so the answer should include eight holes:
2 holes x 4 layers = 8.

A good rule-of-thumb is to look for the original holes in the answer choices, and first
eliminate the answers that do not include them.
When we compare the holes in the answer choices to the holes in the original image
above, we can immediately eliminate answer choices 2 and 4 because they do not
contain the holes in the original image.
Answer choice 5 has fewer than eight holes, so it can be eliminated too.
Answer choice 1 has more than eight holes - notice the added hole in the center - so it
can be eliminated as well.

We are left with the 3rd choice, which is the only correct answer.

79. Explanation
The correct answer is 4.

First the paper was folded in half lengthwise, and then the outer upper
corner was folded diagonally towards the center of the square.
Then 1 hole was cut out.
Since the paper was folded twice, the hole goes through four layers of paper.
Therefore, the answer should include four holes: 1 hole x 4 layers = 4.

33
Answer choice 3 has fewer than four holes, so it can be eliminated.
Answer choice 5 has more than four holes, so it can be eliminated as well.
Answer choices 1 and 2 have the right number of holes, but they are in the wrong
places - when the paper is unfolded diagonally there should be holes very close to
edge of the square, and none of these answers have such holes.

We are left with the 4th choice, which is the correct answer.

80. Explanation
The correct answer is (C).

The top and bottom edges of the papers are folded inward, then three holes are
punched in.

As folding of the paper creates a symmetry line, punching-in holes in a folded paper
makes a mirror image along that same symmetry line (after unfolding). This question
has two symmetry lines and, as they are horizontal, the holes also would be mirrored
horizontally. The top two holes would have a pair of two holes above them, as the
appropriate symmetry line is above them:

The lower hole has a symmetry line underneath it, and therefore it would have a
mirrored hole below it:

The only answer which follows this rule is answer (C), which is the correct answer.

Answer (A) and (B) are incorrect, as there are a holes above the bottom hole, instead
of below it.

Answer (D) is incorrect, as the hole in the bottom is diagonal, rather than directly
below the original hole.

Answer (E) is incorrect as its missing two holes in its top part.

34
81. Explanation
The correct answer is (D).

The paper has its top-left and bottom-left corners folded diagonally inwards and then
it is punched in with four holes.
The top-right hole is in an area of the paper which was not folded, therefore it will not
be duplicated and only appear in that area.
The other three holes, on the other hand, are in areas that have been folded and
therefore will have duplicates in the unfolded paper.
Thus, you can tell the paper will have seven holes when unfolded, so answer (C) can
be ruled out.

As folding of the paper creates a symmetry line, punching-in holes in a folded paper
make a mirror image along that same symmetry line (after unfolding). The given
paper is folded diagonally twice, and therefore the symmetry lines will be diagonals.
The top-left holes have a diagonal line above them, hence when the paper unfolds,
there will be symmetric duplicates mirrored around it:

Similarly, the bottom-left hole has a symmetry line below it and therefore it will have
a hole underneath it, mirrored around that diagonal:

Therefore, the correct answer is (D).

Answer (A) is incorrect as the top-left holes are not mirrored around the diagonal
symmetrically.

Answer (B) is incorrect as the bottom-left hole has no duplicate. In addition, there is a
hole around in the bottom-right corner which does not belong.

Answer (C), as mentioned, has 8 holes when there are supposed to be 7.

Answer (E) is incorrect. The bottom-left hole is not mirrored around the diagonal
symmetrically. In addition, the holes in the top left part are also in the wrong position.

82. Explanation
The correct answer is 5.

35
 First, the rectangular paper is folded in half lengthwise.
 Then, a small triangle is cut out of the folded paper.
 Consequently, the unfolded paper will have two triangle cut-outs which
eliminates answer choices 3 and 4.
 Since the paper is folded lengthwise, the second triangle must mirror
horizontally the cut triangle which eliminates answer choices 1 and 2.
We are left with the 5th choice, which is the correct answer.

83. Explanation
The correct answer is 2.
 First, the rectangular paper is folded in half lengthwise.
 Then, a diagonal is cut out of the upper-left and lower-right corners of the
folded paper.
 Consequently, the unfolded paper will have four diagonal cut-outs.
Since the paper is folded lengthwise, the second cut-outs must mirror horizontally the
original cut-outs. This eliminates answer choices 1, 3, 4, and 5.
We are left with the 2nd choice, which is the correct answer.

84. Explanation
The correct answer is 5.
 First, the square paper is folded in half to make a triangle.
 Then, a pentagon and parallelogram are cut out of the folded paper.
 Consequently, there will be two pentagon cut-outs and two parallelogram
cut-outs when the paper is unfolded again.
 Also, the second pentagon and second parallelogram must be rotated 90°
clockwise and flipped vertically.
since the paper was folded along the diagonal. This eliminates answer choices 1, 2, 3,
and 4.
We are left with the 5th choice, which is the correct answer.

85. Explanation
The correct answer is 3.

First, the rectangular paper is folded in half lengthwise. Then, a rectangle, triangle and
oval are cut out of the folded paper. Consequently, the unfolded paper will have two
rectangle cut-outs, two triangle cut-outs and two oval cut-outs. Since the paper is
folded lengthwise, the second rectangle, second triangle and second oval must mirror
horizontally the cut rectangle, triangle and oval which eliminates answer choices 1, 2,
4 and 5.

86. Explanation
The correct answer is 3.

First, the rectangular paper is folded in half lengthwise. Then, three arrows are cut out

36
of the folded paper. Consequently, the unfolded paper will have six arrow cut-outs.
Since the paper is folded lengthwise, the second three arrows must mirror horizontally
the cut three arrows which eliminates answer choices 1, 2, 4 and 5.

87. Explanation
The correct answer is 5.
 First, the rectangular paper is folded in half widthwise and then folded in half
lengthwise.
 Then, one triangle is cut out of the folded paper.
 Consequently, when the paper is unfolded lengthwise the paper will have two
triangle cut-outs. When the paper is then unfolded widthwise the paper will
have four triangle cut-outs.
Since the paper is folded widthwise and then lengthwise, the second triangle must
mirror horizontally the cut triangle, and then the last two triangles must mirror
vertically the first two triangles. This eliminates answer choices 1, 2, 3, and 4.
Therefore, we are left with the 5th choice, which is the correct answer.

88. Explanation
The correct answer is 5.
 First, the rectangular paper is folded in half lengthwise and then folded in half
widthwise.
 Then, a triangle and pie are cut out of the folded paper.
 Consequently, when the paper is unfolded widthwise the paper will have two
triangle cut-outs and two pie cut-outs.
 When the paper is then unfolded lengthwise the paper will have four triangle
cut-outs and four pie cut-outs.
Since the paper is folded lengthwise and then widthwise, the second triangle and
second pie must mirror vertically the cut triangle and pie, and then the last two
triangles and two pies must mirror horizontally the first two triangles and pies. This
eliminates answer choices 1, 2, 3, and 4.
Thus, the only possible answer is the 5th choice, and it is the correct answer.

89. Explanation
The correct answer is 1.
 First, the square paper is folded in half widthwise and then folded in half
lengthwise.
 Then, one arrow, triangle, and rectangle shape are cut out of the folded paper.
 Consequently, when the paper is unfolded lengthwise the paper will have two
arrow cut-outs, two triangle cut-outs and two rectangle cut-outs.
 When the paper is then unfolded widthwise the paper will have four arrow
cut-outs, four triangle cut-outs, and four rectangle cut-outs.
Since the paper is folded widthwise and then lengthwise, the second arrow, second
triangle, and second rectangle shapes must mirror horizontally the cut arrow, triangle,
and rectangle, and then the final two sets of the arrow, triangle, and rectangle shapes

37
must mirror vertically the first two sets of arrow, triangle, and rectangle shapes. This
eliminates answer choices 2, 3, 4, and 5.
Thus, the only remaining possible answer is the 1st choice, and it is the correct
answer.

90. Explanation
The correct answer is 1.
 First, the rectangular paper is folded in half widthwise and then folded in half
lengthwise.
 Then, one heart and two rectangles are cut out of the folded paper.
 Consequently, when the paper is unfolded lengthwise the paper will have two
heart cut-outs and four rectangle cut-outs.
 When the paper is then unfolded widthwise the paper will have four heart
cut-outs and eight rectangle cut-outs.
Since the paper is folded widthwise and then lengthwise, the second heart and second
two rectangles must mirror horizontally the cut heart and two rectangles and then the
last two hearts and four rectangles must mirror vertically the first two hearts and four
rectangles. This eliminates answer choices 2, 3, 4, and 5.
The remaining answer is the 1st choice and it is, therefore, the correct answer.

91. Explanation
The correct answer is 4.
 First, the rectangular paper is folded in half widthwise and then folded in half
lengthwise.
 Then, one trapezoid, triangle, and rectangle shape are cut out of the folded
paper.
 Consequently, when the paper is unfolded lengthwise the paper will have two
trapezoid cut-outs, two triangle cut-outs, and two rectangle cut-outs.
 When the paper is then unfolded widthwise the paper will have four trapezoid
cut-outs, four triangle cut-outs, and four rectangle cut-outs.
Since the paper is folded widthwise and then lengthwise, the second trapezoid, second
triangle, and second rectangle shapes must mirror horizontally the cut trapezoid,
triangle, and rectangle shapes, and then the final two sets of the trapezoid, triangle,
and rectangle shapes must mirror vertically the first two sets of trapezoid, triangle and
rectangle shapes. This eliminates answer choices 1, 2, 3, and 5.
Therefore, the 4th choice is the correct answer.

92. Explanation
The correct answer is 2.
 First, the rectangular paper is folded in half widthwise and then folded in half
lengthwise.
 Then, once the circle, rectangle, and triangle shapes are cut out of the folded
paper.

38
 Consequently, when the paper is unfolded lengthwise the paper will have two
circle cut-outs, two rectangle cut-outs, and two triangle cut-outs.
 When the paper is then unfolded widthwise the paper will have four circle
cut-outs, four rectangle cut-outs, and four triangle cut-outs.
Since the paper is folded widthwise and then lengthwise, the second circle, second
rectangle, and second triangle shapes must mirror horizontally the cut circle, rectangle,
and triangle shapes, and then the final two sets of the circle, rectangle, and triangle
shapes must mirror vertically the first two sets of circle, rectangle and triangle shapes.
This eliminates answer choices 1, 3, 4, and 5.
The only remaining option is the 2nd choice and it is, therefore, the correct answer.

93. Explanation
The correct answer is 1.

First the paper was folded diagonally from the lower-left corner to the upper-right
corner. Then it was folded again from the lower-right corner to the upper-left corner.
Then 3 holes in different shapes were punched out. Therefore, when the paper is
unfolded the holes will mirror each other across the diagonals of the page like this:

To solve this question, look for the answer choices in which the holes do not mirror
each other in the right way. The 2nd choice can be eliminated in this way as the
pie-shaped holes and triangular holes do not mirror each other.

Next, concentrate on the area of each remaining answer choice that appears in the
rightmost frame of the question series (in this case the triangular area on the top) and
eliminate all choices that are different. The 3rd choice can be eliminated as the
pie-shaped hole has been rotated. The 4th choice can also be eliminated as the
triangular hole has been rotated. Finally we can eliminate the 5th answer choice as the
pie-shaped and the triangular holes have switched places.

We are left with the 1st choice as the only correct answer.

94. Explanation
The correct answer is 4.
 First, the rectangular paper is folded in half widthwise.
 Then, three circular shapes are cut out of the folded paper.
 Consequently, the unfolded paper will have six circular cut-outs.
Since the paper is folded widthwise, the additional circular shapes must mirror
vertically the cut circular shapes. This eliminates answer choices 1, 2, 3, and 5.
We are left with the 4th choice, which is the correct answer.

39
95. Explanation
The correct answer is 2.

First, the rectangular paper is folded in half to make a triangle. Then, a trapezoid and
rectangle are cut out of the folded paper. Consequently, there will be two trapezoid
cut-outs and two rectangle cut-outs when the paper is unfolded again which eliminates
answer choices 1, 3 and 4. Also, the second trapezoid and second rectangle must be
rotated 90° clockwise and flipped vertically since the paper was folded along the
diagonal. This eliminates answer choice 5.

96. Explanation
The correct answer is 1.
 First, the square paper is folded in half widthwise.
 Then, two pie shapes are cut out of the folded paper.
 Therefore, the unfolded paper will have four pie shape cut-outs.
 Since the paper is folded widthwise, the shapes on the bottom must mirror the
cut pie shapes on the top.
In the 2nd answer choice, the pies at the bottom do not mirror the pies at the top.
In the 3rd answer choice, the pies at the bottom are exactly the same as the pies at the
top, and in the 4th and 5th answer choice, the top pies do not resemble the pies in the
questions.

We are left with the 1st choice as the correct answer.

97. Explanation
The correct answer is 2.
 First, the rectangular paper is folded in half to make a triangle.
 Then, three triangles are cut out of the folded paper.
 Consequently, there will be six triangle cut-outs when the paper is unfolded
again.
 Also, the three additional triangles must be rotated 90° clockwise and flipped
vertically since the paper was folded along the diagonal. This eliminates
answer choices 1, 3, 4, and 5.
We are left with the 2nd choice, which is the correct answer.

98. Explanation
The correct answer is 2.
 First, the rectangular paper is folded in half widthwise.
 Then, two rectangles and an oval are cut out of the folded paper.
 Consequently, the unfolded paper will have four rectangle cut-outs and two
oval cut-outs.
Since the paper is folded widthwise, the second two rectangles and second oval must
mirror vertically the cut two rectangles and oval. This eliminates answer choices 1, 3,
4, and 5.

40
We are left with the 2nd choice, which is the correct answer.

99. Explanation
The correct answer is 2.
 First, the rectangular paper is folded in half lengthwise and then folded in half
widthwise.
 Then, a moon is cut out of the folded paper.
 Consequently, when the paper is unfolded widthwise the paper will have two
moon cut-outs.
 When the paper is then unfolded lengthwise the paper will have four moon
cut-outs.
Since the paper is folded lengthwise and then widthwise, the second moon must
mirror vertically the cut moon, and then the last two moons must mirror horizontally
the first two moons. This eliminates answer choices 1, 3, 4, and 5.
We are left with the 2nd choice, which is the correct answer.

100. Explanation
The correct answer is 2.
 First, the rectangular paper is folded in half widthwise and then folded in half
lengthwise.
 Then, three circular shapes are cut out of the folded paper.
 Consequently, when the paper is unfolded lengthwise the paper will have six
circular cut-outs.
 When the paper is then unfolded widthwise the paper will have twelve circular
cut-outs.
Since the paper is folded widthwise and then lengthwise, the second set of three
circular shapes must mirror horizontally the three cut circular shapes, and then the
final two sets of three circular shapes must mirror vertically the first two sets of three
circular shapes. This eliminates answer choices 1, 3, 4, and 5.
We are left with the 2nd choice, which is the correct answer.

101. Explanation
The correct answer is 4.

Every time a paper is folded, the layer of paper doubles. Thus, creating a hole, or in
this case two shapes and fold it once, will create two separate pairs of shapes.
Moreover, if the paper is folded twice, four holes will appear.
 The paper in our question has been folded in half widthwise then folded again
in half lengthwise.
 After which, one oval and one rectangle were cut out of the folded paper.
 Consequently, when the paper has been unfolded lengthwise the paper will
have two oval cut-outs and two rectangle cut-outs.
 When the paper is then unfolded widthwise, the paper will have four oval
cut-outs and four rectangle cut-outs.

41
Folded paper creates a symmetry line. Therefore, after unfolding the previously
folded paper and creating holes in it, the unfolded paper will portray a mirror image
along that same symmetry line.

Since the paper was folded widthwise then lengthwise, the second oval and second
rectangle must mirror horizontally.
Likewise, the oval and rectangle and then the last two ovals and two rectangles must
mirror vertically the first two ovals and rectangles. This eliminates answer choices 1,
2, 3, and 5.

102. Explanation
The correct answer is 3.
 First, the rectangular paper is folded in half lengthwise and then folded in half
widthwise.
 Then, a triangle and half-oval are cut out of the folded paper.
 Consequently, when the paper is unfolded widthwise the paper will have two
triangle cut-outs and two half-oval cut-outs.
 When the paper is then unfolded lengthwise the paper will have four triangle
cut-outs and four half-oval cut-outs.
 Since the paper is folded lengthwise and then widthwise, the second triangle
and second half-oval must mirror vertically the cut triangles and half-oval, and
then the last two triangles and two half-ovals must mirror horizontally the first
two triangles and half-ovals. This eliminates answer choices 1, 2, 4, and 5.
Therefore, 3 is the correct answer.

103. Explanation
The correct answer is 5.
 First, the rectangular paper is folded in half widthwise and then folded in half
lengthwise.
 Then, one oval, triangle, and half-oval shape are cut out of the folded paper.
 Consequently, when the paper is unfolded lengthwise the paper will have two
oval cut-outs, two triangle cut-outs, and two half-oval cut-outs.
 When the paper is then unfolded widthwise the paper will have four oval
cut-outs, four triangle cut-outs, and four half-oval cut-outs.
Since the paper is folded widthwise and then lengthwise, the second set of two oval,
two triangle, and two half-oval shapes must mirror horizontally the cut oval, triangle,
and half-oval shapes, and then the final two sets of oval, triangle, and half-oval shapes
must mirror vertically the first two sets of oval, triangle and half-oval shapes. This
eliminates answer choices 1, 2, 3, and 4.
The only remaining option is the 5th choice and it is, therefore, the correct answer.

104. Explanation
The correct answer is 3.

42
 First, the rectangular paper is folded in half widthwise and then folded in half
lengthwise.
 Then, one arrow and two rectangles are cut out of the folded paper.
 Consequently, when the paper is unfolded lengthwise the paper will have two
arrow cut-outs and four rectangle cut-outs.
 When the paper is then unfolded widthwise the paper will have four arrow
cut-outs and eight rectangle cut-outs.
Since the paper is folded widthwise and then lengthwise, the second arrow and second
two rectangles must mirror horizontally the cut arrow and two rectangles, and then the
last two arrows and four rectangles must mirror vertically the first two arrows and
four rectangles. This eliminates answer choices 1, 2, 4, and 5.
The only remaining answer is the 3rd choice and it is, therefore, the correct answer.

105. Explanation
The correct answer is 3.
 First, the paper was folded diagonally from the upper-left corner to the
lower-right corner.
 Then it was folded again from the lower-left corner to the upper-right corner.
 Then 3 holes in different shapes were punched out.
 Therefore, when the paper is unfolded the holes will mirror each other across
the diagonals of the page like this:

To solve this question look for the answer choices in which the holes do not mirror
each other in the right way. The 2nd and 4th choices can be eliminated in this way.

Next, concentrate on the area of each remaining answer choice that appears in the
rightmost frame of the question series (in this case the triangular area on the right side)
and eliminate all choices that are different. The 1st choice can be eliminated as the
circular hole is under the pentagon hole but should be above it. The 5th choice can
also be eliminated as the pentagon has been rotated.

We are left with the 3rd choice as the only correct answer.

106. Explanation
The correct answer is 5.
 First, the paper was folded diagonally from the upper-right corner to the
lower-left corner.
 Then it was folded again from the upper-left corner to the lower-right corner.
 Then 3 holes in different shapes were punched out.

43
 Therefore, when the paper is unfolded the holes will mirror each other across
the diagonals of the page like this:

To solve this question, look for the answer choices in which the holes do not mirror
each other in the right way. The 3rd choice can be eliminated in this way as the
triangular holes do not mirror each other.

Next, concentrate on the area of each remaining answer choice that appears in the
rightmost frame of the question series (in this case the triangular area on the bottom)
and eliminate all choices that are different. The 1st choice can be eliminated as the
triangular hole has been rotated. The 2nd choice can also be eliminated as the circular
and the triangular holes have switched places. Finally, we can eliminate the 4th choice
as the diamond hole has been rotated (and is now square-shaped).

We are left with the 5th choice as the only correct answer.

107. Explanation

The correct answer is (E).


The paper has 1 hole punched and is folded 3 times.
If we open the first crease, which covers 1 hole, it results in the next picture having 2
holes.
If we open the second crease, which covers 1 hole, it results in the next picture having
3 holes.
If we open the second crease, which covers 3 holes, it results in the final picture
having 6 holes.​
Answers A, C, and D are incorrect because they do not have 6 holes.
Answer B is incorrect because of the hole placement (if you visualize the first fold
opening you can see it).

108. Explanation
The correct answer is (C).

The top and bottom edges of the papers are folded inward, then three holes are
punched in.

44
As folding of the paper creates a symmetry line, punching-in holes in a folded paper
makes a mirror image along that same symmetry line (after unfolding). This question
has two symmetry lines and, as they are horizontal, the holes also would be mirrored
horizontally. The top two holes would have a pair of two holes above them, as the
appropriate symmetry line is above them:

The lower hole has a symmetry line underneath it, and therefore it would have a
mirrored hole below it:

The only answer which follows this rule is answer (C), which is the correct answer.

Answer (A) and (B) are incorrect, as there are a holes above the bottom hole, instead
of below it.

Answer (D) is incorrect, as the hole in the bottom is diagonal, rather than directly
below the original hole.

Answer (E) is incorrect as its missing two holes in its top part.

109. Explanation
The correct answer is (B).

The paper is being folded twice and punched with two holes. However, it is important
to notice how many layers of papers are being punched with holes. In this question,
the paper is punched with two holes. Notice the top-right part of the paper was only
folded once (in the first transition), so it only has two layers of paper. On the other
hand, the hole to its left has four layers, as the paper on that point was folded twice.

Side note #1: Every folding of paper doubles the layers of paper.
As there are two holes punched, one on two layers of paper and the other on four
layers, you can expect there will be six holes in total. Thus, answers (A), (D) and (E)
can be ruled out (as they contain four, five and eight holes, respectively).

45
The remaining answers (B) and (C) are much alike and the difference between them is
the position of the two holes in the bottom-left side of the square.

Since the paper is being folded twice, there would be two symmetry lines which
would mirror the holes: the diagonal line in the first folding, and the horizontal line in
the second folding.
The unfolding of the paper is done, naturally, in the opposite order of the folding.
Therefore, the first unfolding would be across the horizontal line of the second
transition, and as it only affects the hole on the left, unfolding the paper across it will
result in a symmetric hole in the bottom of the paper:

This step is enough to rule out answer choice (C) as the hole there is not directly
beneath the left hole, thus leaving answer choice (B) as the only remaining option.
Alternatively, you can unfold the paper once more to see the end result; creating a
mirror-image across the diagonal line of the first transition:

Side note #2: A folding of the paper creates a symmetry line. Therefore, punching
holes in a folded paper makes a mirror image along that same symmetry line (after
unfolding).

110. Explanation

The correct answer is (B).


The paper has 1 hole punched and is folded 3 times.
If we open the first crease, which covers 1 hole, it results in the next picture having 2
holes.

46
If we open the second crease, which covers 2 holes, it results in the next picture
having 4 holes.
If we open the third crease, which covers 4 holes, it results in the final picture having
8 holes. ​
Answers C and E are incorrect because they do not have 8 holes.
Answer D is incorrect because the hole placement is wrong.

111. Explanation

The correct answer is (A).


The paper has 2 holes punched and is folded 3 times.
If we open the first crease, we expect the 2 holes to have doubled (there would be 4
holes in the next picture), but because the bottom-right hole is over a blank space,
there are only 3 holes in the next picture.
If we open the second crease, which covers 1 hole, it results in the paper having 4
holes in the next picture.
If we open the third crease, which covers all 4 holes, it results in 8 holes in the final
picture.
All other answers can be eliminated because they do not contain 8 holes.

112. Explanation

The correct answer is (D).


The paper has 2 triangles punched and is folded twice.
If we open the first crease, which covers 2 triangles, it results in the next picture
having 4 triangles.
If we open the second crease, which covers 1 triangle, it results in 5 triangles in the
final picture.
Answers A and B are incorrect because they do not have 5 triangles.
Answers C and E because you can see that if we open the first crease, the 2 triangles
should mirror each other, and they do not mirror each other in those choices hence are
incorrect.

47
113. Explanation
The correct answer is 5.

 First, the rectangular paper is folded in half widthwise and then folded in half
lengthwise.
 Then, one oval, triangle, and half-oval shape are cut out of the folded paper.
 Consequently, when the paper is unfolded lengthwise the paper will have two
oval cut-outs, two triangle cut-outs, and two half-oval cut-outs.
 When the paper is then unfolded widthwise the paper will have four oval
cut-outs, four triangle cut-outs, and four half-oval cut-outs.
Since the paper is folded widthwise and then lengthwise, the second set of two oval,
two triangle, and two half-oval shapes must mirror horizontally the cut oval, triangle,
and half-oval shapes, and then the final two sets of oval, triangle, and half-oval shapes
must mirror vertically the first two sets of oval, triangle and half-oval shapes. This
eliminates answer choices 1, 2, 3, and 4.
The only remaining option is the 5th choice and it is, therefore, the correct answer.

114. Explanation

The correct answer is (E).​


The paper has 2 holes punched and is folded 3 times.
If we open the first crease, which covers 1 hole, we have 3 holes in the next picture.
If we open the second crease, we expect the 3 holes to have doubled (there would be 6
holes in the next picture), but because the lower left hole covers a blank space, we
have 5 holes in the next picture.
If we open the third crease, which covers only 2 holes, it results in 7 holes in the final
picture.
All other answers can be eliminated because they do not contain 7 holes.

115. Explanation

The correct answer is (D).

48
The paper has 1 pentagon and 1 triangle punched and is folded twice.
If we open the first crease, which covers both the pentagon and the triangle, it results
in the next picture having 2 pentagons and 2 triangles.
If we open the second crease, which covers 2 triangles, it results in the final picture
having 4 triangles and 2 pentagons.
Answers A, B and E are incorrect because you can see that if we open the first crease,
the 2 figures should mirror each other, and they do not mirror each other in those
choices hence are incorrect.
Answer C is incorrect because you can see that if we open the second crease, the 2
triangles should mirror each other, and they do not mirror each other in answer choice
C, so therefore are incorrect.

116. Explanation

The correct answer is (E).


The paper has 2 holes punched and is folded 3 times
When we open the first crease, which covers 2 holes, we expect the next picture to
have 4 holes, but because the top-right hole is covering blank space, the next picture
has 3 holes.
If we open the second crease, which covers 0 holes, it results in the next picture
having 3 holes.
If we open the third crease, which covers 1 hole, it results in the final picture having 4
holes. ​

117. Explanation
The correct answer is 2.

 First, the rectangular paper is folded in half widthwise and then folded in half
lengthwise.
 Then, once the circle, rectangle, and triangle shapes are cut out of the folded
paper.
 Consequently, when the paper is unfolded lengthwise the paper will have two
circle cut-outs, two rectangle cut-outs, and two triangle cut-outs.
 When the paper is then unfolded widthwise the paper will have four circle
cut-outs, four rectangle cut-outs, and four triangle cut-outs.
Since the paper is folded widthwise and then lengthwise, the second circle, second
rectangle, and second triangle shapes must mirror horizontally the cut circle, rectangle,
and triangle shapes, and then the final two sets of the circle, rectangle, and triangle

49
shapes must mirror vertically the first two sets of circle, rectangle and triangle shapes.
This eliminates answer choices 1, 3, 4, and 5.
The only remaining option is the 2nd choice and it is, therefore, the correct answer.

118. Explanation

The correct answer is (C).


The paper has 1 pentagon and 1 triangle punched and then folded three times.
If we open the first crease, which covers both the pentagon and the triangle, it results
in the picture having 2 pentagons and 2 triangles.
If we open the second crease, which covers 1 pentagon and 2 triangles, it results in the
next picture having 4 triangles and 3 pentagons.
If we open the third crease, which covers 1 triangle, it results in the final picture
having 5 triangles and 3 pentagons.
Answers A and B are incorrect because they do not have 5 triangles and 3 pentagons.
Answer E is incorrect because if we open the first crease, the 2 figures should mirror
each other, and they do not mirror hence are incorrect.

119. Explanation
The correct answer is 3

 First, the paper was folded diagonally from the upper-left corner to the
lower-right corner.
 Then it was folded again from the lower-left corner to the upper-right corner.
 Then 3 holes in different shapes were punched out.
 Therefore, when the paper is unfolded the holes will mirror each other across
the diagonals of the page like this:

To solve this question look for the answer choices in which the holes do not mirror
each other in the right way. The 2nd and 4th choices can be eliminated in this way.

Next, concentrate on the area of each remaining answer choice that appears in the
rightmost frame of the question series (in this case the triangular area on the right side)
and eliminate all choices that are different. The 1st choice can be eliminated as the
circular hole is under the pentagon hole but should be above it. The 5th choice can

50
also be eliminated as the pentagon has been rotated.

We are left with the 3rd choice as the only correct answer.

120. Explanation

The correct answer is (C).


The paper has 1 hole punched and is folded 3 times.
If we open the first crease, which covers 1 hole, it results in the next picture having 2
holes.
If we open the second crease, which covers 0 holes, it results in the next picture
having 2 holes.
If we open the third crease, which covers 1 hole, it results in the final picture having 3
holes.
Answers A and B are incorrect.

121. Explanation

The correct answer is (D).


The paper has 1 hole punched and is folded 3 times.
If we open the first crease, which covers 1 hole, it results in the next picture having 2
holes.
If we open the second crease, which covers 2 holes, we expect the next picture to have
4 holes, but because the lower left hole in the second picture covers a blank space, the
third picture has 3 holes.
If we open the third crease, which covers 3 holes, it results in the final picture having
6 holes.

51
122. Explanation

The correct answer is (C).


The paper has 1 arrow and 1 hole punched and is folded 3 times.
If we open the first crease, which covers both the arrow and the hole, we expect in the
next picture to have 2 holes and 2 arrows but because the hole has blank space
underneath it, the next picture has 2 arrows and 1 hole.
If we open the second crease, which covers 1 arrow, it results in the next picture
having 3 arrows and 1 hole.
If we open the third crease, which covers 3 arrows and 1 hole, it results in the final
picture having 6 arrows and 2 holes.

123. Explanation
The correct answer is 4.

Every time a paper is folded, the layer of paper doubles. Thus, creating a hole, or in
this case two shapes and fold it once, will create two separate pairs of shapes.
Moreover, if the paper is folded twice, four holes will appear.

 The paper in our question has been folded in half widthwise then folded again
in half lengthwise.
 After which, one oval and one rectangle were cut out of the folded paper.
 Consequently, when the paper has been unfolded lengthwise the paper will
have two oval cut-outs and two rectangle cut-outs.
 When the paper is then unfolded widthwise, the paper will have four oval
cut-outs and four rectangle cut-outs.
Folded paper creates a symmetry line. Therefore, after unfolding the previously
folded paper and creating holes in it, the unfolded paper will portray a mirror image
along that same symmetry line.

Since the paper was folded widthwise then lengthwise, the second oval and second
rectangle must mirror horizontally.
Likewise, the oval and rectangle and then the last two ovals and two rectangles must
mirror vertically the first two ovals and rectangles. This eliminates answer choices 1,
2, 3, and 5.

124. Explanation

52
The correct answer is 5.

 First, the rectangular paper is folded in half widthwise and then folded in half
lengthwise.
 Then, one triangle is cut out of the folded paper.
 Consequently, when the paper is unfolded lengthwise the paper will have two
triangle cut-outs. When the paper is then unfolded widthwise the paper will
have four triangle cut-outs.
Since the paper is folded widthwise and then lengthwise, the second triangle must
mirror horizontally the cut triangle, and then the last two triangles must mirror
vertically the first two triangles. This eliminates answer choices 1, 2, 3, and 4.
Therefore, we are left with the 5th choice, which is the correct answer.

125. Explanation
The correct answer is 3.

 First, the rectangular paper is folded in half lengthwise and then folded in half
widthwise.
 Then, a triangle and half-oval are cut out of the folded paper.
 Consequently, when the paper is unfolded widthwise the paper will have two
triangle cut-outs and two half-oval cut-outs.
 When the paper is then unfolded lengthwise the paper will have four triangle
cut-outs and four half-oval cut-outs.
 Since the paper is folded lengthwise and then widthwise, the second triangle
and second half-oval must mirror vertically the cut triangles and half-oval, and
then the last two triangles and two half-ovals must mirror horizontally the first
two triangles and half-ovals. This eliminates answer choices 1, 2, 4, and 5.
Therefore, 3 is the correct answer.

126. Explanation
The correct answer is 1.

 First, the square paper is folded in half widthwise and then folded in half
lengthwise.
 Then, one arrow, triangle, and rectangle shape are cut out of the folded paper.
 Consequently, when the paper is unfolded lengthwise the paper will have two
arrow cut-outs, two triangle cut-outs and two rectangle cut-outs.
 When the paper is then unfolded widthwise the paper will have four arrow
cut-outs, four triangle cut-outs, and four rectangle cut-outs.
Since the paper is folded widthwise and then lengthwise, the second arrow, second
triangle, and second rectangle shapes must mirror horizontally the cut arrow, triangle,
and rectangle, and then the final two sets of the arrow, triangle, and rectangle shapes
must mirror vertically the first two sets of arrow, triangle, and rectangle shapes. This
eliminates answer choices 2, 3, 4, and 5.

53
Thus, the only remaining possible answer is the 1st choice, and it is the correct
answer.

127. Explanation
The correct answer is 5.

First, the paper was folded in half lengthwise, and then both outer (left)
corners were folded towards the center of the square.
Then 2 holes were cut out.
Both holes go through four layers of paper, so the answer should include eight holes:
2 holes x 4 layers = 8.

A good rule-of-thumb is to look for the original holes in the answer choices, and first
eliminate the answers that do not include them.
When we compare the holes in the answer choices to the holes in the original image
above, we can immediately eliminate answer choices 1 and 2 because they do not
contain the holes in the original image.
Answer choice 4 has fewer than eight holes, so it can be eliminated as well.
Answer choice 3 has more than eight holes - notice the added hole in the center - so it
can be eliminated too.
We are left with the 5th choice, which is the correct answer.

128. Explanation
The correct answer is 1.

The paper was folded from each corner towards the center of the square.
The folds do not meet or overlap, creating two layers of paper in each corner.
Then 4 holes were cut out.
All four holes go through two layers of paper, so the answer should include eight
holes (4 holes x 2 layers).

Answer choices 2 and 5 have fewer than eight holes, so they can be eliminated.
Also, the paper in answer choice 5 hasn’t been unfolded.
Answer choices 3 and 4 have holes in all four corners of the square and not only three
as in the image above.

We are left with the 1st choice, which is the only correct answer.

54
129. Explanation

The correct answer is (E).​


The paper has 2 holes punched and is folded 4 times.
If we open the first crease, which covers 2 holes, it results in 4 holes in the next
picture.
When we open the second crease, which covers 4 holes, we expect the 4 holes to have
doubled in the next picture, but because the bottom left and right holes in the second
picture cover blank spaces, the next picture has 6 holes.
If we open the third crease, which covers 1 hole, it results in the next picture having 7
holes.
If we open the 4th crease, which covers 1 hole, it results in the next picture having 8
holes.
The other answers are incorrect because they do not have 8 holes.

130. Explanation
The correct answer is 3.

First the paper was folded in half widthwise, and then the left outer (bottom)
corner was folded diagonally towards the center of the square.
Then 1 hole was cut out.
Since the paper was folded twice, the hole goes through four layers of paper.
Therefore, the answer should include four holes: 1 hole x 4 layers = 4.

A good rule-of-thumb is to look for the original holes in the answer choices, and first
eliminate the answers that do not include them.
When we compare the holes in the answer choices to the holes in the original image
above, we can immediately eliminate answer choice 4 because it does not contain the
holes in the original image.
Answer choice 1 has fewer than four holes, so it can be eliminated too.
Answer choices 2 and 5 have more than four holes, so they can be eliminated as well.
We are left with the 3rd choice, which is the correct answer.

131. Explanation
The correct answer is 4.

First the paper was folded widthwise from the top of the square towards the middle.
Then the outer (bottom) corners were folded towards the center of the square, and 3
holes were cut out.

55
The folds do not overlap, so the paper was folded only once in each section.
Therefore, the holes go through two layers of paper, and the answer should
include six holes: 3 holes x 2 layers = 6.

Answer choices 3 and 5 have fewer than six holes, so they can be eliminated.
Answer choice 1 has the right number of holes, but they are in the wrong places –
notice that in the original image above there are no holes on the bottom left corner of
the folded paper, so there should not be holes in that corner in the unfolded paper.
Answer choice 2 can be eliminated because the lowest hole is in line with the other
holes. Since the fold in that corner is diagonal, the hole should be below and on
the right of the other holes.
We are left with the 4th choice, which is the correct answer.

132. Explanation
The correct answer is 4.

The square paper is first folded in half diagonally, and then five holes are cut out of
the folded paper.

Since the paper was folded in half once, the holes were cut through two layers of
paper.
Therefore, when the paper is unfolded it will have ten holes: 5 x 2 = 10.

A good rule-of-thumb is to look for the original holes in the answer choices, and first
eliminate the answers that do not include them.
When we compare the holes in the answer choices to the holes in the original image
above, we can immediately eliminate answer choices 2, 3, and 5 because they do not
contain vertical hole groups in the center of the square.

In answer choice 1 we have the right number of holes, but they are also in the wrong
positions – when we compare the upper-right corner of this answer choice with the
one in the above image, we can see that the two-hole and three-hole groups have
switched places.

We are left with the 4th answer choice, which is the correct answer.

133. Explanation
The correct answer is 4.

The square paper was first folded in half diagonally to create a triangle and
then folded again from each corner towards the middle of the triangle.
Then, four holes were cut out of the folded paper.

All four holes go through four layers of paper.

56
Therefore, the answer should have sixteen holes (4 holes x 4 layers).
The holes will flip over the fold each time the paper is unfolded – first upwards and
to the right, and then diagonally.

Answer choices 1, 2, 3, and 5 include less than 16 holes, so they can be eliminated.

We are left with the 4th answer choice, which is the correct answer.

134. Explanation
The correct answer is 4.

First, the square paper was folded in half diagonally from the bottom right corner to
the top left corner, and then folded again diagonally from the top left corner to the
middle of the square.
Then, seven holes were punched in the folded paper.
The three left holes and the three right holes go through two layers of paper since the
paper is folded once where they were punched.
The middle hole goes through four layers since the paper is folded twice there.
Therefore, when the paper is unfolded it will have sixteen holes: 6 holes x 2 layers + 1
hole x 4 layers = 16.

Answers 1 and 2 are incorrect because they have less than sixteen holes.
Answer 3 is incorrect because there should be a hole at the top left corner and at the
bottom right corner, since one hole was punched through the part that when unfolded
reaches both corners of the square paper (the middle hole):

Answer 5 is incorrect because the holes at the corners are too close to the middle of
the square. The middle hole was punched right near the edge of the 2nd fold, therefore
the holes on both corners should be near the edge accordingly.

We are left with the 4th answer choice, which is the correct answer.

135. Explanation
The correct answer is 2.

First the paper was folded in half diagonally from the upper-left corner to the
lower-right corner.

57
Then it was folded again from the lower-left corner to the upper-right corner.
Then four holes were cut out of four layers of paper.

Therefore, when the paper is unfolded it will have sixteen holes (4 holes x 4 layers)
which will mirror each other across the diagonals of the paper like this:

A good rule-of-thumb is to look for the original holes in the answer choices, and first
eliminate the answers that do not include them.
When we compare the holes in the answer choices to the holes in the original image
above, we can immediately eliminate answer choices 3 and 5 because they do not
contain the holes in the original image.
Answer choice 4 has fewer than sixteen holes, so it can also be eliminated.
Looking at answer choice 1, we can see that although it has the right number of holes,
the holes are in the wrong places – for example, the holes at the bottom of the square
are too far from the holes on the right and do not mirror them.
We are left with the 2nd choice, which is the correct answer.

136. Explanation

The correct answer is (D).


The paper has 1 hole punched and is folded 3 times.
When we open the first crease, which covers 1 hole, it results in the next picture
having 2 holes.
When we open the second crease, which covers 2 holes, it results in the next picture
having 4 holes.
When we open the third crease, which covers 1 hole, it results in the final picture
having 5 holes.

137. Explanation
The correct answer is 4.

 First, the rectangular paper is folded in half widthwise and then folded in half
lengthwise.

58
 Then, one trapezoid, triangle, and rectangle shape are cut out of the folded
paper.
 Consequently, when the paper is unfolded lengthwise the paper will have two
trapezoid cut-outs, two triangle cut-outs, and two rectangle cut-outs.
 When the paper is then unfolded widthwise the paper will have four trapezoid
cut-outs, four triangle cut-outs, and four rectangle cut-outs.
Since the paper is folded widthwise and then lengthwise, the second trapezoid, second
triangle, and second rectangle shapes must mirror horizontally the cut trapezoid,
triangle, and rectangle shapes, and then the final two sets of the trapezoid, triangle,
and rectangle shapes must mirror vertically the first two sets of trapezoid, triangle and
rectangle shapes. This eliminates answer choices 1, 2, 3, and 5.
Therefore, the 4th choice is the correct answer.

138. Explanation

The correct answer is (B).


The paper has 1 pentagon punched and is folded 3 times.
Because the folds are all symmetrical folds, the final picture results in 8 pentagons

139. Explanation

The correct answer is (C).


The paper has 2 holes and is folded 3 times.
If we open the first crease which covers 2 holes, it results in the next figure having 4
holes.
If we open the second crease, which covers 2 holes, it results in the next picture
having 6 holes.
If we open the third crease, which covers 0 holes, it does not impact the number of
holes in the final picture​ , so the last picture has 6 holes.
Answers A and E are incorrect because they do not have 6 holes
Answers B and D are incorrect because when we open the second crease, we can see
that there should be 4 holes on the bottom right corner of the paper which these 2
answer choices do not have.

59
140. Explanation
The correct answer is 5.

 First, the paper was folded diagonally from the upper-right corner to the
lower-left corner.
 Then it was folded again from the upper-left corner to the lower-right corner.
 Then 3 holes in different shapes were punched out.
 Therefore, when the paper is unfolded the holes will mirror each other across
the diagonals of the page like this:

To solve this question, look for the answer choices in which the holes do not mirror
each other in the right way. The 3rd choice can be eliminated in this way as the
triangular holes do not mirror each other.

Next, concentrate on the area of each remaining answer choice that appears in the
rightmost frame of the question series (in this case the triangular area on the bottom)
and eliminate all choices that are different. The 1st choice can be eliminated as the
triangular hole has been rotated. The 2nd choice can also be eliminated as the circular
and the triangular holes have switched places. Finally, we can eliminate the 4th choice
as the diamond hole has been rotated (and is now square-shaped).

We are left with the 5th choice as the only correct answer.

141. Explanation
The correct answer is 3.

First, the paper was folded in half diagonally from the upper-right corner to the
lower-left corner. Then it was folded again from the upper-left corner to the
lower-right corner.
Then two holes were cut out of four layers of paper.

Therefore, when the paper is unfolded it will have eight holes (2 holes x 4 layers)
which will mirror each other across the diagonals of the paper like this:

60
Answer choices 1, 4, and 5 have fewer than eight holes, so they can be eliminated.
If you look closely at answer choice 2, you can see that although it has the right
number of holes, they are in the wrong places: the holes are located closer to the
edges of the paper than in the image above.

We are left with the 3rd choice, which is the correct answer.

142. Explanation
The correct answer is 3.

First the paper was folded in half lengthwise, and then both outer (bottom)
corners were folded towards the center of the square.
Then 2 holes were cut out.
Both holes go through four layers of paper, so the answer should include eight holes:
2 holes x 4 layers = 8.

A good rule-of-thumb is to look for the original holes in the answer choices, and first
eliminate the answers that do not include them.
When we compare the holes in the answer choices to the holes in the original image
above, we can immediately eliminate answer choices 2 and 4 because they do not
contain the holes in the original image.
Answer choice 5 has fewer than eight holes, so it can be eliminated too.
Answer choice 1 has more than eight holes - notice the added hole in the center - so it
can be eliminated as well.

We are left with the 3rd choice, which is the only correct answer.

143. Explanation

The correct answer is (B).


The paper has one hole punched and is folded four times.
If we open the first crease which covers the hole, it will result in the following picture
with two holes.
If we open the second crease, which covers the two holes, it will result in the
following picture with four holes.
If we open the third crease, which you may notice does not cover any of the holes, it
will result in the following picture with the same four holes.
If we open the fourth crease, which covers all four holes, it will result in a final
picture with eight holes: four on the bottom right corner and the other four on the top
left corner.

61
144. Explanation
The correct answer is 5.

 First, the rectangular paper is folded in half lengthwise and then folded in half
widthwise.
 Then, a triangle and pie are cut out of the folded paper.
 Consequently, when the paper is unfolded widthwise the paper will have two
triangle cut-outs and two pie cut-outs.
 When the paper is then unfolded lengthwise the paper will have four triangle
cut-outs and four pie cut-outs.
Since the paper is folded lengthwise and then widthwise, the second triangle and
second pie must mirror vertically the cut triangle and pie, and then the last two
triangles and two pies must mirror horizontally the first two triangles and pies. This
eliminates answer choices 1, 2, 3, and 4.
Thus, the only possible answer is the 5th choice, and it is the correct answer.

145. Explanation
The correct answer is 1.

 First, the rectangular paper is folded in half widthwise and then folded in half
lengthwise.
 Then, one heart and two rectangles are cut out of the folded paper.
 Consequently, when the paper is unfolded lengthwise the paper will have two
heart cut-outs and four rectangle cut-outs.
 When the paper is then unfolded widthwise the paper will have four heart
cut-outs and eight rectangle cut-outs.
Since the paper is folded widthwise and then lengthwise, the second heart and second
two rectangles must mirror horizontally the cut heart and two rectangles and then the
last two hearts and four rectangles must mirror vertically the first two hearts and four
rectangles. This eliminates answer choices 2, 3, 4, and 5.
The remaining answer is the 1st choice and it is, therefore, the correct answer.

146. Explanation
The correct answer is 1

First the paper was folded diagonally from the lower-left corner to the upper-right
corner. Then it was folded again from the lower-right corner to the upper-left corner.
Then 3 holes in different shapes were punched out. Therefore, when the paper is
unfolded the holes will mirror each other across the diagonals of the page like this:

62
To solve this question, look for the answer choices in which the holes do not mirror
each other in the right way. The 2nd choice can be eliminated in this way as the
pie-shaped holes and triangular holes do not mirror each other.

Next, concentrate on the area of each remaining answer choice that appears in the
rightmost frame of the question series (in this case the triangular area on the top) and
eliminate all choices that are different. The 3rd choice can be eliminated as the
pie-shaped hole has been rotated. The 4th choice can also be eliminated as the
triangular hole has been rotated. Finally we can eliminate the 5th answer choice as the
pie-shaped and the triangular holes have switched places.

We are left with the 1st choice as the only correct answer.

147. Explanation
The correct answer is 3.

First, the paper was folded in half diagonally from the upper-right corner to the
lower-left corner. Then it was folded again from the upper-left corner to the
lower-right corner.
Then two holes were cut out of four layers of paper.

Therefore, when the paper is unfolded it will have eight holes (2 holes x 4 layers)
which will mirror each other across the diagonals of the paper like this:

Answer choices 1, 4, and 5 have fewer than eight holes, so they can be eliminated.
If you look closely at answer choice 2, you can see that although it has the right
number of holes, they are in the wrong places: the holes are located closer to the
edges of the paper than in the image above.

We are left with the 3rd choice, which is the correct answer.

148. Explanation
The correct answer is 3.

First the paper was folded in half lengthwise, and then both outer (bottom)
corners were folded towards the center of the square.
Then 2 holes were cut out.

63
Both holes go through four layers of paper, so the answer should include eight holes:
2 holes x 4 layers = 8.

A good rule-of-thumb is to look for the original holes in the answer choices, and first
eliminate the answers that do not include them.
When we compare the holes in the answer choices to the holes in the original image
above, we can immediately eliminate answer choices 2 and 4 because they do not
contain the holes in the original image.
Answer choice 5 has fewer than eight holes, so it can be eliminated too.
Answer choice 1 has more than eight holes - notice the added hole in the center - so it
can be eliminated as well.

We are left with the 3rd choice, which is the only correct answer.

149. Explanation

The correct answer is (B).


The paper has one hole punched and is folded four times.
If we open the first crease which covers the hole, it will result in the following picture
with two holes.
If we open the second crease, which covers the two holes, it will result in the
following picture with four holes.
If we open the third crease, which you may notice does not cover any of the holes, it
will result in the following picture with the same four holes.
If we open the fourth crease, which covers all four holes, it will result in a final
picture with eight holes: four on the bottom right corner and the other four on the top
left corner.

150. Explanation
The correct answer is 5.

 First, the rectangular paper is folded in half lengthwise and then folded in half
widthwise.
 Then, a triangle and pie are cut out of the folded paper.
 Consequently, when the paper is unfolded widthwise the paper will have two
triangle cut-outs and two pie cut-outs.
 When the paper is then unfolded lengthwise the paper will have four triangle
cut-outs and four pie cut-outs.
Since the paper is folded lengthwise and then widthwise, the second triangle and
second pie must mirror vertically the cut triangle and pie, and then the last two

64
triangles and two pies must mirror horizontally the first two triangles and pies. This
eliminates answer choices 1, 2, 3, and 4.
Thus, the only possible answer is the 5th choice, and it is the correct answer.

151. Explanation
The correct answer is 1.

 First, the rectangular paper is folded in half widthwise and then folded in half
lengthwise.
 Then, one heart and two rectangles are cut out of the folded paper.
 Consequently, when the paper is unfolded lengthwise the paper will have two
heart cut-outs and four rectangle cut-outs.
 When the paper is then unfolded widthwise the paper will have four heart
cut-outs and eight rectangle cut-outs.
Since the paper is folded widthwise and then lengthwise, the second heart and second
two rectangles must mirror horizontally the cut heart and two rectangles and then the
last two hearts and four rectangles must mirror vertically the first two hearts and four
rectangles. This eliminates answer choices 2, 3, 4, and 5.
The remaining answer is the 1st choice and it is, therefore, the correct answer.

152. Explanation
The correct answer is 1

First the paper was folded diagonally from the lower-left corner to the upper-right
corner. Then it was folded again from the lower-right corner to the upper-left corner.
Then 3 holes in different shapes were punched out. Therefore, when the paper is
unfolded the holes will mirror each other across the diagonals of the page like this:

To solve this question, look for the answer choices in which the holes do not mirror
each other in the right way. The 2nd choice can be eliminated in this way as the
pie-shaped holes and triangular holes do not mirror each other.

Next, concentrate on the area of each remaining answer choice that appears in the
rightmost frame of the question series (in this case the triangular area on the top) and
eliminate all choices that are different. The 3rd choice can be eliminated as the
pie-shaped hole has been rotated. The 4th choice can also be eliminated as the
triangular hole has been rotated. Finally we can eliminate the 5th answer choice as the
pie-shaped and the triangular holes have switched places.

We are left with the 1st choice as the only correct answer.

65
153. Explanation
The correct answer is 4.

The paper was folded from each corner towards the center of the square.
The folds do not meet or overlap, creating two layers of paper in each corner.
Then four holes were cut out.
All four holes go through two layers of paper, so the answer should include eight
holes (4 holes x 2 layers).

Answer choices 3 and 5 have fewer than eight holes, so they can be eliminated.
Also, the square in answer choice 3 hasn’t been unfolded.
Answer choices 1 and 2 have holes in all four corners of the square and not only three
corners as in the image above, so they too can be eliminated.

We are left with the 4th choice, which is the correct answer.

154. Explanation
The correct answer is 5.

The square paper was first folded in half diagonally to create a triangle, and
then folded again from each corner towards the middle of the triangle.
Then, four holes were cut out of the folded paper.

All four holes go through four layers of paper.


Therefore, the answer should have sixteen holes (4 holes x 4 layers).
The holes will flip over the fold each time the paper is unfolded – first upwards and
to the left, and then diagonally.

Answer choices 1, 2, and 3 have less than 16 holes, so they can be eliminated.
Answer choice 4 does not mirror correctly the holes in the folded paper above.

We are left with the 5th answer choice, which is the correct answer.

155. Explanation

66
The correct answer is (E).
The paper has 1 hole and 1 triangle punched and is folded 4 times.
When we open the first crease, which covers 1 triangle and 1 hole, we expect both
figures to have doubled in the next picture (2 triangles and 2 holes), but because the
triangle has blank space underneath it, the next picture has 1 triangle and 2 holes.
When we open the second crease, which covers 2 holes and 1 triangle, we expect the
figures to have doubled in the next picture (4 holes and 2 triangles), but underneath
the triangle and the dot next to it is blank space, so in the next picture there are 3 holes
and 1 triangle.
When we open the third crease, which covers 3 holes and 1 triangle, it results in the
final picture having 2 triangles and 6 holes.

156. Explanation
The correct answer is 4.

 The square paper was first folded diagonally in half, and then three holes were
cut out of the folded paper.
 Since the paper was folded in half once, the holes were cut through two layers
of paper.
 Therefore, when the paper is unfolded it will have six holes: 3 x 2 = 6.
Answer choices 1, 2, 3, and 5 have fewer than six holes, so they can each be
eliminated.

Therefore, we are left with the 4th answer choice, which is the correct answer.

157. Explanation
The correct answer is 3.

Here we have a square with two of its corners folded inwards without meeting or
overlapping, creating two layers of paper in the upper-left and lower-right areas of
the square.
Notice that the three holes aligned from the upper-right corner to the lower-left
corner only go through one layer of paper, but the two other holes go through two
layers.
Therefore, the answer should include seven holes (3 holes + 2 holes x 2 layers =7).

Answer choices 1 and 2 have fewer than seven holes, so they can be eliminated.
Answer choice 4 has more than seven holes and the square has not been unfolded – so
it can be eliminated as well.
Answer choice 5 also has more than seven holes, and can therefore be eliminated.

We are left with the 3rd choice, which is the correct answer.

67
158. Explanation

The correct answer is (E).​


The paper has 2 holes punched and is folded 3 times.
When we open the first crease (from left to right), the holes have doubled because the
crease covers both holes, which results in 4 holes being punched.
If we open the second crease, we can see it only covers 1 hole, which results in the
paper having 5 holes punched.
In the last crease, it is the same as the second crease, which results in the paper being
punched with 6 holes.
Answer C can be eliminated since it includes fewer than 6 holes.

159. Explanation
The correct answer is 4.

First the paper was folded in half lengthwise, and then the outer upper
corner was folded diagonally towards the center of the square.
Then 1 hole was cut out.
Since the paper was folded twice, the hole goes through four layers of paper.
Therefore, the answer should include four holes: 1 hole x 4 layers = 4.

Answer choice 3 has fewer than four holes, so it can be eliminated.


Answer choice 5 has more than four holes, so it can be eliminated as well.
Answer choices 1 and 2 have the right number of holes, but they are in the wrong
places - when the paper is unfolded diagonally there should be holes very close to
edge of the square, and none of these answers have such holes.

We are left with the 4th choice, which is the correct answer.

160. Explanation
The correct answer is 3.

 First, the rectangular paper is folded in half widthwise and then folded in half
lengthwise.
 Then, one arrow and two rectangles are cut out of the folded paper.
 Consequently, when the paper is unfolded lengthwise the paper will have two
arrow cut-outs and four rectangle cut-outs.

68
 When the paper is then unfolded widthwise the paper will have four arrow
cut-outs and eight rectangle cut-outs.
Since the paper is folded widthwise and then lengthwise, the second arrow and second
two rectangles must mirror horizontally the cut arrow and two rectangles, and then the
last two arrows and four rectangles must mirror vertically the first two arrows and
four rectangles. This eliminates answer choices 1, 2, 4, and 5.
The only remaining answer is the 3rd choice and it is, therefore, the correct answer.

161. Explanation

The correct answer is (C).


The paper has 1 hole punched and is folded twice.
If we open the first crease (from left to right), which only covers 1 hole, it results in
the paper having 2 holes in the next picture.
If we open the second crease, which covers both holes, it results in the paper having 4
holes. Answers A and B can be eliminated because they do not have 4 holes.

162. Explanation

The correct answer is (E).​


The paper has 2 holes punched and is folded twice.
If we open the first crease, which covers 2 holes, it results in 4 holes in the next
picture.
If we open the second crease, which covers 1 hole, it results in 5 holes in the final
picture.
Answer A is incorrect because the placement of the holes is incorrect.

163. Explanation
The correct answer is 3.

The square paper was first folded in half diagonally, and then folded
again from the upper-left corner towards the lower-left corner.
Then, two holes were cut out of the folded paper.

69
The upper hole goes through four layers of paper since the paper is
folded twice where it is cut.
The lower hole goes through only two layers since the paper is folded once there.
Therefore, the answer should have six holes: 1 hole x 2 layers + 1 hole x 4 layers = 6.
The holes will flip over the fold each time the paper is unfolded – first upwards and
then diagonally.

Answer choices 1, 2 and 4 include fewer than six holes, so they can be eliminated.
Answer choice 5 has the right number of holes, but they are in the wrong places – this
can be seen when we compare the holes on the lower-left corner to the holes in the
image above.

We are left with the 3rd answer choice, which is the correct answer.

164. Explanation
The correct answer is 4.

 First, the square paper was folded in half widthwise, and then it was folded in
half again lengthwise.
 Then, three holes were cut out of the folded paper.
 Since the paper was folded in half twice, the holes were cut through four
layers of paper.
 Therefore, when the paper is unfolded it will have twelve holes: 3 x 4 = 12.
 Since the paper was folded in half widthwise and then again in half lengthwise,
each quarter must mirror the quarters above or below it and the quarter to its
left or right.
Answer choices 1, 3, and 5 include fewer than 12 holes, so they can be eliminated.
If you look closely at answer choice 2, you can see that the quarters do not mirror
each other in this way. The holes in the top half are positioned exactly the same as
those in the bottom half instead of mirroring them.

We are left with the 4th answer choice, which is the correct answer.

165. Explanation

The correct answer is (C).


The paper has 2 holes punched and is folded twice.
If we open the first crease, we expect the 2 holes to have doubled (so there would be 4
holes in the next picture), but because the upper hole in the first picture has a blank
space underneath it, there are only 3 holes in the next picture.

70
If we open the second crease, which covers all 3 holes, then it results in the last
picture having 6 holes.
​ All other answers are incorrect because they do not have 6 holes.

166. Explanation

The correct answer is (E).


The paper has 2 holes punched and is folded twice.
When we open the first crease (from left to right), we expect the holes to have
doubled (there would be 4 holes in the next picture), but because the upper-left hole is
punched on a blank surface, there are only 3 holes in the next picture.
If we open the second crease, we can see it covers only 1 hole, which results in the
paper having 4 holes.
Answers A and C can be eliminated since they include fewer than 4 holes.
The unfolded product of the first fold (which is unfolded second in the illustration
above) should be diagonal to the punched hole, and therefore answers B and D do not
match, and the only possible answer left is E, which is the correct answer.

167. Explanation

The correct answer is (E).


The paper has 1 hole and 1 triangle punched and is folded 4 times.
If we open the first crease, which covers a hole and triangle, it results in the next
picture having 2 holes and 2 triangles.
If we open the second crease, which covers 2 holes and 2 triangles, it results in the
next picture having 4 holes and 4 triangles.
If we open the third crease, which covers 2 holes, it results in the next picture having
4 triangles and 6 holes.
If we open the 4th crease, which covers 2 holes, it results in a final picture with 8
holes and 4 triangles.

168.Explanation

71
The correct answer is (E).
The paper has 2 holes punched and is folded three times.
When we open the first crease, we expect the 2 holes to have doubled (there would be
4 holes in the next picture), but because the lower left hole covers a blank space, the
next picture has 3 holes.
When we open the second crease, which covers 2 holes, it results in the next picture
having 5 holes.
When we open the third crease, which covers 0 holes, it results in the last picture
having 5 holes.
All the other answers are incorrect because they do not have 5 holes.

169. Explanation
The correct answer is 2.

 First, the rectangular paper is folded in half widthwise and then folded in half
lengthwise.
 Then, three circular shapes are cut out of the folded paper.
 Consequently, when the paper is unfolded lengthwise the paper will have six
circular cut-outs.
 When the paper is then unfolded widthwise the paper will have twelve circular
cut-outs.
Since the paper is folded widthwise and then lengthwise, the second set of three
circular shapes must mirror horizontally the three cut circular shapes, and then the
final two sets of three circular shapes must mirror vertically the first two sets of three
circular shapes. This eliminates answer choices 1, 3, 4, and 5.
We are left with the 2nd choice, which is the correct answer.

170. Explanation
The correct answer is 2.

The square paper was first folded in half diagonally, and then folded
again from the upper-right corner towards the lower-right corner.
Then, two holes were cut out of the folded paper.
Both holes go through four layers of paper, since the paper is folded twice where
they are cut.
Therefore, the answer should have eight holes (2 holes x 4 layers).
The holes will flip over the fold each time the paper is unfolded – first upwards and
then diagonally.

Answer choice 5 includes only four holes, so it can be eliminated.


Answer choice 4 doesn’t have any holes on the right edge of the square, where they
appear in the image above.
Answer choices 1 and 3 have holes in the wrong places, as they do not mirror the first
holes upwards and diagonally.

72
We are left with the 2nd answer choice, which is the correct answer.

171.Explanation
The correct answer is 5.

First the paper was folded lengthwise from the right edge of the square towards the
middle.
Then the outer corners were folded towards the center of the square.
Then 3 holes were cut out.
The folds do not overlap, so the paper was folded only once in each section.
Therefore, the holes go through two layers of paper, and the answer should
include six holes: 3 holes x 2 layers = 6.

Answer choices 2 and 4 have fewer than six holes, so they can be eliminated.
Answer choice 1 has more than six holes, so it can be eliminated as well.
Answer choice 3 has the right number of holes, but they are in the wrong places –
notice that the hole on the left side of the square is in line with the other holes. Since
the fold in that corner is diagonal, the hole should be higher than the other holes .

We are left with the 5th choice, which is the correct answer.

172. Explanation

The correct answer is (D).​


The paper has 2 holes punched and is folded three times.
When we open the first crease (from left to right), which only covers 1 hole, it results
in the paper having 3 holes in the next picture.
When we open the second crease, we expect the 3 holes to have doubled (there would
be 6 holes in the next picture), but because the top-left hole is over a blank space,
there are only 5 holes in the next picture.
If we open the third crease, we can see it covers only 1 hole, which results in the final
picture having 6 holes.
All the other answers can be eliminated because they do not have 6 holes.

73
173.Explanation

The correct answer is (A).


The paper has 2 holes punched and is folded three times.
If we open the first crease, which covers 1 hole, it results in the next picture having 3
holes.
If we open the second crease, which covers 1 hole, it results in the next picture having
4 holes.
If we open the third crease, which covers 1 hole, it results in the final picture having 5
holes.​
Answers C and E are incorrect because they do not have 5 holes.
Answers B and D are incorrect because when we open the second crease there should
be a hole punched in the lower left corner of the paper, which n1 of those choices
have.

74
2. Figure Recognition
1. Explanation

The correct answer is (E).

It is important to remember the answer choice must contain the given shape, while
keeping its original proportions. The given shape includes two vertices pointing
upwards-right:

Answers (A) and (D) are incorrect because they do not contain the given shape. Both
these answers include similar, but not identical shapes.
Answer (B) is incorrect because the angle formed by the vertex at the top is not the
same as the one in the given figure:

Answer (C) contains the given figure but rotated 90 degrees. Therefore, it is also
incorrect.
The remaining answer (E) is the correct answer.

2. Explanation

The correct answer is (A).


As you can see in the picture below, the given shape contains two parallel horizontal
lines and 2 vertices pointing to the left as well as a vertical line:

75
Try to find a characteristic in the given shape that is easy to spot. In this case, the
90-degree angle at the bottom-left corner. Almost immediately after the angle is
formed there is a diagonal line to look for.
Answer choice (A) includes several horizontal lines, and those in the middle of the
shape are at the same distance as those in the given shape. Those lines also connect to
two vertices which point to the left, helping you notice the given shape. Therefore,
this is the correct answer.
Answers (B), (D), and (E) can be ruled out for not having both the properties (in the
case of answer (D), there are no 90-degree angles at all).
Answer choice (C) includes a shape which resembles the given shape, but it has
different proportions, and therefore it is also incorrect.

3. Explanation

The correct answer is (B).


The given shape is a parallelogram (which is characterized by two pairs of congruent
parallel sides). Notice one pair of its sides is vertical while the other pair is diagonal.

Answer choice (B) contains only one pair of vertical lines. Focusing on those lines, it
is possible to note that they intersect with a pair of diagonal lines to form the given
parallelogram, making answer choice (B) the correct answer.
Answer choice (A) includes two vertical lines which form a rectangle, rather than a
parallelogram. Therefore, it does not contain the given shape and it is incorrect.
Answer choices (C) and (E) do not include a pair of vertical lines, thus they cannot
contain the given parallelogram and they are also incorrect.
Answer choice (D) contains a pair of parallel vertical lines; however they are too far
apart to form the given shape, making this choice also incorrect.

4. Explanation

76
The correct answer is (E).
The given shape is a right-angled triangle comprised of a vertical, horizontal and a
diagonal line stretching from top to bottom-right. You should look for the 90-degree
angle to rule out answers more easily.

Answer choice (E) includes a square whose diagonal lines intersect and create
right-angled triangles. One of those triangles has the same orientation and size as the
given triangle, making answer choice (E) the correct answer.
Answers (A), (C), and (D) are incorrect because even though there is a 90-degree
angle in the same direction as the given triangle, there are no diagonal sides to
complete the triangle.
Answer (B) is incorrect because it has no 90-degree angle.

5. Explanation

The correct answer is (C).


The shape given is comprised of straight vertical and horizontal lines. Therefore, it is
possible to deduce that the diagonal lines are meant to draw attention away from the
given shape.

Answer choice (C) depicts a grid and a triangle, although the triangle, which is
comprised of diagonal lines, is only meant as a distraction. The grid itself is
comprised of vertical and horizontal lines which form the given shape, making answer
choice (C) the correct answer.
Answer choices (A), (B), (D) and (E) are missing line segments to form the correct
shape, therefore they are incorrect.
Note: Answer choice (B) includes a shape similar to the one given, but with different
proportions, as shown below.

77
6. Explanation

The correct answer is (E).


The given shape contains diagonal lines and two parallel horizontal lines.
Answer (A) is missing a line segment to form the top vertex in order to form the given
shape, and therefore it can be ruled out.

Answer (B) includes a pair of horizontal lines but is missing a vertex on the right in
order to form the correct shape, making it also incorrect.

Answer (C) has two parallel horizontal lines, however they are too far apart to form
the given shape, thus it is eliminated.
Answer (D) includes a shape similar to the given shape, however its left side has a
different angle than the left side of the given shape (As shown in the picture below).
In addition, the vertex on the right has a different angle than the one in the given
shape. Therefore, it does not include the given shape and it is also incorrect.

78
7. Explanation

The correct answer is (D).


The given shape is a parallelogram with two vertical parallel sides. Notice that the
vertical parallel lines in answers (A), (B) and (C) are too far from one another to form
the given shape, and thus you can eliminate them.
Answer (E), on the other hand, includes several parallel vertical lines; however, none
of them are intersected by a pair of diagonal parallel lines as in the given
parallelogram. Therefore, it is also incorrect.
The remaining answer (D) is the correct answer. You can notice the parallelogram in
the figures top-left side by observing the vertical parallel lines intersecting with the
parallel diagonals.

8. Explanation

The correct answer is (A).


The given shape is an equilateral triangle which have vertices pointing up, down and
right. Another prominent feature of this triangle is its left side which is a vertical line.
Answer (A) includes two vertical lines which can be served as the left side of the
triangle, so in order to find the given triangle you can look for a vertex pointing to the
right to the right side of that line. When observing the left vertical line, you can notice
such vertex does exist and find the given shape. Therefore, this is the correct answer.
Answer (B) has an incomplete vertical on its left side and no other vertical lines,
therefore it cannot form the given shape and it is incorrect.
Answer (C) includes a vertical line which has a vertex to its right which also point in
the right direction, however it has different proportions than the given shape (as you
can see in the picture below). Therefore it does not contain the given shape and it is
incorrect.

79
Answer (D) includes a vertical line and to its right many vertices pointing to the right.
However, the triangles formed using that line are too big to form the given triangle,
and therefore it is incorrect.
Answer (E) has a prominent vertex pointing right, however it is not part of a triangle.
It does not contain the given shape and therefore it is incorrect.

9. Explanation

The correct answer is (A).


The given triangle has vertices pointing up, right and down-left. It also has a side
which is a horizontal line.
Answer (A) has only one horizontal line. In order to form the shape, it must intersect
with two different diagonal lines and form the correct angles. Once focused on that
line it is possible to notice it creates a vertex pointing to the right and forms the given
shape. Therefore, this is the correct answer.
Answer (B) includes two horizontal lines as the outer lines of the figure. The bottom
line cannot form the given triangle, as the horizontal line of the triangle is on the top
of the shape. The top line of answer (B), forms a shape similar to the given triangle;
however, it is bigger. Since the proportions must be kept, it does not contain the given
shape and it is incorrect.
Answer (C) contains two horizontal lines. In addition, those line are intersected by a
diagonal going from bottom-left to top-right with the same angle as the bottom side of
the given triangle. However, the horizontal lines are not intersected by a line with the
same angle as the left side of the given triangle. Therefore, it does not contain the
given shape and it is also incorrect.

Answer (D) includes two horizontal lines as the outer lines of the figure. The bottom
line cannot form the given shape, as the horizontal line of the shape is on the top of

80
the shape. The top line is missing a segment in order to form the given shape, and
therefore it also does not contain the shape and it is incorrect.
You can also notice a similar, but larger, triangle appears in this answer choice.

Answer (E) has three horizontal lines. Just like answers (B) and (D), the one in the
bottom cannot be part of the given triangle. The other two horizontal lines are missing
a side to create the given shape.

10. Explanation

The correct answer is (B).


The strong features of the given shape are an isosceles triangle pointing right (without
its base) and a pair of parallel horizontal lines intersected with a perpendicular vertical
line:

Answer (B) uses symmetry to hide the given shape. The rectangle which appears in
the picture has two parallel horizontal lines as necessary, and the rotated square
instead has a vertex pointing to the right. Once you notice this, you can follow the
lines to form the given shape. Therefore, this is the correct answer. Notice that the left
side of the rectangle is the perpendicular vertical line.
Answer (A) includes a shape similar to the given shape, however the triangle shown
in the picture has different angles than those of the given shape, making answer (A)
incorrect.

Answer (C) has a pair of parallel horizontal lines which intersect with a vertical line.
However, the left side is not connected to an isosceles triangle pointing to the right, as
needed. It does not contain the given shape and therefore it is incorrect.

81
Answer (D) includes only one isosceles triangle and it is too big to form the given
shape. Moreover, the upper part of the triangle is not connected with a horizontal line,
as needed, making answer (D) incorrect.
Answer (E) has no vertical lines at all, so it cannot form the given shape. Therefore, it
is also incorrect.

11. Explanation

The correct answer is (D).

The shape given is comprised of three horizontal parallel lines and three diagonal
lines, two of which are parallel, as can be seen in the picture below.

Answer choice (D) includes two prominent diagonal parallel lines which incline with
the same angle as the given shape, so this would be the best place to look for the
given shape. Once you examine those lines you can see the three horizontal parallel
lines intersecting them, forming the given shape. Therefore, this is the correct answer.

Answer choice (A) includes two parallel diagonal lines which incline with the same
angle as the given shape, however they do not form the given shape; therefore, this
answer choice is incorrect.

12. Explanation

The correct answer is (C).

The given shape is a pentagon, which includes a vertex pointing upwards.

Answer choice (C) includes one vertex pointing upwards in the middle of the picture.
Further examination reveals it is the top part of the given pentagon.
The picture below shows that this image actually includes two pentagons combined.

82
Answer choice (A) includes a shape which resembles a pentagon, however one of its
sides has been detached and changed. It does not contain the given shape and
therefore it is incorrect.

Answer choice (B) and (D) also include shapes which resemble the given pentagon,
however those shapes include a side which is vertical instead of diagonal. Therefore,
they do not include the exact given shape, making them incorrect.

Answer choice (E) includes trapezoids and triangles instead of a pentagon, making it
also incorrect.

13. Explanation

The correct answer is (D).

The given shape is comprised of two horizontal parallel lines and diagonal lines. In
addition, it includes two vertices pointing to the right:

Answer choice (D) shows a parallelogram whose horizontal parallel lines are at the
same distance from one another as the given shape’s parallel lines. The diagonal sides
of that parallelogram form the same angles as the left side of the given shape (as
shown in the picture below).

83
However, only the left side of the parallelogram can be used as the left side of the
given shape (as the only other line with the same angle is the right side of the
parallelogram, and it has no lines to its right to form the given shape). Once you focus
on this line, you can notice the diagonal lines to its right form the given shape.

Answer choice (A) does include two vertices pointing to the right, however the
diagonal lines which form them have different angles than the lines of the given shape,
making it unproportionate.

Answer choice (B) includes one diagonal line which inclines with the same angle as
the left side of the given shape, however this line cannot form the given shape, as it
does not have enough lines to its right.

Answer choices (C) and (E) do not include diagonal lines which incline with the same
angle as the left side of the given shape, therefore they do not contain the given shape
and they are also incorrect.

The remaining answer choice (D) is the correct answer.

14. Explanation

The correct answer is (E).

The given shape has a bottom line which is horizontal and vertices which point up-left
and two that point to the right.

Answer (A) is incorrect, as it is not wide enough to contain the given shape.

84
Both answers (B) and (C) include a shape which resembles the given shape, however
the top vertex pointing right in both answers is made using a horizontal line rather
than a diagonal line as in the given shape.

Answer choice (D) is missing the bottom-left diagonal line to form the given shape
(as shown below), thus it is also incorrect.

The remaining answer choice (E) is the correct answer.

You could also solve this question without using answer choice elimination. Answer
(E) includes vertices pointing in the right directions, and most of its outer lines form
the given shape.

15. Explanation

The correct answer is (D).

You can picture the given shape as two rectangles, one vertical and the other
horizontal.

Notice the vertical rectangle of the given shape is located in the middle of the
horizontal rectangle. Therefore, our first hint would be to look for the vertical
rectangle within the inner lines of the pictures, rather than in the borders, since
forming the given shape using a vertical rectangle appearing on the borders would
cause part of it to be outside of the borders.

85
Therefore, the answer must appear in one of two different locations:

Answer (D) includes a pair of vertical and parallel inner lines. Those lines are
perpendicular to horizontal lines, and focusing on them reveals that together, they
form the given shape. Therefore, answer (D) is the correct answer.

Answer (A) includes parallel vertical inner lines, as necessary. However, it is missing
a line segment to form the given shape:

Answer (B) has two vertical parallel inner lines. However, those are not in the middle
of a horizontal rectangle and therefore it does not form the given shape and it is
incorrect.

Answer (C) includes vertical inner lines, however only one is long enough to be a side
for a vertical rectangle. Therefore, it does not contain the given shape and it is
incorrect.

Answer (E), like answer (A), is missing a line segment to form the given shape.

16. Explanation

The correct answer is (E).

The given shape is an isosceles trapezoid with a pair of vertical parallel lines and a
pair of congruent diagonal sides.

86
Answer (E) includes three vertical parallel lines (as shown in the picture below). The
leftmost line is too far from the middle line to form the given shape. However, the
middle line and the rightmost line are close enough, and they are being intersected by
diagonals, as necessary, forming the given shape. Therefore, answer (E) is the correct
answer.

Answer (A) includes several vertical lines, so using them to find the trapezoid might
get tricky. Instead, you can notice the upper outer line of the figure is a diagonal
inclining with the same angle as the upper side of the trapezoid. In order to contain
the shape, the figure must have a diagonal line with the same angle as the bottom line
of the trapezoid close enough to the upper line to form the shape. You can notice that
it only happens in the right side of the picture (as shown in the picture below).
However, it is still missing a line segment to form the given shape, and therefore
answer (A) is incorrect.

Answer (B) does not include a diagonal line with the same angle as the bottom line of
the trapezoid. The figure has only one line stretching from bottom-left to top-right and
its angle is different. Therefore, it is also incorrect.

Answer (C) has only two vertical parallel lines and they are too far apart to form the
given shape, making it also incorrect.

Answer (D) contains one isosceles trapezoid, but it is bigger than the given shape and
therefore it is also incorrect:

17. Explanation

The correct answer is (B).

The given shape is half a star. It has vertices pointing up, right and down-right. In
addition, it has a vertical line.

Answer (B) includes one vertical line and attached to it are lines forming vertices
pointing towards the right. It contains the given shape, making it the correct answer.

87
Answer (A) has no vertical lines and therefore it cannot contain the given shape,
making it incorrect.

Answer (C) includes a shape which resembles the half-star given. However, the angle
of the vertex pointing bottom-right is different. Moreover, the lengths of the sides
forming the star are different.

Answer (D) has no horizontal lines, and therefore cannot form the given shape (as it
must include a horizontal line), and therefore it is incorrect.
Answer (E) includes a vertical line; however, it is too short to form the given shape.
Therefore, it is also incorrect.

18. Explanation

The correct answer is (C).


The shape given resembles a right-angled triangle on top of a rectangle, as seen in the
picture below.

It is characterized by two pairs of parallel lines (two vertical and two horizontal) and a
vertex pointing upwards.

Answer (C) has three horizontal parallel lines (as shown below). The line at the top is
too far away from the line underneath, so it cannot be used to form the rectangle part
of the given shape. However, the line in the middle and the line underneath it has the
correct distance. Moreover, there is a vertex pointing up right above the middle line,
completing the given shape. Therefore, this is the correct answer.

88
Answer (A) also includes three horizontal parallel lines. And much like answer (C),
the top line is too far from the line underneath it to form the given shape, so you
should try and find the given shape using the middle line and the line in the bottom.
However, there is no right-angled triangle above those lines and therefore they cannot
form the given shape, making answer (A) incorrect.

In answer (B) you can use the vertical parallel lines. There are three of those (as
shown below) and they may be used as sides to form the given shape.

However, those lines either form a square, or a rectangle, which is bigger than the one
in the given shape, and therefore answer (B) is also incorrect.

Answer (D) has three vertical lines. Those lines are either too close to each other or
too far from one another to be the sides of the rectangle in the given shape. Therefore,
answer (D) does not contain the given shape and it is incorrect.

Answer (E) includes several horizontal parallel lines. However, none of those lines
have on top of it a right-angled triangle besides one (as shown below).

The line that does, creates the given shape, but in a mirror image (meaning it is
flipped vertically):
As the given shape must keep its original orientation, it does not appear in answer (E)
and answer (E) is incorrect.

89
19. Explanation

The correct answer is (D).

The hourglass given is comprised of two isosceles triangles pointing each other.
The bottom triangle in answer (A) is not isosceles, therefore it cannot form the given
shape and it is incorrect.
Answer (B) has both a bottom isosceles triangle and a top isosceles triangle, however,
each of them faces a non-isosceles triangle and therefore it also does not form the
given shape and it is incorrect.
Answer (C) is missing a line segment to form the given shape, as the base of the top
isosceles triangle is missing.

Answer (E), like answer (C), is also missing a line segment to form the given shape,
as the base of the bottom isosceles triangle is missing.

The remaining answer (D) is the correct answer. You can notice that the diagonals on
the right side of the image form two isosceles triangles facing each other and that the
outer rectangle serves as their base, completing the hourglass. Therefore, this is the
correct answer.

20. Explanation

The correct answer is (C).

90
The given shape is an arrowhead pointing left. It is comprised of a pair of parallel
vertical lines and two vertices pointing left next to each other.

Answer (C) includes three parallel horizontal lines. The line that goes through the
middle of the trapezoid can be overlooked, as it is too close to the other horizontal
lines in order to form the given shape. In addition, this figure includes diagonals
which incline with angles similar to those of the given shape, and some of them create
vertices pointing left. Due to the given shape’s height, compared to the height of the
figure, you can deduce the vertex you are looking for should be in the middle of the
figure. There are three vertices that match this criterion.

The rightmost and middle vertices form a shape which is similar to the given shape,
but are missing line segments. However, the vertex on the left is part of the given
shape and therefore this is the correct answer.
Answer (A) includes a shape which resembles the given shape, however it is missing
a line segment in order to complete it:

Answer (B) includes two vertices pointing left next to each other, however one of the
diagonals which creates them has a different angle than the diagonal of the given
shape:

Answer (D) has several vertices pointing left:


However, the lines forming them have different angles than those of the given shape,
therefore it is also incorrect.

91
Answer (E) includes only one vertex pointing left in a similar way to the given shape,
therefore it also cannot form the given shape and therefore, this answer choice is
incorrect.

21. Explanation

The correct answer is (B).

The shape given is a plus (+). It is possible viewing it as two rectangles – one is
vertical and the other horizontal:

Since all answer choices given are surrounded by a square and the proportions of the
shape needed to be found must be kept, we can conclude there must be two vertical
and two horizontal lines going through the middle of the square stretching from one
side to the other. Answer choice (B) is the only one containing those lines and thus, it
is correct.

Solving tip: If you are unable to find the correct answer, answer choice elimination
could be a powerful tool (as shown in this question). The logic involves focusing on
the characteristics of the given shape (such as proportions, vertices, size and more) to
eliminate any answer choice that does not share those characteristics.

Answer choice (A) is incorrect as it does not contain any vertical lines going through
the middle of the square, so it cannot contain the plus.

Answer choice (C) contains only one horizontal inner line going through the middle,
thus it cannot contain the plus and it is incorrect.

Although both answer choices (D) and (E) have the correct amount of inner horizontal
and vertical lines, each contains a vertical line which is too short to be the side of a
plus, and so they are also incorrect.

The only answer choice left is (B), which is the correct answer.

92
22. Explanation

The correct answer is (E).

Start by analysing the shape you need to identify. Once you know its properties, you
can use these to find the correct answer or rule out other answer choices.

The shape is symmetric with respect to a horizontal line. Furthermore, the shape given
has three pairs of parallel lines (one pair of lines that are horizontal, and the last two
pairs can be found above and below the symmetry line) and the lines in these pairs
have the same length. Finally, the intersecting edges (sides) in the middle of the shape
are directed to the right. Another way to see this is by thinking of the middle vertices
as changing the direction of the lines.

Now that you know the characteristics to look for, examine the answer choices:

Answer (E) is correct because you can see the small shape included here:

Answer (A) is incorrect because it does not include the smaller shape. You can see
this by noticing that no pair of lines that are parallel have the same length, or because
there is no vertex that changes the direction of the side lines as in the original shape.

Answer (B) is incorrect because even though it has many pairs of parallel lines, the
shape is not included inside it. You can see this by focusing on the vertex mentioned
before, which changes the direction of the lines (the intersection of the diagonals) but
the original shape has two such vertices, and answer (B) only has one. Answer (B) is
also incorrect because it has no inner parallel lines .

Answer (C) is incorrect because the inside lines intersect at different heights, meaning
it is asymmetric, as opposed to the original shape.

Answer (D) is the most interesting answer among the incorrect ones. This is because
it has parallel sides of equal length and two vertices that change the direction of lines.

93
However, the lines that change the direction are not parallel to one another as in the
original shape. Therefore, this answer is incorrect.

Solving Tip: Remember that symmetry can always be a helpful tool to find the
correct answer or rule out others. Another tool that will help you is finding parallel
lines and compare them to others to see if they share the same qualities as the ones in
the given shape.

23. Explanation

The correct answer is (A).

The shape given is a right-angled triangle. The pictures contain many shapes and lines
that are meant to draw the attention away from the shape. A good way of finding the
shape would be to look at its characteristics and choose the one which will be easiest
to find in the pictures. In this question, the triangle presented contains a horizontal
line as one of its sides and you can notice that there are few horizontal lines in the
given pictures.

Answer choice (A) includes three horizontal lines, one goes through its middle and
the others are the sides of the surrounding square. A further examination of the left
side of the square shows there is a perpendicular vertical line crossing it that could be
used as another side of the triangle. The diagonal line that connects them completes
the triangle, forming the given shape. Therefore, this is the correct answer.

Answer choice (B) contains one horizontal line going through its middle and one line
that is perpendicular to it. However, the diagonal line that completes them to form a
triangle creates a smaller triangle than the one given. Therefore, it does not contain
the given shape and it is incorrect.

Answer choice (C) includes one horizontal line which serves as a side of a
right-angled triangle, but that triangle is pointing downwards instead of upwards.
Thus, it also does not contain the given shape and it is also incorrect.

Answer choice (D) contains perpendicular lines that can serve as two sides of a
triangle. However, all the triangles that use those lines are in the wrong direction.
Therefore, it does not contain the given shape and it is incorrect.

Similarly, answer choice (E) also contains perpendicular lines, however they serve as
sides for a taller triangle than the given triangle. Therefore, it also does not contain the
given shape and it is incorrect.

94
24. Explanation

The correct answer is (C).

The shape given is a leaf. It has a vertex pointing upwards (Vertex 1 in the picture
below) and another vertex pointing to the left (Vertex 2 in the picture below):

The pictures show different flowers to draw attention away from the given shape. The
correct answer must contain the given shape in the same size and direction.

Answer choice (C) includes few vertices pointing upwards. The right most leaf of the
flower also contains a vertex pointing to the left and a closer look shows the given
shape (with a curved line drawn on it). Therefore, it is the correct answer.

Answer choices (A) and (E) have no vertices pointing upwards; therefore, they do not
contain the given shape and are incorrect.

Answer choice (B) does not include vertices pointing to the left and therefore it does
not contain the given shape. Thus, it is incorrect.

Although answer choice (D) does contain a vertex pointing to the left, the shape it
forms is different from the one given, thus making it also incorrect.

The correct answer is (C), which is the only one that remains.

25. Explanation

The correct answer is (D).

95
The given shape is rather complicated. Its prominent characteristics are parallel
straight lines (horizontal and vertical), and right-angled edges. Therefore, diagonal
lines in the given pictures are only meant to draw attention away from the given shape.
In addition, all of the given pictures share the same height as the given shape, and the
proportions of the given shape must be kept, therefore the horizontal lines’ height in
the pictures should also be positioned at the same height as shown in the given shape.

Answer choice (D) includes horizontal lines positioned at the same height as shown in
the given picture. Ignoring the diagonal line, the sides of the square create the right
angles and form the given shape. Therefore, this is the correct answer.

Answer choice (A) also includes horizontal lines in the correct place. However, it is
missing a vertical line near the left side of the rectangle in order to form the given
shape. Thus, it does not contain the given shape and it is incorrect.

Answer choice (B) includes two inner horizontal lines, one of which is too short for
the given shape; therefore, it does not contain the given shape, making it incorrect.

Answer choice (C) does contain horizontal lines as needed, but the right most inner
vertical line is too short to form the given shape and therefore it is also incorrect.

Answer choice (E) also includes horizontal lines in the correct place, however it
contains only one vertical line in the left side of the rectangle, thus it cannot form the
given shape, making it incorrect.

26. Explanation

The correct answer is (C).

The quadrilateral given includes two right angles in its top side and a diagonal line
stretching from its bottom vertex to the right. A good way to find the given shape in
the pictures would be to find a diagonal line which inclines with the same angle as in
the given shape:

Answer choice (C) contains one diagonal line that stretches with a similar angle.
Focusing on the left rectangle, the diagonal stretches from the bottom vertex into its
right side and forms the given shape. Thus, this is the correct answer.

96
Notice that the left rectangle forms a similar shape, however it is smaller in size than
the given shape.

The diagonal lines in answer choice (A) that stretch from the bottom to the right,
incline at a different angle than the diagonal in the given shape, therefore they cannot
form the given shape, making this answer choice incorrect.

Although answer choice (B) contains a diagonal line which inclines with the correct
angle, it is missing a line segment to form the given quadrilateral. Thus, it does not
contain the given shape and it is incorrect.

Answer choice (D) contains a diagonal line which stretches in the right direction,
however it leans in a different angle, thus it cannot form the given shape. Therefore, it
is also incorrect.

Answer choice (E) contains only two vertical lines, however they are too far apart to
form the given shape, therefore it is also incorrect.

The remaining answer choice (C) is the correct answer.

27. Explanation

The correct answer is (C).

The given shape is comprised of straight lines and contains a vertex pointing to the
right. Another prominent feature is that two of its sides are parallel vertical lines.

Answer (C) is correct because the given shape is included. You can see this more
quickly by noticing the left half of the shape. The triangle that is formed inside the
shape is meant to distract and conceal the given shape.

Answer (A) is incorrect because the angles of the side lines (the lines to the right of
the shape) are different from the given shape.

Answer (B) contains a similar shape to the one given, but the angle formed in the
rightmost vertex is not the same as the angle in the given shape (furthermore, even if
the angle were the same angle, the dimensions would be bigger than the given shape)

97
Answer choices (D) and (E) include two parallel vertical lines, however they are
closer to each other or further from each other, respectively, than shown in the given
picture. Since the proportions of the given shape need to be kept, they cannot contain
the given shape and they are incorrect as well.

28. Explanation

The correct answer is (D).

The shape given includes several parallel straight lines which are either horizontal or
vertical. Thus, diagonal lines drawn in the pictures are only meant to distract you from
the correct answer. In addition, the straight lines create two distinctive right angles, as
shown below:

Answer choice (D) contains many diagonal lines; therefore, it is important to focus on
the straight lines. Some of the straight lines inside the square in the picture are
perpendicular, thus creating right angles as necessary. Examining those corners
reveals the given shape. Therefore, this is the correct answer.

Answer choice (A) is missing a line segment in order to form the given shape,
therefore it is incorrect.

If this shape is inside one of the options above, that option must have its upper half
divided into two by a vertical line. Answers (B), (C), and (E) are incorrect because
there is no such vertical line.

98
29. Explanation

The correct answer is (B).

The given shape is a right-angled triangle pointing upwards to the right. Look for this
angle in the options above. As soon as you recognize the angle, check to see if you
can find the given shape based on that.

Answer choice (B) includes two right angles pointing in a direction similar to the
given shape, as shown in the picture below. Notice that angle #2 (as marked in the
picture below) is actually a part of the given shape. Therefore, this is the correct
answer:

Answer choice (A) includes a few right-angles vertices, but none of them form a
triangle pointing in the right direction. Therefore, it does not include the given shape
and it is incorrect.

Answers (C) and (E) are incorrect because they do not contain a 90-degree angle.

You can also rule out answer (D) because even though it contains 90-degree angles,
no triangle is formed using those angles.

30. Explanation

The correct answer is (B).

The given shape is a plus (‘+’), comprised of vertical and horizontal lines, therefore it
is possible to deduce that the diagonal lines are only meant to draw attention away
from the correct answer.

Answer choice (B) contains an isosceles triangle which is used to cover the given
shape, which appears in the top left side of the picture, making this the correct answer
choice.

99
Answer choices (A), (C), (D) and (E) include several vertical and horizontal lines,
however they either lack line segments to form the given plus or have different
proportions than those of the given plus. Therefore, they are incorrect.

31. Explanation

The correct answer is (A).

The given shape has few prominent features; a vertical line, two vertices -- one of
which points to the right while the other points to the left -- and four parallel
horizontal lines. Using those features you should try and locate the given shape.

Answer choice (A) includes vertices pointing to the left and the right. Following those
vertices, it is possible to see they connect using horizontal lines to form the given
shape, making answer choice (A) the correct answer.

Answer choice (B) also contains vertices pointing in the right directions, however
they are too close to one another to form the given shape. Is answer choice does not
contain the given shape and therefore it is incorrect.

The only straight lines answer choice (C) includes are the boundaries of the picture,
meaning they cannot form the angles which appear in the given shape. It does not
contain the given shape and therefore it is also incorrect.

Answer choice (D), like answer choice (B), contains vertices pointing in the right
directions, however here those vertices are too far apart to form the given shape,
making answer choice (D) incorrect.

Answer choice (E) is also incorrect, as it does not contain a vertex pointing to the
right, which means it cannot contain the given shape.

32. Explanation

The correct answer is (A).

100
The given shape is an isosceles trapezoid comprised of parallel sides, which are
diagonal, a vertical line and a horizontal line. A quick way to rule out answers would
be to look for diagonals which incline with the same angle and have the same
distance.

Answer (A) contains diagonals which incline with an angle similar to the trapezoid
parallel sides. In addition, they intersect with the outer lines of the square (which is
comprised of vertical and horizontal lines), thus forming the given trapezoid.

Answer (B) includes a pair of diagonal parallel lines, however they are too close to
one another to form the given trapezoid, thus it is incorrect.

Answer (C) includes trapezoids, however they are rotated 180 degrees and also have
different sizes than the one given.

Answers (D) and (E) also include a pair of diagonal parallel lines which incline with
the same degree shown in the given trapezoid. However, those lines are too far from
one another to form the given shape.

33. Explanation

The correct answer is (C).

In order to find the given shape, you should focus on its prominent features: a pair of
parallel diagonal lines, which also intersects with a line forming 90-degree angles and
a vertex pointing down-left:

Answer (C) includes one vertex pointing in the same direction as in the given shape.
In addition, above that vertex there are two diagonal lines inclining with an angle
similar to the given shape. Connecting these diagonal lines reveals the given shape.
Therefore, this is the correct answer.

Answer (A) has two diagonals which incline with the same angle as in the given
shape, forming a shape similar to it. However, those lines are longer than the diagonal

101
lines of the given shape, making it longer. Therefore, it does not contain the given
shape and it is incorrect.

Answer (B) includes three diagonal lines which incline the same angle as the lines in
the given shape (as shown in the picture below). However, the bottom one is too short
to be part of the wanted shape and the other two are too far apart from one another.
Moreover, the angle of the triangle pointing down is different than the one in the
given shape. Thus, answer (B) is also incorrect.

Answer (D) has many lines, making it hard to distinguish different shapes. You can
look for the given shape in two ways:
1. Diagonals – Answer (D) includes two diagonals which incline with the same angle
as the diagonals of the given shape. Under those diagonals there is a vertex pointing
down-left as necessary. Together they indicate a shape which resembles the given
shape. However, a closer look shows it is shorter than the given shape and therefore
answer (D) is incorrect.
2. Vertex – You could also first look for vertices which point in the right direction
(bottom-left). Once you find the vertex, you look for the diagonals and come to the
same conclusion.

Answer (E) includes one vertex pointing left-down. However, in the given shape, the
angle pointing in the same direction is created by two incongruent lines while in
answer (E) it is formed by two congruent lines. Thus, it does not form the given shape
and answer (E) is incorrect.

34. Explanation

102
The correct answer is (D).

The shape presented includes two parallel edges (sides) that look similar to a
parallelogram. That parallelogram was joined with a triangle on its side:

Notice that one set of the parallel side is horizontal while the other is inclined to the
right.

Answer choice (A) has no parallel lines that are inclined and therefore cannot contain
the given parallelogram, and so this answer is incorrect.

Answer choice (B) has parallel lines that incline similarly to the given shape, however
the triangles in the picture are not joined with them, thus it cannot contain the shape,
making this choice incorrect.

Answer choice (C) contains triangles that are different than the one in the given shape
and therefore it also cannot contain the shape and is incorrect.

Answer choice (E) has parallel lines; however, they do not incline with the same
angle at which the given shape inclines, so they cannot be used to create a correct
parallelogram. Therefore, it cannot contain the given shape, making it also incorrect.

35. Explanation

The correct answer is (B).

The given shape is an arrow head pointing up-right. You should look for it in the
pictures above, using the angles which its lines form and the direction the arrow is
pointing towards.

103
Answer (B) includes two vertices pointing in a direction similar to the given arrow
head and focusing on them reveals they actually form it, making this the correct
answer:

Answer (A) also includes a few vertices pointing in a direction similar to the given
arrow head, however, the shape forming those angles is different than the given arrow
head. Therefore, it is incorrect.

Answer (C) includes a shape which resembles the given arrow head, with vertices
pointing in the right direction, however it is formed using six edges instead of four,
thus it cannot be the given shape and it is incorrect.

Answer (D) contains two triangles pointing in a direction similar to the given shape,
however putting them one on top of the other in the way shown does not form the
given shape. Therefore, answer (D) is also incorrect.

Answer (E) also includes an arrow head, however the bottom lines comprising it
intersect in a different place, creating a different shape:

36. Explanation

The correct answer is (E).

104
You should focus on the shape’s outline and find a prominent line or a characteristic
that will help in eliminating answer choices. The shape given can be viewed as
rectangle and a triangle with one line missing from the base of the triangle:

Answer choices (B), (C) and (D) are surrounded by squares. In order to fit into the
squares, the vertical line (side) of the triangle must appear in the top-middle part of
the squares. You can immediately eliminate answer choice (A) which does not
contain a line going through its middle.

Answer choice (B) does not contain any diagonal lines, hence it is also incorrect.

Although answer choice (C) does include a line going through the middle of the
square, it does not stretch to the top part of the picture as necessary and therefore it is
also incorrect.

Answer choice (D) does contain a diagonal line, however it is inclined to the right,
opposing the given shape (in which the triangle diagonal is inclined to the left), and so
it is also incorrect.

The only answer choice remaining is answer choice (E), which is the correct answer

37. Explanation

The correct answer is (A).

The shape given is an equilateral triangle with a vertex pointing upwards. A quick
look in the answer choices shows Answer choice (B) does not contain a vertex
pointing upwards, so it cannot contain the given triangle and thus it is eliminated.

Answer choice (C) includes several triangles, however, none of them resembles the
equilateral triangle given and so it is also eliminated.

105
The triangle in answer choice (D) is pointing downwards (instead of upwards), and
therefore it is also incorrect.

Answer choice (E) contains a triangle, but it is smaller than the triangle we need to
find; therefore, this answer is also incorrect.

Answer choice (A) remains the only possible option and therefore it is correct.

38. Explanation

The correct answer is (B).

The shape presented is a rhombus. It is characterized by two pairs of parallel sides and
in this question, it has two vertices pointing upwards and downwards and two vertices
pointing towards the sides. Another way of picturing a rhombus is as two identical
triangles with a joined base line:

Answer choice (A) contains several parallel lines, however it does not contain any
vertices pointed upwards or downwards and therefore cannot be the correct answer.

Answer choice (C) contains one triangle pointing left, but its base is joined with a
trapezoid rather than another triangle, and so it is also incorrect.

Answer choice (D) has no triangles and therefore cannot be the correct answer.

Answer choice (E) has one side missing from the rhombus and so is also incorrect.

The answer remaining – answer choice (B) is the correct answer.

39. Explanation

The correct answer is (B).

106
First, notice the given shape has no diagonal lines. You can deduct that the diagonal
lines in the answer choices are either there to distract, or to cover the correct shape.
The shape given can be seen as two joined rectangles, one of which is vertical and the
other horizontal.

Answer choice (A) contains a vertical rectangle similar to the given shape, however it
has no horizontal rectangle that can complete the shape, thus it is incorrect.

Answer choices (C), (D) and (E) contain a horizontal rectangle in the top part rather
than the bottom, so can also be eliminated.

The only remaining answer choice (B) is the correct answer.

40. Explanation

The correct answer is C.

The answer choices in this question include many different details meant to confuse
and hide the shape. Focusing on specific characteristics of the shape will help a great
deal in not being thrown off by the number of lines and shapes involved.

Look at the picture below:

The four marked vertices can be helpful in finding the shape. Vertices 1 and 3 point to
the left and down, respectively. Thus, the correct answer choice must contain vertices
pointing the same way. Moreover, notice that vertices 1 and 3 are connected with a
diagonal which has a missing part. In addition, vertices 2 and 4 are facing the same
direction (up-right), thus creating some sort of “parallel corners”.

Answer choice (A) contains many parallel lines, however the “parallel corners” it
contains are too small to form the shape given, therefore it is incorrect.

107
Answer choice (B) contains one diagonal, however that diagonal is not connected to a
“parallel corner” facing the correct direction (up-right) and so is incorrect.

Answer choice (D) contains only one “parallel corner” which faces the correct
direction, however the diagonals are not passing through the corner to create the
correct shape, therefore it is also incorrect.

Answer choice (E) has no “parallel corners” facing the right direction, thus also
incorrect.

The only remaining answer choice (C) is the correct answer.

41. Explanation

The correct answer is (E).

The shape given contains an arc and two perpendicular lines, one vertical and the
other horizontal:

Although answer choice (A) contains perpendicular lines, they are diagonal which
means they cannot form the given shape, thus it is eliminated.

Answer choice (B) contains one horizontal line, which is the base of the triangle. This
line is too short to form the shape given, thus answer choice (B) is also eliminated.

Answer choice (C) contains no vertical lines, therefore it cannot form the given shape
and it is eliminated as well. Notice it also contains the shape given rotated 90 degrees
clockwise as a distractor.

Answer choice (D) does contain perpendicular lines, however they are rotated 180
degrees. Therefore, it cannot contain the given shape and it is incorrect.

108
The remaining answer choice (E) contains the shape and it is the correct answer.

42. Explanation

The correct answer is (D).

The shape given is comprised of a rectangle on top of which is attached an isosceles


triangle. The triangle’s vertex is located in the middle of the shape and so it should
appear in the answer choices in the same position. Moreover, the horizontal lines in
the rectangle stretch from one side of the picture to the other, excluding the base of
the triangle, so it should act the same way in the answer choices.

Answer choices (B), (C) and (D) are the only one with vertices in the correct place.
Notice answer choice (A) includes two triangles on the top part. However, neither is
in the correct position. Answer choice (E) includes a right-angled triangle instead of
an isosceles triangle. Therefore, both answer choices (A) and (E) cannot form the
given shape and they are incorrect.

Answer choice (B) is missing a horizontal line segment on the right, therefore it
cannot form the given shape and it is incorrect.

Answer choice (C) is comprised from a triangle on top of a trapezoid instead of a


rectangle and, therefore, it is also incorrect.

The remaining answer choice (D) includes both the vertex needed and the horizontal
lines needed to form the given shape, making it the correct answer.

43. Explanation

The correct answer is (A).

The shape given is amorphic. In order to find the given shape, it is best to use its
prominent characteristics and look for them in the answer choices. In this question,
the shape includes a vertex which seemingly “points” to the left. In addition, the lines
forming the given shape incline in a unique way that should be kept in the correct
answer choice.

109
Answer choice (A)’s left-most vertex, alongside the sides which create it, matches the
given shape’s proportions and, in fact, form the given shape. Therefore, this is the
correct answer.

Answer choices (B), (C), (D) and (E) do not contain the given shape. None of them
create the given shape with accurate proportions and therefore they are incorrect.

Answer choice (D), for example, shows a similar shape with different proportions as
seen below:

44. Explanation

The correct answer is (B).

The shape given is two equilateral triangles pointing towards each other. The pictures
given include many details that are meant to draw attention away from the given
shape, therefore it is important to search the shape characteristics thoroughly. This
shape is characterized by having two parallel vertical lines and two diagonal lines that
cross each other.

Answer choice (B) contains a prominent pair of diagonal lines crossing each other. A
closer look shows these diagonal lines are part of two equilateral triangles pointing
towards each other, forming the given shape. Therefore, it is the correct answer.

Answer choice (A)’s only vertical parallel lines are too far apart to keep the
proportions of the given shape. Thus, it does not contain the given shape and it is
eliminated.

Although answer choice (C) contains two parallel vertical lines, those lines do not
form two equilateral triangles. Therefore, it also does not contain the given shape and
it is eliminated.

110
Answer choice (D) contains only one vertical line, so it cannot contain the given
shape and therefore it is eliminated.

Answer choice (E) has two vertical parallel lines. However, they are too close to form
the given shape and are not a part of two equilateral triangles, therefore it also cannot
create the given shape and is eliminated.

We are left with answer choice (B) as our only option, which is the correct answer.

45. Explanation

The correct answer is (C).

The given shape is a crescent with a horizontal line going through its middle. The
pictures in this question are all aquatic animals drawn with complex lines aimed to
draw attention away from the correct shape.

Answer choice (C) contains only one horizontal line which is inside a crescent as
needed, therefore it is the correct answer.

Answer choices (A), (B) and (D) do not contain any horizontal line, thus they cannot
contain the given shape and are incorrect.

Answer choice (E) contains one horizontal line at the top of the fish’s fin, however it
does not form the given shape and therefore it is also incorrect.

46. Explanation

The correct answer is (B).

The given shape is a "C". It is comprised of two semi-ellipses, one inside the other,
which are joined with a vertical line.

111
Answer choice (B) includes two ellipses, one inside the other, connected with a
vertical line. Focusing on the left side of the picture and ignoring the diagonal lines
drawn on the shape, it is possible to see the “C”. Therefore, this is the correct answer.

Answer choice (A) contains a crescent rather than a “C”, therefore it is in correct.

Answer choice (C) contains a circle inside of an ellipse, rather than two ellipses (as
shown below). Therefore, it does not contain the given shape and it is incorrect.

Answer choice (E) includes only one ellipse, thus it cannot contain the given shape
and it is incorrect.

Answer choice (D) does contain two ellipses. However, they are not connected with
vertical lines and therefore cannot form the given shape. This answer is also incorrect.

Answer choice (B) remains, which is the correct answer.

47. Explanation

The correct answer is (D).

The shape given is “half a heart” which can also be described as a vertical line
attached with a curved line.

Answer choice (D) contains two vertical lines that can serve as part of the shape. The
right vertical line is, in fact, connected with a curved line, thus forming “half a heart”
as needed. Therefore, it is the correct answer.

Answer choice (A) contains several vertical lines. However, none of the shapes
attached to those lines create the correct angles to form “half a heart”.

Answer choice (B) includes a shape similar to “half a heart”. However, the curved
line is not curved for all its length (as shown in the picture). Therefore, it does not
contain the given shape and it is incorrect.

112
Although answer choice (C) contains a vertical line, the line attached to it is not
curved for all its length, and a major part of it is straight (as shown in the picture
below). Therefore, it is also incorrect.
Answer choice (E) has no vertical lines, so it cannot form “half a heart” and it is also
incorrect.

Answer choice (D) remains, making it the correct answer.

48. Explanation

The correct answer is (E).

The given shape is a parallelogram, characterized by two pairs of parallel sides, one of
which is horizontal, with the other one a diagonal inclining to the right.

Answer choice (E) contains a set of horizontal parallel lines and diagonal parallel
lines that incline to the right. Together they form the given shape, thus making this the
correct answer.

Neither of the rest of the other answer choices contain a pair of diagonal lines which
incline to the right, so they cannot form the given shape, leaving answer choice (E) as
the only possible answer.

49. Explanation

The correct answer is (E).

The shape given has four sides and four vertices. Prominent characteristics of the
given shape are the horizontal bottom line and the vertex pointing upwards. Another

113
characteristic would be the smallest side of the given shape which inclines to the right
and, as mentioned, is relatively small.

Answer choice (E) does contain a small line inclined to the right which forms the
given shape, therefore it is the correct answer.

Answer choices (A), (B), (C) do not include small lines that incline in the same
fashion and therefore cannot contain the given shape, making these answer choices
incorrect.

Although answer choice (D) does contain a small line similar to the one that appears
in the given shape, it forms a different shape than the one that is given, thus it is also
incorrect.

50. Explanation

The correct answer is (A).

The shape given is a Pentagon. Its prominent characteristics are a horizontal line (side)
at the bottom and vertices pointing upwards, to the right and to the left. Notice that
apart from the bottom side of the pentagon, all its sides are diagonal.

Answer choice (A) includes several vertices pointing upwards, two of which are more
prominent – the vertex at the top of the picture, and the vertex directly below it. Both
of those vertices connect to lines that lead to vertices pointing to the sides, as
necessary. However, the former connects to vertical lines, unlike the latter which
connects to a diagonal line that stretches down forming the pentagon given. Therefore,
this is the correct answer.

Answer choice (B) does share many characteristics of the given shape, however the
lines that connect the vertex pointing to the right and the bottom side that should form
the pentagon are vertical and not diagonal (as shown in the picture below). This
answer choice does not contain the given shape and thus it is incorrect.

114
Answer choice (C) has few vertices pointing upwards and examining them reveals
they do not form a pentagon; therefore, this is incorrect.

Answer choice (D) includes a shape similar to a pentagon, however one of its sides is
missing and was replaced with other lines. It does not contain a pentagon and
therefore it is incorrect.

Answer choice (E) contains one vertex pointing upwards which could be used to form
a pentagon; however, it forms a triangle rather than the given shape. Therefore, it is
incorrect.

51. Explanation

The correct answer is (B).

The given shape is comprised of two vertical lines and four horizontal lines; therefore,
you can deduce that the diagonal lines aim to draw attention away from the given
shape. It also can be viewed as two rectangles joined, one of which is big, and the
other is small.

In answer choice (B), prominent vertical lines are the outer lines and they have similar
proportions to the given shape. Those lines are connected to perpendicular lines that
form the top and bottom sides of the shape given. Therefore, this is the correct
answer.

Answer choice (A) is comprised of a large rectangle and a small trapezoid, instead of
a small rectangle. It does not contain the given shape and therefore it is incorrect.

The vertical lines in answer choice (C) are not in the right position to form the big
rectangle which the given shape is comprised of, therefore it cannot form it, thus it is
incorrect.

Answer choice (D)’s right vertical line is partially diagonal and therefore it does not
form the given shape, meaning it is also incorrect.

Answer choice (E) contains a large and a small rectangle as necessary. However, the
small rectangle is on the right side instead of the left side, therefore it is also incorrect.

Answer choice (B), the only one that remains, is the correct answer.

115
52. Explanation

The correct answer is (D).

The given shape is an isosceles trapezoid. It is characterized by a pair of parallel lines


joined by two congruent sides.

Although answer choice (D) shows a large isosceles trapezoid containing a triangle,
this trapezoid is in different proportions of the given shape, thus it does not indicate
this is the correct answer:

However, another way of looking at this trapezoid would be as two isosceles


trapezoids combined, each of them with the same proportions as the given shape (as
shown in the picture below). Therefore, this is, in fact, the correct answer.

Answer choice (A) contains a pair of parallel lines, but they form a parallelogram
rather than a trapezoid. Therefore, it is incorrect.

Answer choice (B), (C) and (E) contain shapes which resemble the given isosceles
trapezoid, however these are missing a line segment to completely form the given
shape, thus they are incorrect.

Solving tip: An answer choice which contains the given shape with different
proportions might use the shape characteristics to hide the actual given shape. In this
question, the large trapezoid hides the smaller trapezoids using its parallel lines and
congruent sides.

116
53. Explanation

The correct answer is (A).

The given shape is comprised of straight vertical and horizontal lines. You can look
for four vertical lines (two pairs, each with its own length) inside the shapes to help
you rule out answers.

Answers (C) and (D) are incorrect because they do not contain two pairs of such lines.

Answer (E) is incorrect because there is a side missing to get the given shape:

You are now left with answers (A) and (B); Answer (B) is incorrect because the
parallel verticals are not connected by a fully-straight line, they are connected by a
line which has a step.

The remaining answer choice (A) is the correct answer.

54. Explanation

The correct answer is (A).

The shape given is an equilateral triangle with a vertex pointing upwards. A quick
look in the answer choices shows Answer choice (B) does not contain a vertex
pointing upwards, so it cannot contain the given triangle and thus it is eliminated.

Answer choice (C) includes several triangles, however, none of them resembles the
equilateral triangle given and so it is also eliminated.

The triangle in answer choice (D) is pointing downwards (instead of upwards), and
therefore it is also incorrect.

117
Answer choice (E) contains a triangle, but it is smaller than the triangle we need to
find; therefore, this answer is also incorrect.

Answer choice (A) remains the only possible option and therefore it is correct.

55. Explanation

The correct answer is (E).

You should focus on the shape’s outline and find a prominent line or a characteristic
that will help in eliminating answer choices. The shape given can be viewed as
rectangle and a triangle with one line missing from the base of the triangle:

Answer choices (B), (C) and (D) are surrounded by squares. In order to fit into the
squares, the vertical line (side) of the triangle must appear in the top-middle part of
the squares. You can immediately eliminate answer choice (A) which does not
contain a line going through its middle.

Answer choice (B) does not contain any diagonal lines, hence it is also incorrect.

Although answer choice (C) does include a line going through the middle of the
square, it does not stretch to the top part of the picture as necessary and therefore it is
also incorrect.

Answer choice (D) does contain a diagonal line, however it is inclined to the right,
opposing the given shape (in which the triangle diagonal is inclined to the left), and so
it is also incorrect.

The only answer choice remaining is answer choice (E), which is the correct answer.

56. Explanation

118
The correct answer is (B).

The shape presented is a rhombus. It is characterized by two pairs of parallel sides and
in this question, it has two vertices pointing upwards and downwards and two vertices
pointing towards the sides. Another way of picturing a rhombus is as two identical
triangles with a joined base line:

Answer choice (A) contains several parallel lines, however it does not contain any
vertices pointed upwards or downwards and therefore cannot be the correct answer.

Answer choice (C) contains one triangle pointing left, but its base is joined with a
trapezoid rather than another triangle, and so it is also incorrect.

Answer choice (D) has no triangles and therefore cannot be the correct answer.

Answer choice (E) has one side missing from the rhombus and so is also incorrect.

The answer remaining – answer choice (B) is the correct answer.

57. Explanation

The correct answer is C.

The answer choices in this question include many different details meant to confuse
and hide the shape. Focusing on specific characteristics of the shape will help a great
deal in not being thrown off by the number of lines and shapes involved.

Look at the picture below:

119
The four marked vertices can be helpful in finding the shape. Vertices 1 and 3 point to
the left and down, respectively. Thus, the correct answer choice must contain vertices
pointing the same way. Moreover, notice that vertices 1 and 3 are connected with a
diagonal which has a missing part. In addition, vertices 2 and 4 are facing the same
direction (up-right), thus creating some sort of “parallel corners”.

Answer choice (A) contains many parallel lines, however the “parallel corners” it
contains are too small to form the shape given, therefore it is incorrect.

Answer choice (B) contains one diagonal, however that diagonal is not connected to a
“parallel corner” facing the correct direction (up-right) and so is incorrect.

Answer choice (D) contains only one “parallel corner” which faces the correct
direction, however the diagonals are not passing through the corner to create the
correct shape, therefore it is also incorrect.

Answer choice (E) has no “parallel corners” facing the right direction, thus also
incorrect.

The only remaining answer choice (C) is the correct answer.

58. Explanation

The correct answer is (E).

The shape given contains an arc and two perpendicular lines, one vertical and the
other horizontal:

120
Although answer choice (A) contains perpendicular lines, they are diagonal which
means they cannot form the given shape, thus it is eliminated.

Answer choice (B) contains one horizontal line, which is the base of the triangle. This
line is too short to form the shape given, thus answer choice (B) is also eliminated.

Answer choice (C) contains no vertical lines, therefore it cannot form the given shape
and it is eliminated as well. Notice it also contains the shape given rotated 90 degrees
clockwise as a distractor.

Answer choice (D) does contain perpendicular lines, however they are rotated 180
degrees. Therefore, it cannot contain the given shape and it is incorrect.

The remaining answer choice (E) contains the shape and it is the correct answer.

59. Explanation

The correct answer is (D).


The shape given is comprised of a rectangle on top of which is attached an isosceles
triangle. The triangle’s vertex is located in the middle of the shape and so it should
appear in the answer choices in the same position. Moreover, the horizontal lines in
the rectangle stretch from one side of the picture to the other, excluding the base of
the triangle, so it should act the same way in the answer choices.
Answer choices (B), (C) and (D) are the only one with vertices in the correct place.
Notice answer choice (A) includes two triangles on the top part. However, neither is
in the correct position. Answer choice (E) includes a right-angled triangle instead of
an isosceles triangle. Therefore, both answer choices (A) and (E) cannot form the
given shape and they are incorrect.
Answer choice (B) is missing a horizontal line segment on the right, therefore it
cannot form the given shape and it is incorrect.
Answer choice (C) is comprised from a triangle on top of a trapezoid instead of a
rectangle and, therefore, it is also incorrect.

121
The remaining answer choice (D) includes both the vertex needed and the horizontal
lines needed to form the given shape, making it the correct answer.

60. Explanation

The correct answer is (A).

The shape given is amorphic. In order to find the given shape, it is best to use its
prominent characteristics and look for them in the answer choices. In this question,
the shape includes a vertex which seemingly “points” to the left. In addition, the lines
forming the given shape incline in a unique way that should be kept in the correct
answer choice.

Answer choice (A)’s left-most vertex, alongside the sides which create it, matches the
given shape’s proportions and, in fact, form the given shape. Therefore, this is the
correct answer.

Answer choices (B), (C), (D) and (E) do not contain the given shape. None of them
create the given shape with accurate proportions and therefore they are incorrect.

Answer choice (D), for example, shows a similar shape with different proportions as
seen below:

61. Explanation

The correct answer is (B).

122
The shape given is two equilateral triangles pointing towards each other. The pictures
given include many details that are meant to draw attention away from the given
shape, therefore it is important to search the shape characteristics thoroughly. This
shape is characterized by having two parallel vertical lines and two diagonal lines that
cross each other.

Answer choice (B) contains a prominent pair of diagonal lines crossing each other. A
closer look shows these diagonal lines are part of two equilateral triangles pointing
towards each other, forming the given shape. Therefore, it is the correct answer.

Answer choice (A)’s only vertical parallel lines are too far apart to keep the
proportions of the given shape. Thus, it does not contain the given shape and it is
eliminated.

Although answer choice (C) contains two parallel vertical lines, those lines do not
form two equilateral triangles. Therefore, it also does not contain the given shape and
it is eliminated.

Answer choice (D) contains only one vertical line, so it cannot contain the given
shape and therefore it is eliminated.

Answer choice (E) has two vertical parallel lines. However, they are too close to form
the given shape and are not a part of two equilateral triangles, therefore it also cannot
create the given shape and is eliminated.

We are left with answer choice (B) as our only option, which is the correct answer.

62. Explanation

The correct answer is (C).

The given shape is a crescent with a horizontal line going through its middle. The
pictures in this question are all aquatic animals drawn with complex lines aimed to
draw attention away from the correct shape.

Answer choice (C) contains only one horizontal line which is inside a crescent as
needed, therefore it is the correct answer.

123
Answer choices (A), (B) and (D) do not contain any horizontal line, thus they cannot
contain the given shape and are incorrect.

Answer choice (E) contains one horizontal line at the top of the fish’s fin, however it
does not form the given shape and therefore it is also incorrect.

63. Explanation

The correct answer is (B).

The given shape is a "C". It is comprised of two semi-ellipses, one inside the other,
which are joined with a vertical line.

Answer choice (B) includes two ellipses, one inside the other, connected with a
vertical line. Focusing on the left side of the picture and ignoring the diagonal lines
drawn on the shape, it is possible to see the “C”. Therefore, this is the correct answer.

Answer choice (A) contains a crescent rather than a “C”, therefore it is in correct.

Answer choice (C) contains a circle inside of an ellipse, rather than two ellipses (as
shown below). Therefore, it does not contain the given shape and it is incorrect.

Answer choice (E) includes only one ellipse, thus it cannot contain the given shape
and it is incorrect.

Answer choice (D) does contain two ellipses. However, they are not connected with
vertical lines and therefore cannot form the given shape. This answer is also incorrect.

Answer choice (B) remains, which is the correct answer.

124
64. Explanation

The correct answer is (D).

The shape given is “half a heart” which can also be described as a vertical line
attached with a curved line.

Answer choice (D) contains two vertical lines that can serve as part of the shape. The
right vertical line is, in fact, connected with a curved line, thus forming “half a heart”
as needed. Therefore, it is the correct answer.

Answer choice (A) contains several vertical lines. However, none of the shapes
attached to those lines create the correct angles to form “half a heart”.

Answer choice (B) includes a shape similar to “half a heart”. However, the curved
line is not curved for all its length (as shown in the picture). Therefore, it does not
contain the given shape and it is incorrect.

Although answer choice (C) contains a vertical line, the line attached to it is not
curved for all its length, and a major part of it is straight (as shown in the picture
below). Therefore, it is also incorrect.
Answer choice (E) has no vertical lines, so it cannot form “half a heart” and it is also
incorrect.

Answer choice (D) remains, making it the correct answer.

65. Explanation

The correct answer is (E).

125
The given shape is a parallelogram, characterized by two pairs of parallel sides, one of
which is horizontal, with the other one a diagonal inclining to the right.

Answer choice (E) contains a set of horizontal parallel lines and diagonal parallel
lines that incline to the right. Together they form the given shape, thus making this the
correct answer.

Neither of the rest of the other answer choices contain a pair of diagonal lines which
incline to the right, so they cannot form the given shape, leaving answer choice (E) as
the only possible answer.

66. Explanation

The correct answer is (E).

The shape given has four sides and four vertices. Prominent characteristics of the
given shape are the horizontal bottom line and the vertex pointing upwards. Another
characteristic would be the smallest side of the given shape which inclines to the right
and, as mentioned, is relatively small.

Answer choice (E) does contain a small line inclined to the right which forms the
given shape, therefore it is the correct answer.

Answer choices (A), (B), (C) do not include small lines that incline in the same
fashion and therefore cannot contain the given shape, making these answer choices
incorrect.

Although answer choice (D) does contain a small line similar to the one that appears
in the given shape, it forms a different shape than the one that is given, thus it is also
incorrect.

67. Explanation

The correct answer is (A).

The shape given is a Pentagon. Its prominent characteristics are a horizontal line (side)
at the bottom and vertices pointing upwards, to the right and to the left. Notice that

126
apart from the bottom side of the pentagon, all its sides are diagonal.

Answer choice (A) includes several vertices pointing upwards, two of which are more
prominent – the vertex at the top of the picture, and the vertex directly below it. Both
of those vertices connect to lines that lead to vertices pointing to the sides, as
necessary. However, the former connects to vertical lines, unlike the latter which
connects to a diagonal line that stretches down forming the pentagon given. Therefore,
this is the correct answer.

Answer choice (B) does share many characteristics of the given shape, however the
lines that connect the vertex pointing to the right and the bottom side that should form
the pentagon are vertical and not diagonal (as shown in the picture below). This
answer choice does not contain the given shape and thus it is incorrect.

Answer choice (C) has few vertices pointing upwards and examining them reveals
they do not form a pentagon; therefore, this is incorrect.

Answer choice (D) includes a shape similar to a pentagon, however one of its sides is
missing and was replaced with other lines. It does not contain a pentagon and
therefore it is incorrect.

Answer choice (E) contains one vertex pointing upwards which could be used to form
a pentagon; however, it forms a triangle rather than the given shape. Therefore, it is
incorrect.

The remaining answer choice (A) is the correct answer.

68. Explanation

The correct answer is (B).

The given shape is comprised of two vertical lines and four horizontal lines; therefore,
you can deduce that the diagonal lines aim to draw attention away from the given
shape. It also can be viewed as two rectangles joined, one of which is big, and the
other is small.

127
In answer choice (B), prominent vertical lines are the outer lines and they have similar
proportions to the given shape. Those lines are connected to perpendicular lines that
form the top and bottom sides of the shape given. Therefore, this is the correct answer.

Answer choice (A) is comprised of a large rectangle and a small trapezoid, instead of
a small rectangle. It does not contain the given shape and therefore it is incorrect.

The vertical lines in answer choice (C) are not in the right position to form the big
rectangle which the given shape is comprised of, therefore it cannot form it, thus it is
incorrect.

Answer choice (D)’s right vertical line is partially diagonal and therefore it does not
form the given shape, meaning it is also incorrect.

Answer choice (E) contains a large and a small rectangle as necessary. However, the
small rectangle is on the right side instead of the left side, therefore it is also incorrect.

Answer choice (B), the only one that remains, is the correct answer.

69. Explanation

The correct answer is (D).

The given shape is an isosceles trapezoid. It is characterized by a pair of parallel lines


joined by two congruent sides.

Although answer choice (D) shows a large isosceles trapezoid containing a triangle,
this trapezoid is in different proportions of the given shape, thus it does not indicate
this is the correct answer:

However, another way of looking at this trapezoid would be as two isosceles


trapezoids combined, each of them with the same proportions as the given shape (as
shown in the picture below). Therefore, this is, in fact, the correct answer.

128
Answer choice (A) contains a pair of parallel lines, but they form a parallelogram
rather than a trapezoid. Therefore, it is incorrect.

Answer choice (B), (C) and (E) contain shapes which resemble the given isosceles
trapezoid, however these are missing a line segment to completely form the given
shape, thus they are incorrect.

Solving tip: An answer choice which contains the given shape with different
proportions might use the shape characteristics to hide the actual given shape. In this
question, the large trapezoid hides the smaller trapezoids using its parallel lines and
congruent sides.

70. Explanation

The correct answer is (A).

The given shape is comprised of straight vertical and horizontal lines. You can look
for four vertical lines (two pairs, each with its own length) inside the shapes to help
you rule out answers.

Answers (C) and (D) are incorrect because they do not contain two pairs of such lines.

Answer (E) is incorrect because there is a side missing to get the given shape:

You are now left with answers (A) and (B); Answer (B) is incorrect because the
parallel verticals are not connected by a fully-straight line, they are connected by a
line which has a step.

The remaining answer choice (A) is the correct answer.

129
71. Explanation

The correct answer is (B).

The shape given is a plus (+). It is possible viewing it as two rectangles – one is
vertical and the other horizontal:

Since all answer choices given are surrounded by a square and the proportions of the
shape needed to be found must be kept, we can conclude there must be two vertical
and two horizontal lines going through the middle of the square stretching from one
side to the other. Answer choice (B) is the only one containing those lines and thus, it
is correct.

Solving tip: If you are unable to find the correct answer, answer choice elimination
could be a powerful tool (as shown in this question). The logic involves focusing on
the characteristics of the given shape (such as proportions, vertices, size and more) to
eliminate any answer choice that does not share those characteristics.

Answer choice (A) is incorrect as it does not contain any vertical lines going through
the middle of the square, so it cannot contain the plus.

Answer choice (C) contains only one horizontal inner line going through the middle,
thus it cannot contain the plus and it is incorrect.

Although both answer choices (D) and (E) have the correct amount of inner horizontal
and vertical lines, each contains a vertical line which is too short to be the side of a
plus, and so they are also incorrect.

The only answer choice left is (B), which is the correct answer.

72. Explanation

The correct answer is (E).

Start by analysing the shape you need to identify. Once you know its properties, you

130
can use these to find the correct answer or rule out other answer choices.

The shape is symmetric with respect to a horizontal line. Furthermore, the shape given
has three pairs of parallel lines (one pair of lines that are horizontal, and the last two
pairs can be found above and below the symmetry line) and the lines in these pairs
have the same length. Finally, the intersecting edges (sides) in the middle of the shape
are directed to the right. Another way to see this is by thinking of the middle vertices
as changing the direction of the lines.

Now that you know the characteristics to look for, examine the answer choices:

Answer (E) is correct because you can see the small shape included here:

Answer (A) is incorrect because it does not include the smaller shape. You can see
this by noticing that no pair of lines that are parallel have the same length, or because
there is no vertex that changes the direction of the side lines as in the original shape.

Answer (B) is incorrect because even though it has many pairs of parallel lines, the
shape is not included inside it. You can see this by focusing on the vertex mentioned
before, which changes the direction of the lines (the intersection of the diagonals) but
the original shape has two such vertices, and answer (B) only has one. Answer (B) is
also incorrect because it has no inner parallel lines .

Answer (C) is incorrect because the inside lines intersect at different heights, meaning
it is asymmetric, as opposed to the original shape.

Answer (D) is the most interesting answer among the incorrect ones. This is because
it has parallel sides of equal length and two vertices that change the direction of lines.
However, the lines that change the direction are not parallel to one another as in the
original shape. Therefore, this answer is incorrect.

Solving Tip: Remember that symmetry can always be a helpful tool to find the
correct answer or rule out others. Another tool that will help you is finding parallel
lines and compare them to others to see if they share the same qualities as the ones in
the given shape.

131
73. Explanation

The correct answer is (A).

The shape given is a right-angled triangle. The pictures contain many shapes and lines
that are meant to draw the attention away from the shape. A good way of finding the
shape would be to look at its characteristics and choose the one which will be easiest
to find in the pictures. In this question, the triangle presented contains a horizontal
line as one of its sides and you can notice that there are few horizontal lines in the
given pictures.

Answer choice (A) includes three horizontal lines, one goes through its middle and
the others are the sides of the surrounding square. A further examination of the left
side of the square shows there is a perpendicular vertical line crossing it that could be
used as another side of the triangle. The diagonal line that connects them completes
the triangle, forming the given shape. Therefore, this is the correct answer.

Answer choice (B) contains one horizontal line going through its middle and one line
that is perpendicular to it. However, the diagonal line that completes them to form a
triangle creates a smaller triangle than the one given. Therefore, it does not contain
the given shape and it is incorrect.

Answer choice (C) includes one horizontal line which serves as a side of a
right-angled triangle, but that triangle is pointing downwards instead of upwards.
Thus, it also does not contain the given shape and it is also incorrect.

Answer choice (D) contains perpendicular lines that can serve as two sides of a
triangle. However, all the triangles that use those lines are in the wrong direction.
Therefore, it does not contain the given shape and it is incorrect.

Similarly, answer choice (E) also contains perpendicular lines, however they serve as
sides for a taller triangle than the given triangle. Therefore, it also does not contain the
given shape and it is incorrect.

132
74. Explanation

The correct answer is (E).

The given shape is a kite comprised of two isosceles triangles. It has one horizontal
line going through its middle which serves as a base for the two isosceles triangles.

Answer choice (E) contains few horizontal lines. After ruling out the square’s sides, it
has one horizontal line which is the same length as the horizontal line that appears in
the given shape. Upon examination, you will notice it serves as a base for two
isosceles triangles as needed, thus forming the given shape. Therefore, answer choice
(E) is the correct answer.

Answer choice (A) includes two horizontal lines, one of which is the bottom side of
the rectangle which cannot be used to form the given shape. The other one is a base
for just one isosceles triangle and not two as needed and, therefore, it does not contain
the given shape and is incorrect.

Answer choice (B) contains few horizontal lines, none of which serve as a base for
two isosceles triangles, and therefore it is also incorrect.

Answer choice (C) includes two isosceles but they are pointing towards each other
instead of sharing the base of each other. Thus, they cannot form the given shape,
making this answer also incorrect.
Answer choice (D)’s only horizontal lines are the sides of the square which do not
serve as a base for two isosceles triangles and therefore it is also incorrect.
The remaining answer choice (E) is the correct answer.

75. Explanation

The correct answer is (D).

The given shape is rather complicated. Its prominent characteristics are parallel
straight lines (horizontal and vertical), and right-angled edges. Therefore, diagonal
lines in the given pictures are only meant to draw attention away from the given shape.
In addition, all of the given pictures share the same height as the given shape, and the

133
proportions of the given shape must be kept, therefore the horizontal lines’ height in
the pictures should also be positioned at the same height as shown in the given shape.

Answer choice (D) includes horizontal lines positioned at the same height as shown in
the given picture. Ignoring the diagonal line, the sides of the square create the right
angles and form the given shape. Therefore, this is the correct answer.

Answer choice (A) also includes horizontal lines in the correct place. However, it is
missing a vertical line near the left side of the rectangle in order to form the given
shape. Thus, it does not contain the given shape and it is incorrect.

Answer choice (B) includes two inner horizontal lines, one of which is too short for
the given shape; therefore, it does not contain the given shape, making it incorrect.

Answer choice (C) does contain horizontal lines as needed, but the right most inner
vertical line is too short to form the given shape and therefore it is also incorrect.

Answer choice (E) also includes horizontal lines in the correct place, however it
contains only one vertical line in the left side of the rectangle, thus it cannot form the
given shape, making it incorrect.

76. Explanation

The correct answer is (C).

The given shape is comprised of straight lines and contains a vertex pointing to the
right. Another prominent feature is that two of its sides are parallel vertical lines.

Answer (C) is correct because the given shape is included. You can see this more
quickly by noticing the left half of the shape. The triangle that is formed inside the
shape is meant to distract and conceal the given shape.

Answer (A) is incorrect because the angles of the side lines (the lines to the right of
the shape) are different from the given shape.

Answer (B) contains a similar shape to the one given, but the angle formed in the
rightmost vertex is not the same as the angle in the given shape (furthermore, even if
the angle were the same angle, the dimensions would be bigger than the given shape)

134
Answer choices (D) and (E) include two parallel vertical lines, however they are
closer to each other or further from each other, respectively, than shown in the given
picture. Since the proportions of the given shape need to be kept, they cannot contain
the given shape and they are incorrect as well.

77. Explanation

The correct answer is (D).

The shape given includes several parallel straight lines which are either horizontal or
vertical. Thus, diagonal lines drawn in the pictures are only meant to distract you from
the correct answer. In addition, the straight lines create two distinctive right angles, as
shown below:

Answer choice (D) contains many diagonal lines; therefore, it is important to focus on
the straight lines. Some of the straight lines inside the square in the picture are
perpendicular, thus creating right angles as necessary. Examining those corners
reveals the given shape. Therefore, this is the correct answer.

Answer choice (A) is missing a line segment in order to form the given shape,
therefore it is incorrect.

If this shape is inside one of the options above, that option must have its upper half
divided into two by a vertical line. Answers (B), (C), and (E) are incorrect because
there is no such vertical line.

135
78. Explanation

The correct answer is (B).

The given shape is a right-angled triangle pointing upwards to the right. Look for this
angle in the options above. As soon as you recognize the angle, check to see if you
can find the given shape based on that.

Answer choice (B) includes two right angles pointing in a direction similar to the
given shape, as shown in the picture below. Notice that angle #2 (as marked in the
picture below) is actually a part of the given shape. Therefore, this is the correct
answer:

Answer choice (A) includes a few right-angles vertices, but none of them form a
triangle pointing in the right direction. Therefore, it does not include the given shape
and it is incorrect.

Answers (C) and (E) are incorrect because they do not contain a 90-degree angle.

You can also rule out answer (D) because even though it contains 90-degree angles,
no triangle is formed using those angles.

79. Explanation

The correct answer is (B).

The given shape is a plus (‘+’), comprised of vertical and horizontal lines, therefore it
is possible to deduce that the diagonal lines are only meant to draw attention away
from the correct answer.

Answer choice (B) contains an isosceles triangle which is used to cover the given
shape, which appears in the top left side of the picture, making this the correct answer
choice.

136
Answer choices (A), (C), (D) and (E) include several vertical and horizontal lines,
however they either lack line segments to form the given plus or have different
proportions than those of the given plus. Therefore, they are incorrect.

80. Explanation

The correct answer is (A).

The given shape has few prominent features; a vertical line, two vertices -- one of
which points to the right while the other points to the left -- and four parallel
horizontal lines. Using those features you should try and locate the given shape.

Answer choice (A) includes vertices pointing to the left and the right. Following those
vertices, it is possible to see they connect using horizontal lines to form the given
shape, making answer choice (A) the correct answer.

Answer choice (B) also contains vertices pointing in the right directions, however
they are too close to one another to form the given shape. Is answer choice does not
contain the given shape and therefore it is incorrect.

The only straight lines answer choice (C) includes are the boundaries of the picture,
meaning they cannot form the angles which appear in the given shape. It does not
contain the given shape and therefore it is also incorrect.

Answer choice (D), like answer choice (B), contains vertices pointing in the right
directions, however here those vertices are too far apart to form the given shape,
making answer choice (D) incorrect.

Answer choice (E) is also incorrect, as it does not contain a vertex pointing to the
right, which means it cannot contain the given shape.

81. Explanation

The correct answer is (A).

The given shape is an isosceles trapezoid comprised of parallel sides, which are

137
diagonal, a vertical line and a horizontal line. A quick way to rule out answers would
be to look for diagonals which incline with the same angle and have the same distance.

Answer (A) contains diagonals which incline with an angle similar to the trapezoid
parallel sides. In addition, they intersect with the outer lines of the square (which is
comprised of vertical and horizontal lines), thus forming the given trapezoid.

Answer (B) includes a pair of diagonal parallel lines, however they are too close to
one another to form the given trapezoid, thus it is incorrect.

Answer (C) includes trapezoids, however they are rotated 180 degrees and also have
different sizes than the one given.

Answers (D) and (E) also include a pair of diagonal parallel lines which incline with
the same degree shown in the given trapezoid. However, those lines are too far from
one another to form the given shape.

82. Explanation

The correct answer is (D).

The shape presented includes two parallel edges (sides) that look similar to a
parallelogram. That parallelogram was joined with a triangle on its side:

Notice that one set of the parallel side is horizontal while the other is inclined to the
right.

Answer choice (A) has no parallel lines that are inclined and therefore cannot contain
the given parallelogram, and so this answer is incorrect.

Answer choice (B) has parallel lines that incline similarly to the given shape, however

138
the triangles in the picture are not joined with them, thus it cannot contain the shape,
making this choice incorrect.

Answer choice (C) contains triangles that are different than the one in the given shape
and therefore it also cannot contain the shape and is incorrect.

Answer choice (E) has parallel lines; however, they do not incline with the same
angle at which the given shape inclines, so they cannot be used to create a correct
parallelogram. Therefore, it cannot contain the given shape, making it also incorrect.

83. Explanation

The correct answer is (E).

It is important to remember the answer choice must contain the given shape, while
keeping its original proportions. The given shape includes two vertices pointing
upwards-right:

Answers (A) and (D) are incorrect because they do not contain the given shape. Both
these answers include similar, but not identical shapes.

Answer (B) is incorrect because the angle formed by the vertex at the top is not the
same as the one in the given figure:

Answer (C) contains the given figure but rotated 90 degrees. Therefore, it is also
incorrect.

The remaining answer (E) is the correct answer.

139
84. Explanation

The correct answer is (A).

As you can see in the picture below, the given shape contains two parallel horizontal
lines and 2 vertices pointing to the left as well as a vertical line:

Try to find a characteristic in the given shape that is easy to spot. In this case, the
90-degree angle at the bottom-left corner. Almost immediately after the angle is
formed there is a diagonal line to look for.

Answer choice (A) includes several horizontal lines, and those in the middle of the
shape are at the same distance as those in the given shape. Those lines also connect to
two vertices which point to the left, helping you notice the given shape. Therefore,
this is the correct answer.

Answers (B), (D), and (E) can be ruled out for not having both the properties (in the
case of answer (D), there are no 90-degree angles at all).

Answer choice (C) includes a shape which resembles the given shape, but it has
different proportions, and therefore it is also incorrect.

85. Explanation

The correct answer is (B).

The given shape is a parallelogram (which is characterized by two pairs of congruent


parallel sides). Notice one pair of its sides is vertical while the other pair is diagonal.

Answer choice (B) contains only one pair of vertical lines. Focusing on those lines, it
is possible to note that they intersect with a pair of diagonal lines to form the given

140
parallelogram, making answer choice (B) the correct answer.

Answer choice (A) includes two vertical lines which form a rectangle, rather than a
parallelogram. Therefore, it does not contain the given shape and it is incorrect.

Answer choices (C) and (E) do not include a pair of vertical lines, thus they cannot
contain the given parallelogram and they are also incorrect.

Answer choice (D) contains a pair of parallel vertical lines; however they are too far
apart to form the given shape, making this choice also incorrect.

86. Explanation

The correct answer is (E).

The given shape is a right-angled triangle comprised of a vertical, horizontal and a


diagonal line stretching from top to bottom-right. You should look for the 90-degree
angle to rule out answers more easily.

Answer choice (E) includes a square whose diagonal lines intersect and create
right-angled triangles. One of those triangles has the same orientation and size as the
given triangle, making answer choice (E) the correct answer.

Answers (A), (C), and (D) are incorrect because even though there is a 90-degree
angle in the same direction as the given triangle, there are no diagonal sides to
complete the triangle.

Answer (B) is incorrect because it has no 90-degree angle.

87. Explanation

The correct answer is (C).

The shape given is comprised of straight vertical and horizontal lines. Therefore, it is
possible to deduce that the diagonal lines are meant to draw attention away from the

141
given shape.

Answer choice (C) depicts a grid and a triangle, although the triangle, which is
comprised of diagonal lines, is only meant as a distraction. The grid itself is
comprised of vertical and horizontal lines which form the given shape, making answer
choice (C) the correct answer.

Answer choices (A), (B), (D) and (E) are missing line segments to form the correct
shape, therefore they are incorrect.

Note: Answer choice (B) includes a shape similar to the one given, but with different
proportions, as shown below.

88. Explanation

The correct answer is (E).

The given shape contains diagonal lines and two parallel horizontal lines.

Answer (A) is missing a line segment to form the top vertex in order to form the given
shape, and therefore it can be ruled out.

Answer (B) includes a pair of horizontal lines but is missing a vertex on the right in
order to form the correct shape, making it also incorrect.

142
Answer (C) has two parallel horizontal lines, however they are too far apart to form
the given shape, thus it is eliminated.

Answer (D) includes a shape similar to the given shape, however its left side has a
different angle than the left side of the given shape (As shown in the picture below).
In addition, the vertex on the right has a different angle than the one in the given
shape. Therefore, it does not include the given shape and it is also incorrect.

89. Explanation

The correct answer is (D).

The given shape is a parallelogram with two vertical parallel sides. Notice that the
vertical parallel lines in answers (A), (B) and (C) are too far from one another to form
the given shape, and thus you can eliminate them.
Answer (E), on the other hand, includes several parallel vertical lines; however, none
of them are intersected by a pair of diagonal parallel lines as in the given
parallelogram. Therefore, it is also incorrect.
The remaining answer (D) is the correct answer. You can notice the parallelogram in
the figures top-left side by observing the vertical parallel lines intersecting with the
parallel diagonals.

143
90. Explanation

The correct answer is (A).

The given shape is an equilateral triangle which have vertices pointing up, down and
right. Another prominent feature of this triangle is its left side which is a vertical line.

Answer (A) includes two vertical lines which can be served as the left side of the
triangle, so in order to find the given triangle you can look for a vertex pointing to the
right to the right side of that line. When observing the left vertical line, you can notice
such vertex does exist and find the given shape. Therefore, this is the correct answer.

Answer (B) has an incomplete vertical on its left side and no other vertical lines,
therefore it cannot form the given shape and it is incorrect.

Answer (C) includes a vertical line which has a vertex to its right which also point in
the right direction, however it has different proportions than the given shape (as you
can see in the picture below). Therefore it does not contain the given shape and it is
incorrect.

Answer (D) includes a vertical line and to its right many vertices pointing to the right.
However, the triangles formed using that line are too big to form the given triangle,
and therefore it is incorrect.

Answer (E) has a prominent vertex pointing right, however it is not part of a triangle.
It does not contain the given shape and therefore it is incorrect.

144
91. Explanation

The correct answer is (A).

The given triangle has vertices pointing up, right and down-left. It also has a side
which is a horizontal line.

Answer (A) has only one horizontal line. In order to form the shape, it must intersect
with two different diagonal lines and form the correct angles. Once focused on that
line it is possible to notice it creates a vertex pointing to the right and forms the given
shape. Therefore, this is the correct answer.

Answer (B) includes two horizontal lines as the outer lines of the figure. The bottom
line cannot form the given triangle, as the horizontal line of the triangle is on the top
of the shape. The top line of answer (B), forms a shape similar to the given triangle;
however, it is bigger. Since the proportions must be kept, it does not contain the given
shape and it is incorrect.

Answer (C) contains two horizontal lines. In addition, those line are intersected by a
diagonal going from bottom-left to top-right with the same angle as the bottom side of
the given triangle. However, the horizontal lines are not intersected by a line with the
same angle as the left side of the given triangle. Therefore, it does not contain the
given shape and it is also incorrect.

Answer (D) includes two horizontal lines as the outer lines of the figure. The bottom
line cannot form the given shape, as the horizontal line of the shape is on the top of
the shape. The top line is missing a segment in order to form the given shape, and
therefore it also does not contain the shape and it is incorrect.
You can also notice a similar, but larger, triangle appears in this answer choice.

Answer (E) has three horizontal lines. Just like answers (B) and (D), the one in the
bottom cannot be part of the given triangle. The other two horizontal lines are missing
a side to create the given shape.

145
92. Explanation

The correct answer is (B).

The strong features of the given shape are an isosceles triangle pointing right (without
its base) and a pair of parallel horizontal lines intersected with a perpendicular vertical
line:

Answer (B) uses symmetry to hide the given shape. The rectangle which appears in
the picture has two parallel horizontal lines as necessary, and the rotated square
instead has a vertex pointing to the right. Once you notice this, you can follow the
lines to form the given shape. Therefore, this is the correct answer. Notice that the left
side of the rectangle is the perpendicular vertical line.

Answer (A) includes a shape similar to the given shape, however the triangle shown
in the picture has different angles than those of the given shape, making answer (A)
incorrect.

Answer (C) has a pair of parallel horizontal lines which intersect with a vertical line.
However, the left side is not connected to an isosceles triangle pointing to the right, as
needed. It does not contain the given shape and therefore it is incorrect.

Answer (D) includes only one isosceles triangle and it is too big to form the given
shape. Moreover, the upper part of the triangle is not connected with a horizontal line,
as needed, making answer (D) incorrect.

Answer (E) has no vertical lines at all, so it cannot form the given shape. Therefore, it
is also incorrect.

146
93. Explanation

The correct answer is (D).

The shape given is comprised of three horizontal parallel lines and three diagonal
lines, two of which are parallel, as can be seen in the picture below.

Answer choice (D) includes two prominent diagonal parallel lines which incline with
the same angle as the given shape, so this would be the best place to look for the
given shape. Once you examine those lines you can see the three horizontal parallel
lines intersecting them, forming the given shape. Therefore, this is the correct answer.

Answer choice (A) includes two parallel diagonal lines which incline with the same
angle as the given shape, however they do not form the given shape; therefore, this
answer choice is incorrect.

94. Explanation

The correct answer is (C).

The given shape is a pentagon, which includes a vertex pointing upwards.

Answer choice (C) includes one vertex pointing upwards in the middle of the picture.
Further examination reveals it is the top part of the given pentagon.
The picture below shows that this image actually includes two pentagons combined.

Answer choice (A) includes a shape which resembles a pentagon, however one of its
sides has been detached and changed. It does not contain the given shape and
therefore it is incorrect.

147
Answer choice (B) and (D) also include shapes which resemble the given pentagon,
however those shapes include a side which is vertical instead of diagonal. Therefore,
they do not include the exact given shape, making them incorrect.

Answer choice (E) includes trapezoids and triangles instead of a pentagon, making it
also incorrect.

95. Explanation

The correct answer is (D).

The given shape is comprised of two horizontal parallel lines and diagonal lines. In
addition, it includes two vertices pointing to the right:

Answer choice (D) shows a parallelogram whose horizontal parallel lines are at the
same distance from one another as the given shape’s parallel lines. The diagonal sides
of that parallelogram form the same angles as the left side of the given shape (as
shown in the picture below).

However, only the left side of the parallelogram can be used as the left side of the
given shape (as the only other line with the same angle is the right side of the
parallelogram, and it has no lines to its right to form the given shape). Once you focus
on this line, you can notice the diagonal lines to its right form the given shape.

Answer choice (A) does include two vertices pointing to the right, however the
diagonal lines which form them have different angles than the lines of the given shape,
making it unproportionate.

Answer choice (B) includes one diagonal line which inclines with the same angle as
the left side of the given shape, however this line cannot form the given shape, as it

148
does not have enough lines to its right.

Answer choices (C) and (E) do not include diagonal lines which incline with the same
angle as the left side of the given shape, therefore they do not contain the given shape
and they are also incorrect.

The remaining answer choice (D) is the correct answer.

96. Explanation

The correct answer is (D).

You can picture the given shape as two rectangles, one vertical and the other
horizontal.

Notice the vertical rectangle of the given shape is located in the middle of the
horizontal rectangle. Therefore, our first hint would be to look for the vertical
rectangle within the inner lines of the pictures, rather than in the borders, since
forming the given shape using a vertical rectangle appearing on the borders would
cause part of it to be outside of the borders.

Therefore, the answer must appear in one of two different locations:

Answer (D) includes a pair of vertical and parallel inner lines. Those lines are
perpendicular to horizontal lines, and focusing on them reveals that together, they
form the given shape. Therefore, answer (D) is the correct answer.

Answer (A) includes parallel vertical inner lines, as necessary. However, it is missing
a line segment to form the given shape:

149
Answer (B) has two vertical parallel inner lines. However, those are not in the middle
of a horizontal rectangle and therefore it does not form the given shape and it is
incorrect.

Answer (C) includes vertical inner lines, however only one is long enough to be a side
for a vertical rectangle. Therefore, it does not contain the given shape and it is
incorrect.

Answer (E), like answer (A), is missing a line segment to form the given shape.

97. Explanation

The correct answer is (E).

The given shape is an isosceles trapezoid with a pair of vertical parallel lines and a
pair of congruent diagonal sides.

Answer (E) includes three vertical parallel lines (as shown in the picture below). The
leftmost line is too far from the middle line to form the given shape. However, the
middle line and the rightmost line are close enough, and they are being intersected by
diagonals, as necessary, forming the given shape. Therefore, answer (E) is the correct
answer.

Answer (A) includes several vertical lines, so using them to find the trapezoid might
get tricky. Instead, you can notice the upper outer line of the figure is a diagonal
inclining with the same angle as the upper side of the trapezoid. In order to contain
the shape, the figure must have a diagonal line with the same angle as the bottom line
of the trapezoid close enough to the upper line to form the shape. You can notice that
it only happens in the right side of the picture (as shown in the picture below).
However, it is still missing a line segment to form the given shape, and therefore

150
answer (A) is incorrect.

Answer (B) does not include a diagonal line with the same angle as the bottom line of
the trapezoid. The figure has only one line stretching from bottom-left to top-right and
its angle is different. Therefore, it is also incorrect.

Answer (C) has only two vertical parallel lines and they are too far apart to form the
given shape, making it also incorrect.

Answer (D) contains one isosceles trapezoid, but it is bigger than the given shape and
therefore it is also incorrect:

98. Explanation

The correct answer is (B).

The given shape is half a star. It has vertices pointing up, right and down-right. In
addition, it has a vertical line.

Answer (B) includes one vertical line and attached to it are lines forming vertices
pointing towards the right. It contains the given shape, making it the correct answer.

Answer (A) has no vertical lines and therefore it cannot contain the given shape,
making it incorrect.

Answer (C) includes a shape which resembles the half-star given. However, the angle
of the vertex pointing bottom-right is different. Moreover, the lengths of the sides
forming the star are different.

Answer (D) has no horizontal lines, and therefore cannot form the given shape (as it
must include a horizontal line), and therefore it is incorrect.

151
Answer (E) includes a vertical line; however, it is too short to form the given shape.
Therefore, it is also incorrect.

99. Explanation

The correct answer is (C).

The given shape is an arrowhead pointing left. It is comprised of a pair of parallel


vertical lines and two vertices pointing left next to each other.

Answer (C) includes three parallel horizontal lines. The line that goes through the
middle of the trapezoid can be overlooked, as it is too close to the other horizontal
lines in order to form the given shape. In addition, this figure includes diagonals
which incline with angles similar to those of the given shape, and some of them create
vertices pointing left. Due to the given shape’s height, compared to the height of the
figure, you can deduce the vertex you are looking for should be in the middle of the
figure. There are three vertices that match this criterion.

The rightmost and middle vertices form a shape which is similar to the given shape,
but are missing line segments. However, the vertex on the left is part of the given
shape and therefore this is the correct answer.

Answer (A) includes a shape which resembles the given shape, however it is missing
a line segment in order to complete it:

Answer (B) includes two vertices pointing left next to each other, however one of the

152
diagonals which creates them has a different angle than the diagonal of the given
shape:

Answer (D) has several vertices pointing left:


However, the lines forming them have different angles than those of the given shape,
therefore it is also incorrect.

Answer (E) includes only one vertex pointing left in a similar way to the given shape,
therefore it also cannot form the given shape and therefore, this answer choice is
incorrect.

100. Explanation

The correct answer is (E).

The given shape is a right-angled triangle comprised of a vertical, horizontal and a


diagonal line stretching from top to bottom-right. You should look for the 90-degree
angle to rule out answers more easily.

Answer choice (E) includes a square whose diagonal lines intersect and create
right-angled triangles. One of those triangles has the same orientation and size as the
given triangle, making answer choice (E) the correct answer.

Answers (A), (C), and (D) are incorrect because even though there is a 90-degree
angle in the same direction as the given triangle, there are no diagonal sides to
complete the triangle.

Answer (B) is incorrect because it has no 90-degree angle.


Was this explanation helpful?

153
101. Explanation

The correct answer is (D).

The hourglass given is comprised of two isosceles triangles pointing each other.

The bottom triangle in answer (A) is not isosceles, therefore it cannot form the given
shape and it is incorrect.

Answer (B) has both a bottom isosceles triangle and a top isosceles triangle, however,
each of them faces a non-isosceles triangle and therefore it also does not form the
given shape and it is incorrect.

Answer (C) is missing a line segment to form the given shape, as the base of the top
isosceles triangle is missing.

Answer (E), like answer (C), is also missing a line segment to form the given shape,
as the base of the bottom isosceles triangle is missing.

The remaining answer (D) is the correct answer. You can notice that the diagonals on
the right side of the image form two isosceles triangles facing each other and that the
outer rectangle serves as their base, completing the hourglass. Therefore, this is the
correct answer.

154
102. Explanation

The correct answer is (E).

The given shape has a bottom line which is horizontal and vertices which point up-left
and two that point to the right.

Answer (A) is incorrect, as it is not wide enough to contain the given shape.

Both answers (B) and (C) include a shape which resembles the given shape, however
the top vertex pointing right in both answers is made using a horizontal line rather
than a diagonal line as in the given shape.

Answer choice (D) is missing the bottom-left diagonal line to form the given shape
(as shown below), thus it is also incorrect.

The remaining answer choice (E) is the correct answer.

155
You could also solve this question without using answer choice elimination. Answer
(E) includes vertices pointing in the right directions, and most of its outer lines form
the given shape.

103. Explanation
The correct answer is E.

104. Explanation
The correct answer is B.

105. Explanation
The correct answer is C.

156
106. Explanation

The correct answer is (D).

You can picture the given shape as two rectangles, one vertical and the other
horizontal.

Notice the vertical rectangle of the given shape is located in the middle of the
horizontal rectangle. Therefore, our first hint would be to look for the vertical
rectangle within the inner lines of the pictures, rather than in the borders, since
forming the given shape using a vertical rectangle appearing on the borders would
cause part of it to be outside of the borders.

Therefore, the answer must appear in one of two different locations:

157
Answer (D) includes a pair of vertical and parallel inner lines. Those lines are
perpendicular to horizontal lines, and focusing on them reveals that together, they
form the given shape. Therefore, answer (D) is the correct answer.

Answer (A) includes parallel vertical inner lines, as necessary. However, it is missing
a line segment to form the given shape:

Answer (B) has two vertical parallel inner lines. However, those are not in the middle
of a horizontal rectangle and therefore it does not form the given shape and it is
incorrect.

Answer (C) includes vertical inner lines, however only one is long enough to be a side
for a vertical rectangle. Therefore, it does not contain the given shape and it is
incorrect.

Answer (E), like answer (A), is missing a line segment to form the given shape.

107. Explanation
The correct answer is D.

158
108. Explanation

The correct answer is (C).

The shape given resembles a right-angled triangle on top of a rectangle, as seen in the
picture below.

It is characterized by two pairs of parallel lines (two vertical and two horizontal) and a
vertex pointing upwards.

Answer (C) has three horizontal parallel lines (as shown below). The line at the top is
too far away from the line underneath, so it cannot be used to form the rectangle part
of the given shape. However, the line in the middle and the line underneath it has the
correct distance. Moreover, there is a vertex pointing up right above the middle line,
completing the given shape. Therefore, this is the correct answer.

Answer (A) also includes three horizontal parallel lines. And much like answer (C),
the top line is too far from the line underneath it to form the given shape, so you
should try and find the given shape using the middle line and the line in the bottom.
However, there is no right-angled triangle above those lines and therefore they cannot
form the given shape, making answer (A) incorrect.

In answer (B) you can use the vertical parallel lines. There are three of those (as
shown below) and they may be used as sides to form the given shape.

However, those lines either form a square, or a rectangle, which is bigger than the one
in the given shape, and therefore answer (B) is also incorrect.

159
Answer (D) has three vertical lines. Those lines are either too close to each other or
too far from one another to be the sides of the rectangle in the given shape. Therefore,
answer (D) does not contain the given shape and it is incorrect.

Answer (E) includes several horizontal parallel lines. However, none of those lines
have on top of it a right-angled triangle besides one (as shown below).

The line that does, creates the given shape, but in a mirror image (meaning it is
flipped vertically):
As the given shape must keep its original orientation, it does not appear in answer (E)
and answer (E) is incorrect.

109. Explanation
The correct answer is A.

160
110. Explanation

The correct answer is (B).

The given shape is half a star. It has vertices pointing up, right and down-right. In
addition, it has a vertical line.

Answer (B) includes one vertical line and attached to it are lines forming vertices
pointing towards the right. It contains the given shape, making it the correct answer.

Answer (A) has no vertical lines and therefore it cannot contain the given shape,
making it incorrect.

Answer (C) includes a shape which resembles the half-star given. However, the angle
of the vertex pointing bottom-right is different. Moreover, the lengths of the sides
forming the star are different.

Answer (D) has no horizontal lines, and therefore cannot form the given shape (as it
must include a horizontal line), and therefore it is incorrect.

Answer (E) includes a vertical line; however, it is too short to form the given shape.
Therefore, it is also incorrect.

111. Explanation
The correct answer is D.

161
112. Explanation
The correct answer is B.

113. Explanation

The correct answer is (C).

The shape given is a leaf. It has a vertex pointing upwards (Vertex 1 in the picture
below) and another vertex pointing to the left (Vertex 2 in the picture below):

162
The pictures show different flowers to draw attention away from the given shape. The
correct answer must contain the given shape in the same size and direction.

Answer choice (C) includes few vertices pointing upwards. The right most leaf of the
flower also contains a vertex pointing to the left and a closer look shows the given
shape (with a curved line drawn on it). Therefore, it is the correct answer.

Answer choices (A) and (E) have no vertices pointing upwards; therefore, they do not
contain the given shape and are incorrect.

Answer choice (B) does not include vertices pointing to the left and therefore it does
not contain the given shape. Thus, it is incorrect.

Although answer choice (D) does contain a vertex pointing to the left, the shape it
forms is different from the one given, thus making it also incorrect.

The correct answer is (C), which is the only one that remains.

114. Explanation
The correct answer is E.

163
115. Explanation

The correct answer is (E).

The given shape is a kite comprised of two isosceles triangles. It has one horizontal
line going through its middle which serves as a base for the two isosceles triangles.

Answer choice (E) contains few horizontal lines. After ruling out the square’s sides, it
has one horizontal line which is the same length as the horizontal line that appears in
the given shape. Upon examination, you will notice it serves as a base for two
isosceles triangles as needed, thus forming the given shape. Therefore, answer choice
(E) is the correct answer.

Answer choice (A) includes two horizontal lines, one of which is the bottom side of
the rectangle which cannot be used to form the given shape. The other one is a base
for just one isosceles triangle and not two as needed and, therefore, it does not contain
the given shape and is incorrect.

Answer choice (B) contains few horizontal lines, none of which serve as a base for
two isosceles triangles, and therefore it is also incorrect.

Answer choice (C) includes two isosceles but they are pointing towards each other
instead of sharing the base of each other. Thus, they cannot form the given shape,
making this answer also incorrect.

Answer choice (D)’s only horizontal lines are the sides of the square which do not
serve as a base for two isosceles triangles and therefore it is also incorrect.

The remaining answer choice (E) is the correct answer.

116. Explanation
The correct answer is A.

164
117. Explanation
The correct answer is E.

118. Explanation
The correct answer is D.

165
119. Explanation

The correct answer is (C).

The shape given is comprised of straight vertical and horizontal lines. Therefore, it is
possible to deduce that the diagonal lines are meant to draw attention away from the
given shape.

Answer choice (C) depicts a grid and a triangle, although the triangle, which is
comprised of diagonal lines, is only meant as a distraction. The grid itself is
comprised of vertical and horizontal lines which form the given shape, making answer
choice (C) the correct answer.

Answer choices (A), (B), (D) and (E) are missing line segments to form the correct
shape, therefore they are incorrect.

Note: Answer choice (B) includes a shape similar to the one given, but with different
proportions, as shown below.

166
120. Explanation
The correct answer is C.

121. Explanation

The correct answer is (D).

The shape presented includes two parallel edges (sides) that look similar to a
parallelogram. That parallelogram was joined with a triangle on its side:

Notice that one set of the parallel side is horizontal while the other is inclined to the
right.

Answer choice (A) has no parallel lines that are inclined and therefore cannot contain
the given parallelogram, and so this answer is incorrect.

Answer choice (B) has parallel lines that incline similarly to the given shape, however

167
the triangles in the picture are not joined with them, thus it cannot contain the shape,
making this choice incorrect.

Answer choice (C) contains triangles that are different than the one in the given shape
and therefore it also cannot contain the shape and is incorrect.

Answer choice (E) has parallel lines; however, they do not incline with the same
angle at which the given shape inclines, so they cannot be used to create a correct
parallelogram. Therefore, it cannot contain the given shape, making it also incorrect.

122. Explanation
The correct answer is D.

123. Explanation
The correct answer is B.

168
124. Explanation

The correct answer is (C).

The given shape is an arrowhead pointing left. It is comprised of a pair of parallel


vertical lines and two vertices pointing left next to each other.

Answer (C) includes three parallel horizontal lines. The line that goes through the
middle of the trapezoid can be overlooked, as it is too close to the other horizontal
lines in order to form the given shape. In addition, this figure includes diagonals
which incline with angles similar to those of the given shape, and some of them create
vertices pointing left. Due to the given shape’s height, compared to the height of the
figure, you can deduce the vertex you are looking for should be in the middle of the
figure. There are three vertices that match this criterion.

The rightmost and middle vertices form a shape which is similar to the given shape,
but are missing line segments. However, the vertex on the left is part of the given
shape and therefore this is the correct answer.

Answer (A) includes a shape which resembles the given shape, however it is missing
a line segment in order to complete it:

Answer (B) includes two vertices pointing left next to each other, however one of the
diagonals which creates them has a different angle than the diagonal of the given
shape:

169
Answer (D) has several vertices pointing left:
However, the lines forming them have different angles than those of the given shape,
therefore it is also incorrect.

Answer (E) includes only one vertex pointing left in a similar way to the given shape,
therefore it also cannot form the given shape and therefore, this answer choice is
incorrect.

125. Explanation
The correct answer is D.

126. Explanation

The correct answer is (C).

170
The quadrilateral given includes two right angles in its top side and a diagonal line
stretching from its bottom vertex to the right. A good way to find the given shape in
the pictures would be to find a diagonal line which inclines with the same angle as in
the given shape:

Answer choice (C) contains one diagonal line that stretches with a similar angle.
Focusing on the left rectangle, the diagonal stretches from the bottom vertex into its
right side and forms the given shape. Thus, this is the correct answer.
Notice that the left rectangle forms a similar shape, however it is smaller in size than
the given shape.

The diagonal lines in answer choice (A) that stretch from the bottom to the right,
incline at a different angle than the diagonal in the given shape, therefore they cannot
form the given shape, making this answer choice incorrect.

Although answer choice (B) contains a diagonal line which inclines with the correct
angle, it is missing a line segment to form the given quadrilateral. Thus, it does not
contain the given shape and it is incorrect.

Answer choice (D) contains a diagonal line which stretches in the right direction,
however it leans in a different angle, thus it cannot form the given shape. Therefore, it
is also incorrect.

Answer choice (E) contains only two vertical lines, however they are too far apart to
form the given shape, therefore it is also incorrect.

The remaining answer choice (C) is the correct answer.

127. Explanation
The correct answer is C.

171
128. Explanation
The correct answer is A.

129. Explanation

The correct answer is (C).

The given shape is comprised of straight lines and contains a vertex pointing to the
right. Another prominent feature is that two of its sides are parallel vertical lines.

Answer (C) is correct because the given shape is included. You can see this more
quickly by noticing the left half of the shape. The triangle that is formed inside the
shape is meant to distract and conceal the given shape.

Answer (A) is incorrect because the angles of the side lines (the lines to the right of
the shape) are different from the given shape.

Answer (B) contains a similar shape to the one given, but the angle formed in the
rightmost vertex is not the same as the angle in the given shape (furthermore, even if
the angle were the same angle, the dimensions would be bigger than the given shape)

172
Answer choices (D) and (E) include two parallel vertical lines, however they are
closer to each other or further from each other, respectively, than shown in the given
picture. Since the proportions of the given shape need to be kept, they cannot contain
the given shape and they are incorrect as well.

130. Explanation

The correct answer is (E).

Start by analysing the shape you need to identify. Once you know its properties, you
can use these to find the correct answer or rule out other answer choices.

The shape is symmetric with respect to a horizontal line. Furthermore, the shape given
has three pairs of parallel lines (one pair of lines that are horizontal, and the last two
pairs can be found above and below the symmetry line) and the lines in these pairs
have the same length. Finally, the intersecting edges (sides) in the middle of the shape
are directed to the right. Another way to see this is by thinking of the middle vertices
as changing the direction of the lines.

Now that you know the characteristics to look for, examine the answer choices:

Answer (E) is correct because you can see the small shape included here:

Answer (A) is incorrect because it does not include the smaller shape. You can see
this by noticing that no pair of lines that are parallel have the same length, or because
there is no vertex that changes the direction of the side lines as in the original shape.

Answer (B) is incorrect because even though it has many pairs of parallel lines, the
shape is not included inside it. You can see this by focusing on the vertex mentioned
before, which changes the direction of the lines (the intersection of the diagonals) but
the original shape has two such vertices, and answer (B) only has one. Answer (B) is

173
also incorrect because it has no inner parallel lines .

Answer (C) is incorrect because the inside lines intersect at different heights, meaning
it is asymmetric, as opposed to the original shape.

Answer (D) is the most interesting answer among the incorrect ones. This is because
it has parallel sides of equal length and two vertices that change the direction of lines.
However, the lines that change the direction are not parallel to one another as in the
original shape. Therefore, this answer is incorrect.

Solving Tip: Remember that symmetry can always be a helpful tool to find the
correct answer or rule out others. Another tool that will help you is finding parallel
lines and compare them to others to see if they share the same qualities as the ones in
the given shape.
Was this explanation helpful?

131. Explanation
The correct answer is B.

132. Explanation

The correct answer is (B).

174
First, notice the given shape has no diagonal lines. You can deduct that the diagonal
lines in the answer choices are either there to distract, or to cover the correct shape.
The shape given can be seen as two joined rectangles, one of which is vertical and the
other horizontal.

Answer choice (A) contains a vertical rectangle similar to the given shape, however it
has no horizontal rectangle that can complete the shape, thus it is incorrect.

Answer choices (C), (D) and (E) contain a horizontal rectangle in the top part rather
than the bottom, so can also be eliminated.

The only remaining answer choice (B) is the correct answer.

133. Explanation

The correct answer is (E).

The given shape is a parallelogram, characterized by two pairs of parallel sides, one of
which is horizontal, with the other one a diagonal inclining to the right.

Answer choice (E) contains a set of horizontal parallel lines and diagonal parallel
lines that incline to the right. Together they form the given shape, thus making this the
correct answer.

Neither of the rest of the other answer choices contain a pair of diagonal lines which
incline to the right, so they cannot form the given shape, leaving answer choice (E) as
the only possible answer.

134. Explanation

The correct answer is (B).

The shape presented is a rhombus. It is characterized by two pairs of parallel sides and
in this question, it has two vertices pointing upwards and downwards and two vertices
pointing towards the sides. Another way of picturing a rhombus is as two identical
triangles with a joined base line:

175
Answer choice (A) contains several parallel lines, however it does not contain any
vertices pointed upwards or downwards and therefore cannot be the correct answer.

Answer choice (C) contains one triangle pointing left, but its base is joined with a
trapezoid rather than another triangle, and so it is also incorrect.

Answer choice (D) has no triangles and therefore cannot be the correct answer.

Answer choice (E) has one side missing from the rhombus and so is also incorrect.

The answer remaining – answer choice (B) is the correct answer.

135. Explanation

The correct answer is (E).

It is important to remember the answer choice must contain the given shape, while
keeping its original proportions. The given shape includes two vertices pointing
upwards-right:

Answers (A) and (D) are incorrect because they do not contain the given shape. Both
these answers include similar, but not identical shapes.

Answer (B) is incorrect because the angle formed by the vertex at the top is not the
same as the one in the given figure:

Answer (C) contains the given figure but rotated 90 degrees. Therefore, it is also
incorrect.

The remaining answer (E) is the correct answer.

176
136. Explanation

The correct answer is (A).

As you can see in the picture below, the given shape contains two parallel horizontal
lines and 2 vertices pointing to the left as well as a vertical line:

Try to find a characteristic in the given shape that is easy to spot. In this case, the
90-degree angle at the bottom-left corner. Almost immediately after the angle is
formed there is a diagonal line to look for.

Answer choice (A) includes several horizontal lines, and those in the middle of the
shape are at the same distance as those in the given shape. Those lines also connect to
two vertices which point to the left, helping you notice the given shape. Therefore,
this is the correct answer.

Answers (B), (D), and (E) can be ruled out for not having both the properties (in the
case of answer (D), there are no 90-degree angles at all).

Answer choice (C) includes a shape which resembles the given shape, but it has
different proportions, and therefore it is also incorrect.

137. Explanation

The correct answer is (E).

The given shape contains diagonal lines and two parallel horizontal lines.

Answer (A) is missing a line segment to form the top vertex in order to form the given
shape, and therefore it can be ruled out.

177
Answer (B) includes a pair of horizontal lines but is missing a vertex on the right in
order to form the correct shape, making it also incorrect.

Answer (C) has two parallel horizontal lines, however they are too far apart to form
the given shape, thus it is eliminated.

Answer (D) includes a shape similar to the given shape, however its left side has a
different angle than the left side of the given shape (As shown in the picture below).
In addition, the vertex on the right has a different angle than the one in the given
shape. Therefore, it does not include the given shape and it is also incorrect.

138. Explanation
The correct answer is E.

178
139. Explanation

The correct answer is (E).

The given shape is an isosceles trapezoid with a pair of vertical parallel lines and a
pair of congruent diagonal sides.

Answer (E) includes three vertical parallel lines (as shown in the picture below). The
leftmost line is too far from the middle line to form the given shape. However, the
middle line and the rightmost line are close enough, and they are being intersected by
diagonals, as necessary, forming the given shape. Therefore, answer (E) is the correct
answer.

Answer (A) includes several vertical lines, so using them to find the trapezoid might
get tricky. Instead, you can notice the upper outer line of the figure is a diagonal
inclining with the same angle as the upper side of the trapezoid. In order to contain
the shape, the figure must have a diagonal line with the same angle as the bottom line
of the trapezoid close enough to the upper line to form the shape. You can notice that
it only happens in the right side of the picture (as shown in the picture below).
However, it is still missing a line segment to form the given shape, and therefore
answer (A) is incorrect.

Answer (B) does not include a diagonal line with the same angle as the bottom line of
the trapezoid. The figure has only one line stretching from bottom-left to top-right and
its angle is different. Therefore, it is also incorrect.

Answer (C) has only two vertical parallel lines and they are too far apart to form the
given shape, making it also incorrect.

Answer (D) contains one isosceles trapezoid, but it is bigger than the given shape and
therefore it is also incorrect:

140. Explanation
The correct answer is C.

179
141. Explanation

The correct answer is (D).

The shape given is comprised of a rectangle on top of which is attached an isosceles


triangle. The triangle’s vertex is located in the middle of the shape and so it should
appear in the answer choices in the same position. Moreover, the horizontal lines in
the rectangle stretch from one side of the picture to the other, excluding the base of
the triangle, so it should act the same way in the answer choices.

Answer choices (B), (C) and (D) are the only one with vertices in the correct place.
Notice answer choice (A) includes two triangles on the top part. However, neither is
in the correct position. Answer choice (E) includes a right-angled triangle instead of
an isosceles triangle. Therefore, both answer choices (A) and (E) cannot form the
given shape and they are incorrect.

Answer choice (B) is missing a horizontal line segment on the right, therefore it
cannot form the given shape and it is incorrect.

Answer choice (C) is comprised from a triangle on top of a trapezoid instead of a


rectangle and, therefore, it is also incorrect.

The remaining answer choice (D) includes both the vertex needed and the horizontal
lines needed to form the given shape, making it the correct answer.

180
142. Explanation

The correct answer is (C).

In order to find the given shape, you should focus on its prominent features: a pair of
parallel diagonal lines, which also intersects with a line forming 90-degree angles and
a vertex pointing down-left:

Answer (C) includes one vertex pointing in the same direction as in the given shape.
In addition, above that vertex there are two diagonal lines inclining with an angle
similar to the given shape. Connecting these diagonal lines reveals the given shape.
Therefore, this is the correct answer.

Answer (A) has two diagonals which incline with the same angle as in the given
shape, forming a shape similar to it. However, those lines are longer than the diagonal
lines of the given shape, making it longer. Therefore, it does not contain the given
shape and it is incorrect.

Answer (B) includes three diagonal lines which incline the same angle as the lines in
the given shape (as shown in the picture below). However, the bottom one is too short
to be part of the wanted shape and the other two are too far apart from one another.
Moreover, the angle of the triangle pointing down is different than the one in the
given shape. Thus, answer (B) is also incorrect.

Answer (D) has many lines, making it hard to distinguish different shapes. You can
look for the given shape in two ways:
1. Diagonals – Answer (D) includes two diagonals which incline with the same angle
as the diagonals of the given shape. Under those diagonals there is a vertex pointing
down-left as necessary. Together they indicate a shape which resembles the given

181
shape. However, a closer look shows it is shorter than the given shape and therefore
answer (D) is incorrect.
2. Vertex – You could also first look for vertices which point in the right direction
(bottom-left). Once you find the vertex, you look for the diagonals and come to the
same conclusion.

Answer (E) includes one vertex pointing left-down. However, in the given shape, the
angle pointing in the same direction is created by two incongruent lines while in
answer (E) it is formed by two congruent lines. Thus, it does not form the given shape
and answer (E) is incorrect.

143. Explanation

The correct answer is (E).

The given shape has no edges and resembles a stick pointing both upwards-left and
bottom-right.

Answer choice (E) contains few shapes with no edges. The stirring wheel is placed on
a shape which points up-right and bottom-left. This shape is the given shape, making
answer choice (E) the correct answer.

Answer choices (A) and (B) are comprised mostly of straight lines so it is rather
simple to notice they do not contain the given shape, making them incorrect.

Answer choices (C) and (D) have no edges, however, none of the shapes in them have
a shape pointing in the same direction as the given shape, making them also incorrect.

Answer choice (B) includes two parallel diagonal lines which incline at the same
angle as the given shape; however, they are too close to each other to form the given
shape. Therefore, this answer choice is incorrect.

182
Answer choices (C) and (E) do not include a pair of parallel diagonal lines, therefore
they cannot form the given shape meaning they are also incorrect.

144. Explanation

The correct answer is (E).

You should focus on the shape’s outline and find a prominent line or a characteristic
that will help in eliminating answer choices. The shape given can be viewed as
rectangle and a triangle with one line missing from the base of the triangle:

Answer choices (B), (C) and (D) are surrounded by squares. In order to fit into the
squares, the vertical line (side) of the triangle must appear in the top-middle part of
the squares. You can immediately eliminate answer choice (A) which does not
contain a line going through its middle.

Answer choice (B) does not contain any diagonal lines, hence it is also incorrect.

Although answer choice (C) does include a line going through the middle of the
square, it does not stretch to the top part of the picture as necessary and therefore it is
also incorrect.

Answer choice (D) does contain a diagonal line, however it is inclined to the right,
opposing the given shape (in which the triangle diagonal is inclined to the left), and so
it is also incorrect.

The only answer choice remaining is answer choice (E), which is the correct answer.

145. Explanation

The correct answer is (E).

183
The shape given has four sides and four vertices. Prominent characteristics of the
given shape are the horizontal bottom line and the vertex pointing upwards. Another
characteristic would be the smallest side of the given shape which inclines to the right
and, as mentioned, is relatively small.

Answer choice (E) does contain a small line inclined to the right which forms the
given shape, therefore it is the correct answer.

Answer choices (A), (B), (C) do not include small lines that incline in the same
fashion and therefore cannot contain the given shape, making these answer choices
incorrect.

Although answer choice (D) does contain a small line similar to the one that appears
in the given shape, it forms a different shape than the one that is given, thus it is also
incorrect.

146. Explanation

The correct answer is (D).

The given shape is a parallelogram with two vertical parallel sides. Notice that the
vertical parallel lines in answers (A), (B) and (C) are too far from one another to form
the given shape, and thus you can eliminate them.

Answer (E), on the other hand, includes several parallel vertical lines; however, none
of them are intersected by a pair of diagonal parallel lines as in the given
parallelogram. Therefore, it is also incorrect.

The remaining answer (D) is the correct answer. You can notice the parallelogram in
the figures top-left side by observing the vertical parallel lines intersecting with the
parallel diagonals.

147. Explanation
The correct answer is A.

184
148. Explanation
The correct answer is D.

149. Explanation
The correct answer is D.

185
150. Explanation

The correct answer is (D).

The given shape is comprised of two horizontal parallel lines and diagonal lines. In
addition, it includes two vertices pointing to the right:

Answer choice (D) shows a parallelogram whose horizontal parallel lines are at the
same distance from one another as the given shape’s parallel lines. The diagonal sides
of that parallelogram form the same angles as the left side of the given shape (as
shown in the picture below).

However, only the left side of the parallelogram can be used as the left side of the
given shape (as the only other line with the same angle is the right side of the
parallelogram, and it has no lines to its right to form the given shape). Once you focus
on this line, you can notice the diagonal lines to its right form the given shape.

Answer choice (A) does include two vertices pointing to the right, however the
diagonal lines which form them have different angles than the lines of the given shape,
making it unproportionate.

Answer choice (B) includes one diagonal line which inclines with the same angle as
the left side of the given shape, however this line cannot form the given shape, as it
does not have enough lines to its right.

Answer choices (C) and (E) do not include diagonal lines which incline with the same
angle as the left side of the given shape, therefore they do not contain the given shape
and they are also incorrect.

The remaining answer choice (D) is the correct answer.

186
151.Explanation
The correct answer is C.

152. Explanation
The correct answer is C.

153. Explanation
The correct answer is A.

154. Explanation
The correct answer is D.

187
155. Explanation
The correct answer is D.

156. Explanation
The correct answer is B.

188
157. Explanation

The correct answer is (B).

The strong features of the given shape are an isosceles triangle pointing right (without
its base) and a pair of parallel horizonal lines intersected with a perpendicular vertical
line:

Answer (B) uses symmetry to hide the given shape. The rectangle which appears in
the picture has two parallel horizontal lines as necessary, and the rotated square
instead has a vertex pointing to the right. Once you notice this, you can follow the
lines to form the given shape. Therefore, this is the correct answer. Notice that the left
side of the rectangle is the perpendicular vertical line.

Answer (A) includes a shape similar to the given shape, however the triangle shown
in the picture has different angles than those of the given shape, making answer (A)
incorrect.

189
Answer (C) has a pair of parallel horizontal lines which intersect with a vertical line.
However, the left side is not connected to an isosceles triangle pointing to the right, as
needed. It does not contain the given shape and therefore it is incorrect.

Answer (D) includes only one isosceles triangle and it is too big to form the given
shape. Moreover, the upper part of the triangle is not connected with a horizontal line,
as needed, making answer (D) incorrect.

Answer (E) has no vertical lines at all, so it cannot form the given shape. Therefore, it
is also incorrect.

158. Explanation
The correct answer is E.

159. Explanation
The correct answer is B.

190
160. Explanation

The correct answer is (C).

The given shape is a pentagon, which includes a vertex pointing upwards.

Answer choice (C) includes one vertex pointing upwards in the middle of the picture.
Further examination reveals it is the top part of the given pentagon.
The picture below shows that this image actually includes two pentagons combined.

Answer choice (A) includes a shape which resembles a pentagon, however one of its
sides has been detached and changed. It does not contain the given shape and
therefore it is incorrect.

Answer choice (B) and (D) also include shapes which resemble the given pentagon,
however those shapes include a side which is vertical instead of diagonal. Therefore,
they do not include the exact given shape, making them incorrect.

191
Answer choice (E) includes trapezoids and triangles instead of a pentagon, making it
also incorrect.

161. Explanation
The correct answer is C.

162. Explanation
The correct answer is A.

192

You might also like